Sei sulla pagina 1di 118

Copyright © 2014 Delhi Academy of Medical Sciences, All Rights Reserved.

1/118
Test Information
Test Name OLD GT-8-(MD/MS) Total Questions 300

Test Type Examination Difficulty Level Difficult

Total Marks 1500 Duration 180minutes

Test Question Language:- ENGLISH

(1). Nervi hesitans is:

a. Tibial nerve

b. Deep peroneal nerve

c. Common peroneal nerve

d. Femoral nerve

Solution. Ans-1: (b) Deep peroneal nerve


Ref: Read the text below
Sol :
· Deep peroneal nerve lies lateral to anterior tibial artery in the upper part of leg.
· In the middle part it lies anterior, in the lower part, instead of being lateral it again lies medial to it.
· The nerve is trying to cross the artery, hesitating to cross it. Hence known as nervi hesitans.

Correct Answer. b

(2). Structure passing behind the Lateral malleolus is:

a. Peroneus brevis

b. Tibialis posterior

c. Tibialis anterior

d. Extensor hallucis longus

Solution. Ans-2: (a) Peroneus brevis


Ref: Read the text below
Sol :
· The tibialis posterior passes below the flexor retinaaculum, winding around the medial malleolus & is chiefly inserted in the tuberosity
of navicular bone.
· The tendon which winds around lateral malleolus is Peroneus Brevis.
· The tendon of FHL passes below the Sustentaculum talus.

Correct Answer. a

Copyright © 2014 Delhi Academy of Medical Sciences, All Rights Reserved. 2/118
(3). All venous sinuses of the duramater ultimately drain into the:

a. External jugular veins

b. Internal jugular veins

c. Emissary veins

d. Carotid sinus

Solution. Ans-3 : (b) Internal jugular veins


Ref: Read the text below
Sol :
-The venous sinuses of the brain are situated either between the two layers of the duramater or within the reduplications of the
meningeal layer of the dura.
-All sinuses empty directly or indirectly into the beginning of the internal jugular vein located in the jugular foramen.

Correct Answer. b

(4). The major venous return system of the heart, the coronary sinus, empties into the:

a. Inferior vena cava

b. Left atrium

c. Right atrium

d. Superior vena cava

Solution. Ans-4: (c) Right atrium


Ref: Read the text below
Sol :
-With the exception of the anterior surface of the right ventricle, blood returning from the coronary circulation collects in the coronary
sinus which, in turn, empties directly into the right atrium.
-During right ventricular systole, venous blood is stored in the right atrium, and during ventricular diastole, blood flows from the right
atrium to the right ventricle.

Correct Answer. c

(5). Apex of the urinary bladder :

a. Connected to median umbilical ligament

b. Directed upwards

c. Gives opening to urethra

d. All of the above

Solution. Ans-5: (a) Connected to median umbilical ligament


Ref: Read the text below
Sol :
-The apex of the bladder is connected to the umbilicus by the median umbilical ligament which represents the obliterated embryonic
urachus.

Correct Answer. a

Copyright © 2014 Delhi Academy of Medical Sciences, All Rights Reserved. 3/118
(6). Length of the duodenumis:

a. 10 cms

b. 15 cms

c. 25 cm

d. 30 cms

Solution. Ans-6: (c) 25 cm


Ref: Read the text below
Sol :
-The duodenum is a C-shaped tube about 10 inches (25 cm) long that joins the stomach to the jejunum.
-It is very important because it receives the openings of the bile and the pancreatic ducts.
-The duodenum curves around the head of the pancreas.

Correct Answer. c

(7). A patient is unable to taste a piece of sugar cube placed on the anterior part of his tongue. Which cranial nerve is affected?

a. Facial

b. Glossopharyngeal

c. Vagus

d. Hypoglossal

Solution. Ans-7: (a) Facial


Ref: Read the text below
Sol :
-Nerves which supply the sensation and taste of the tongue differ between the anterior and posterior parts.
-Anteriorly, the sensation of tasteis passed along the chorda tympani, a branch of the facial nerve.
-Sensation is passed along the lingual nerve, a branch of the trigeminal nerve.
-Posteriorly, both taste and sensation are passed along the glossopharyngeal nerve.
-All intrinsic and extrinsic muscles of the tongue are supplied by the hypoglossal nerve, with the exception of the palatoglossus. The
palatoglossus is innervated by the vagus nerve

Correct Answer. a

(8). Lumbar plexuspresent in:

a. Anterior part of psoas muscle

b. Anterior part of quadratus lumborum

c. Posterior part of psoas muscle

d. Posterior part of quadratus lumborum

Solution. Ans 8: (c) Posterior part of psoas muscle


Ref– Read the text below
Sol:
· The Lumbar Plexus 136 (plexus lumbalis)—The lumbar plexus is formed by the loops of communication between the anterior divisions
of the first three and the greater part of the fourth lumbar nerves; the first lumbar often receives a branch from the last thoracic nerve.
It is situated in the posterior part of the Psoas major, in front of the transverse processes of the lumbar vertebræ.

Correct Answer. c

Copyright © 2014 Delhi Academy of Medical Sciences, All Rights Reserved. 4/118
(9). At surgery for mastectomy, the surgeon carries the dissection along the major pathway of lymphatic drainage from the mammary
gland.The major lymphatic channels parallel which of the following?

a. Subcutaneous venous networks to the contralateral breast and abdominal wall

b. Tributaries of the axillary vessels to the axillary nodes

c. Tributaries of the intercostal vessels to the parasternal nodes

d. Tributaries of the internal thoracic (mammary) vessels to the parasternal nodes

Solution. Ans 9: (b) Tributaries of the axillary vessels to the axillary nodes
Ref– Read the text below
Sol:
· The lymphatic drainage of the mammary gland, which follows the path of its blood supply, generally parallels the tributaries of the
axillary, internal thoracic (mammary), thoracoacromial, and intercostal vessels.
· Because about 75% of the breast lies lateral to the nipple, the more significant lateral and inferior portions of the breast drain toward
the axillary nodes.
· The smaller medial portion drains to the parasternal lymphatic chain paralleling the internal thoracic vessels, whereas the very small
superior portion drains toward the nodes associated with the thoracoacromial trunk and the supraclavicular nodes.

Correct Answer. b

(10). The smooth muscle of the bronchial airways is innervated by which of the following nerves?

a. Intercostal nerves

b. Phrenic nerves

c. Thoracic splanchnic nerves

d. Vagus nerve

Solution. Ans 10: (d) Vagus nerve


Ref– Read the text below
Sol:
· Innervation of the bronchial smooth muscleis mediated by parasympathetic neurons carried by the vagus nerve.
· These nerves also stimulate secretion from the bronchial glands.
· Excessive vagal activity may initiate bronchospasm or the asthmatic syndrome.

Correct Answer. d

(11). The deep layer of the which ligament support talus:

a. Spring ligament

b. Deltoid ligament

c. Medial plantar ligament

d. Lateral plantar ligament

Solution. Ans 11: (a) Spring ligament


Ref– Read the text below
Sol:
The deep layer of the spring ligament (plantar calcaneonavicular ligament) and the tendon of tibialis posterior support the head of the
talus gliding inwards and under the forefoot, rather like a soft and elastic curtain loop.

Correct Answer. a

Copyright © 2014 Delhi Academy of Medical Sciences, All Rights Reserved. 5/118
(12). The chorionic villi are derived from which of the following?

a. Combination of fetal and maternal tissues

b. Endometrial glands

c. Endometrial stroma

d. Fetal tissues

Solution. Ans 12: (d) Fetal tissues


Ref– Read the text below
Sol:
The placental structures shown in the photomicrograph are chorionic villi that are fetal tissues.
· The mother's contribution to the placenta is the blood that flows past the chorionic villi. A fertilized ovum reaches the uterus about
four days after fertilization. At that time, it has developed into a multicellular, hollow sphere referred to as a blastocyst.
· The blastocyst soon adheres to the secretory endometrium and differentiates into an inner cell mass that will develop into the embryo
and a layer of primitive trophoblast.
· The expanding trophoblast penetrates the surface endometrium and erodes into maternal blood vessels.
· Eventually, it develops two layers, an inner cytotrophoblast and an outer syncytiotrophoblast. Solid cords of trophoblast form the
chorionic villi, which then are invaded by fetal blood vessels.

Correct Answer. d

Copyright © 2014 Delhi Academy of Medical Sciences, All Rights Reserved. 6/118
(13). Somitesare derived from:

a. Paraxial mesoderm

b. Lateral plate mesoderm

c. Intermediate mesoderm

d. Mesoblastic nephroma

Solution. Ans 13: (a) Paraxial mesoderm


Ref– Read the text below
Sol:
· The important components of somitogenesis (somite formation) are periodicity, epithelialization, specification, and differentiation.
· The first somites appear in the anterior portion of the trunk, and new somites “bud off” from the rostral end of the paraxial mesoderm
at regular intervals.
· Somite formationbegins as paraxial mesoderm cells become organized into whorls of cells called somitomeres.
· The somitomeres become compacted and bound together by an epithelium, and eventually separate from the presomitic paraxial
mesoderm to form individual somites.

Correct Answer. a

Copyright © 2014 Delhi Academy of Medical Sciences, All Rights Reserved. 7/118
(14). Stereociliaseen in:

a. Gall bladder

b. Bronchus

c. Proximal PCT

d. Epididymis

Solution. Ans 14: (d) Epididymis


Ref– Read the text below
Sol:
ØStereociliaare apical modifications of the cell, which are distinct from microvilliand cilia.
ØThough their name is more similar to cilia, they are actually more closely related to microvilli, and some sources consider them to be a
variant of microvilli rather than their own distinct type of structure. It is a long projection of cell membrane, similar in structure to
microvillus
ØThey are characterized by their length (distinguishing them from microvilli) and their lack of motility (distinguishing them from cilia).
They are found in three regions of the body:
· the ductus deferens
· the epididymis(see stereocilia (epididymis)for more details)
· the inner ear(see stereocilia (inner ear)for more details)

Correct Answer. d

(15). Brunner’s glandsfound in:

a. Duodenum

b. Jejunum

c. Colon

d. Esophagus

Solution. Ans 16: (a) Duodenum


Ref– Read the text below
Sol:
· Brunner glands(or Pancreal glands) are compound tubular submucosalglandsfound in that portion of the duodenumwhich is above the
sphincter of Oddi.The main function of these glands is to produce an alkaline secretion (containing bicarbonate) in order to:
-protect the duodenumfrom the acidic content of chyme(which is introduced into the duodenum from the stomach);
-provide an alkaline conditionfor the intestinal enzymes to be active, thus enabling absorption to take place;lubricate the intestinal walls.

Correct Answer. a

(16). Which of the following would be found immediately inferior to the left cardinal (lateral cervical) ligament?

a. Ovarian neurovascular bundle

b. Uterine tube

c. Round ligament of the uterus

d. Ureter

Solution. Ans 18: (d) Ureter


Ref– Read the text below
Sol:
· The ureter, lying just medial to the internal iliac artery in the deep pelvis, passes from posterior to anterior immediately inferior to the
lateral cervical ligament.
· This ligament contains the uterine artery and vein to which the ureters pass inferior approximately midway along their course between
internal iliac artery and uterus.
· The ureter continues inferior to the anterior portion of the lateral cervical ligament (where it can sometimes be palpated through the
walls of the vagina at the lateral fornices) to gain access to the base of the urinary bladder.
· The close association between uterine vessels and ureter is of major importance during surgical procedures in the female pelvis.

Correct Answer. d

Copyright © 2014 Delhi Academy of Medical Sciences, All Rights Reserved. 8/118
(17). Joint at kneeis which type of joint:

a. Synarthrosis

b. Symphysis

c. Amphiarthrosis

d. Diarthrosis

Solution.
Ans-19: (d) Diarthrosis
Ref: Read the text below
Sol :
-Diarthrosis differ from the other two joint classification in that the bones are free to move, there is no glue or direct union of bones.
Instead the bones are connected by a joint capsule.
-The joint is enclosed by ligament (outermost layer) then periostium (middle) and finally synovial membrane (innermost).
-Around the ends of the bones themselves is articular cartlage.
-The three layers create a cavity between the articulating bones and in this cavity is synovial fluid, secreted by the synovial membrane.

Correct Answer. d

(18). Length of the fallopian tube is :

a. 5 cms

b. 7 cms

c. 8.5 cms

d. 10 cms

Solution. Ans-20: (d) 10 cms


Ref: Read the text below
Sol :
-There are two uterine tubes; each is about 4 inches (10 cm) long and lies in the upper border of the broad ligament. Each connects the
peritoneal cavity in the regions of the ovary with the cavity of the uterus.
-For purposes of description it is divided into four parts
-The infundibulum
-The ampulla
-The isthmus
-The intra mural part.
-The infundibulum is the funnel – shaped lateral extremity, which projects beyond the broad ligament and overlies the ovary. The free
edge of the funnel is broken up into a number of fingerlike processes, known as fimbriae, which are draped over the ovary.
-The ampulla is the widest part of the tube.
-The isthmus is the narrowest part of the tube and lies just lateral to the uterus.
-The intramural part is the segment that pierces the uterine wall.
-The uterine tube receives the ovum from the ovary and provides a site where fertilization of the ovum can take place (usually in the
ampulla

Correct Answer. d

(19). Groove below the cuboid bone is for the tendon of :

a. Tibialis anterior

b. Tibialis posterior

c. Peroneus longus

d. Peroneus brevis

Solution. Ans-21: (c) Peroneus longus


Ref: Read the text below
Sol :
· The tendon of peroneus longus passes below the brevis, till the cuboid.
· It grooves the undersurface of cuboid & turns at right angle to get inserted on the lateral aspect of the base of the first metatarsal &
medial cuneiform.

Correct Answer. c

Copyright © 2014 Delhi Academy of Medical Sciences, All Rights Reserved. 9/118
(20). Posterior relationship of both kidneys include all except

a. Diaphragm

b. Latissmus dorsi

c. Quadratus lumborum

d. Psoas major

Solution. Ans-22: (b) Latissmus dorsi


Ref: Read the text below
Sol :

· The kidneys are related posteriorly to the following muscles - psoas major, quadratus lumborum & transverse abdominis
· The subcostal nerve & vessels, Iliohypogastric & Ilioinguinal nerves, nedial & lateral arcuate ligaments are also related posteriorly to
the kidneys.

Correct Answer. b

(21). Stomach bed does not include:

a. Left renal vein

b. Splenic artery

c. Left kidney

d. Left crus of diaphragm

Solution. Ans-23: (a) Left renal vein


Ref: Read the text below
Sol :
· Stomach bed includes - Left Kidney, left suprarenal gland, left Crus of diaphragm, spleen, spleenic artery & transverse colon
· The Left renal is not included in stomach bed.

Correct Answer. a

(22). Ligamentum teres is:

a. Obliterated umblical vein

b. Obliterated ductus venosus

c. Thrombosed artery

d. Obliterated umbilical artery

Solution. Ans-24: (a) Obliterated umblical vein


Ref: Read the text below
Sol :
· The obliterated left umbilical vein forms the ligamentum teres.
· The obliterated umbilical artery forms the medial umbilical ligament.
· The obliterated ductus venosus forms Ligamentum Venosum.

Correct Answer. a

Copyright © 2014 Delhi Academy of Medical Sciences, All Rights Reserved. 10/118
(23). Tendon passing below the sustentaculum tali is:

a. Flexor hallucis longus

b. Flexor digitorum longus

c. Tibialis posterior

d. Peroneus brevis

Solution. Ans-25: (a) Flexor hallucis longus


Ref: Read the text below
Sol :
· The tendon of flexor hallucis longus lies below the sustancelum tali. ( part of the calcaneum supporting talus )

Correct Answer. a

(24). A foreign body entering the nose accidently enters:

a. Apical segment of right lower lobe

b. Medial segment of right middle lobe

c. Basal segment of left lung

d. Posterior basal segment of right lower lobe

Solution. Ans-26: (d) Posterior basal segment of right lower lobe


Ref: Read the text below
Sol :
· Trachea continues as Right Primary Bronchus.
· It continues as Secondary bronchus for the lower lobe of the right lung.
· This further continues as the tertiary bronchus for the Posterior Basal segment. Thus any foreign body directly enters the Posterior
basal segment of right lung.
· The most dependent part of the lung in supine position is Apical segment of the lower lobe. Superior Segment).
· This is the segment commonly affected in Aspiration Pneumonia.

Correct Answer. d

(25). For spinal anesthesia lignocaine is used as

a. 0.5 % solution

b. 1 % solution

c. 2 % solution

d. 5% solution

Solution. Ans-271: (d) 5


Ref.: KD Tripathi, 6th edn. Pg 359-361
Sol :
- Lignocaine is used as 5% (heavy) for spinal anesthesia.

Correct Answer. d

Copyright © 2014 Delhi Academy of Medical Sciences, All Rights Reserved. 11/118
(26). An agent added to local anesthetics to speed the onset of action is

a. Methylparaben

b. Bicarbonate

c. Fentanyl

d. Adrenaline

Solution. Ans-272: (b) Bicarbonate


Ref.: Read the text below
Sol :
- Local anesthetics are weak bases. These require penetration inside the neuron for their action. For entry in the neurons local
anesthetics have to cross the neuronal membrane.
- Unionized drugs (lipid soluble) can easily cross the membrane, therefore addition of NaHCO3 in the local anesthetic solution (weak
bases are un-ionized in the alkaline medium) make them rapid acting.
- Adrenaline increases the duration of action by causing vasoconstriction.
- Methylparapben is the preservative added in local anesthetics solution.

Correct Answer. b

(27). One of the following statements about succinylcholine is true

a. It may induce life threatening hyperkalemia

b. It has a long duration of action

c. It is the drug of choice in non traumatic rhabdomyolysis

d. It is useful in patients with spinal cord injuries with paraplegia

Solution. Ans-273: (a) It may induce life threatening hyperkalemia


Ref.: KDT’s 6th Ed. Pg. 344
Sol :
- SCh is the shortest and fastest acting muscle relaxant.
- SCh is contra-indicated in patients with nerve or muscle disorders due to the risk of hyperkalemia
o Nerve disorders : Hemiplegia, paraplegia, Guillain Barre syndrome etc.
o Muscle disorders : Myopathy, myasthenia gravis, rhabdomyolysis, crush injury etc.

Correct Answer. a

(28). Epinephrine added to a solution of lignocaine for a peripheral nerve block will

a. Increase risk of convulsions

b. Increase the duration of action of the local anesthetic

c. Both (a) and (b)

d. None of these

Solution. Ans-274: (b) Increase the duration of action of the local anesthetic
Ref.: Read the text below
Sol :
- Adrenaline and felypressin are the vasoconstrictors that are added to LA solution.
- By causing vasoconstriction, these drugs decrease the systemic absorption resulting in less CNS adverse effects (decreased changes of
seizures).
- Prolong the stay of drug at the site of action resulting in the increase in duration of action of LA.

Correct Answer. b

Copyright © 2014 Delhi Academy of Medical Sciences, All Rights Reserved. 12/118
(29). A depolarizing neuromuscular block may be produced by

a. Atracurium

b. Edraphonium

c. Gallamine

d. Suxamethonium

Solution. Ans-275: (d) Suxamethonium


Ref.: KD Tripathi, - 339
Sol :
- Suxamethonium is available as the chloride, bromide, or in some countries, as the iodide salt, and is dissolved in sterile water for
injection. It is also known as sucinylcholine.
- After administration it mimics the action of acetylcholine at the motor end plate, resulting in depolarization of the muscle membrane.
This is seen as fasciculation’s.
- Unlike acetylcholine, suxamethonium is not hydrolysed by acetylcholinesterase and depolarization persists for several minutes,
preventing further muscle activity.

Correct Answer. d

(30). N-glycosylation of proteins occurs on which of the following amino acids ?

a. Asparagine

b. Aspartate

c. Lysine

d. Serine

Solution. Ans-49: (a) Asparagine


Ref:Read the text below
Sol:
· After proteins are synthesized (translated), many are glycosylated in the lumen of the endoplasmic reticulum and the Golgi complex.
· Oligosaccharides are attached to proteins by N-glycosylation of asparagine side chains or by O-glycosylation of serine and threonine
side chains.
· The transfer of oligosaccharides is mediated by an activated lipid carrier, dolichol phosphate.

Correct Answer. a

(31). Geometric possibilityfor a particular set of atoms is known as:

a. Configuration

b. Conformation

c. Orientation

d. Alignment

Solution. Ans: 50(a) Configuration


Ref– Read the text below
Sol
Configuration: geometric possibilities for a particular set of atoms. In changing configuration, covalent bonds must be broken. A
particular stereochemistry about a given center is considered to be a configuration. The primary sequence of a protein is a
configuration.
Conformation: 3-D architecture of a protein. It is established by a variety of weak forces. In contrast to configuration,a conformation can
change readily.
The conformation of a protein depends on:
· Its primary structure (amino acid sequence).
· Its interaction with solvent (generally H2O)
· pH and ionic composition

Correct Answer. a

Copyright © 2014 Delhi Academy of Medical Sciences, All Rights Reserved. 13/118
(32). Karyotyping is done for

a. Chromosomal anomalies

b. X linked disorder

c. Autosomal disorder

d. Mitochondrial inheritance

Solution. Ans: 51(a) Chromosomal anomalies


Ref– Harrison 18thed/Pg 512
Sol
· Spectral karyotyping (SKY) and multicolor FISH (m-FISH)techniques use combinatorially labeled probes that create a uniquecolor for
individual chromosomes. This technology is useful in theidentification of unknown chromosome material (such as accessorymarker
chromosomes containing additional material) but has beenmost commonly used with the complex rearrangements seen incancer
specimens.

Correct Answer. a

(33). NADPH is usedin ?

a. Fatty acid synthesis

b. Ketone synthesis

c. Gluconeogensis

d. Glycolysis

Solution. Ans-52: (a) Fatty acid synthesis


Ref:Read the text below
Sol :
-NADPH is the source of reducing equivalentfor fatty acid synthesis.
-This NADPH is derived from 3 sources : A. HMP shunt pathway B. Isocitrate dehydrogenase, C. Malic enzyme.
Ketone body synthesis, gluconeogenesis utilize NADH and glycolysis yields NADH.

Correct Answer. a

(34). A competitive inhibitor of an enzyme

a. Increases K. but does not affect Vm

b. Decreases Km but does not affect Vm

c. Increases Vm but does not affect Km

d. Decreases Vm but does not affect Km

Solution. Ans 53 : (a) Increases K. but does not affect Vm .


Ref– Read the text below
Sol:
· Competitive inhibitorcompetes with the substrate for the active site of the enzyme, in effect increasing the Km.
· As the substrate concentration is increased, the substrate, by competing with the inhibitor, can overcome its inhibitory effects, and
eventually the normal Vm is reached.

Correct Answer. a

Copyright © 2014 Delhi Academy of Medical Sciences, All Rights Reserved. 14/118
(35). The biosynthesis of the enzyme pyruvate carboxylase is repressed by ?

a. Insulin

b. Cortisol

c. Glucagon

d. Epinephrine

Solution. Ans-54: (a) Insulin


Ref:Read the text below
Sol :
-Pyruvate carboxylaseis gluconeogenic hormone.
-Insulin inhibits gluconeogenesis and has got inhibitory effect on pyruvate carboxylase.

Correct Answer. a

(36). Production of which of the following proteins would be most directly affected in scurvy?

a. Myoglobin

b. Collagen

c. Insulin

d. Hemoglobin

Solution. Ans 55 : (b) Collagen .


Ref– Read the text below
Sol:
· Scurvy is caused by a deficiency of vitamin C. The hydroxylation of proline and lysine residues in collagen requires vitamin C and
oxygen.
· Globin synthesismight be indirectly affected because absorption of iron from the intestine is stimulated by vitamin C.
· Ironis involved in heine synthesis, which regulates globin synthesis.

Correct Answer. b

(37). Protein purification and separation can be done by all except ?

a. Chromatography

b. Centrifugation

c. Electrophoresis

d. Densitometery

Solution. Ans-56: (d) Densitometery


Ref:Read the text below
Sol :
-Densitometery is the procedure which is used to measure the quantity of the protein ?
-Remember : For purification and separation following methods are used.
1.Chromatography
2.Dialysis
3.Centrifugation
4.Electrophoresis
5.Precipitation

Correct Answer. d

Copyright © 2014 Delhi Academy of Medical Sciences, All Rights Reserved. 15/118
(38). Method of separation of Proteins using ammonium sulphate solution is:

a. Ion exchange chromatography

b. Thin layer chromatography

c. Affinity chromatography

d. Salting in and salting out

Solution. Ans 57: (d) Salting in and salting out


Ref– Wilson & Walker Biochemistry and Molecular Biology, 7thed/Pg 454
Sol
· Since HIC requires the presence of salting-out compounds such as ammonium sulphate to facilitate the exposure of the hydrophobic
regions on the protein molecule,it is commonly used immediately after fractionation of protein mixtures with ammonium sulphate.

Correct Answer. d

(39). Which of the following is most abundant secondary structure in globular proteins?

a. Helix

b. Sheets

c. Loops

d. Strands

Solution. Ans 58: (a) Helix


Ref– Read the text below
Sol
Helices are the most abundant form (32-38% of residues in globular proteins)
ALPHA-HELIX - 32-38 % of residues
3.10 HELIX - 3.4 % of residues
PI-HELIX - Extremely rare
Glycine and Proline are unfavorable for conformational stability of peptide

Correct Answer. a

(40). VMA is excreted excessively in the urine. What may be the diagnosis ?

a. Carcinoid syndrome

b. Phaeochromocytoma

c. Cushing’s syndrome

d. None

Solution. Ans-59: (b) Phaeochromocytoma


Ref:Read the text below
Sol :
-VMA is end product of catabolism of catecholamines.
-In phaeochromocytoma and neuroblastoma there is excessive synthesis of catecholamines which causes enhanced synthesis of VMA and
it’s excretion in the urine.

Correct Answer. b

Copyright © 2014 Delhi Academy of Medical Sciences, All Rights Reserved. 16/118
(41). Each of the following enzymes is involved in the synthesis of serine from glucose except

a. Phosphofructokinase

b. Aldolase

c. Phosphoserine phosphatase

d. Pyruvate kinase

Solution. Ans 60 : (d) Pyruvate kinase


Ref– Read the text below
Sol:
· Serine is synthesized from glucose. The pathway branches from glycolysis at phosphoglyceric acid, which is reduced, transaminated,
and dephosphorylated by phosphoserine phosphatase.
· Pyruvate kinase is a glycolytic enzymethat functions beyond the branch point for serine synthesis.

Correct Answer. d

(42). FIGLU is a metabolite of ?

a. Folic acid

b. Tyrosine

c. Histidine

d. Alanine

Solution. Ans-61: (c) Histidine


Ref:Read the text below
Sol :
-FIGLU is a intermediate metabolite of histidine catabolism.
-It is excreted in urine in increased quantity when there is folic acid deficiency as folic acid is required for further metabolism of FIGLU.

Correct Answer. c

(43). False about tryptophan?

a. Nonessential amino acid

b. Involved in serotonin synthesis

c. Involved in niacin synthesis

d. Involved in melatonin synthesis

Solution. Ans-62: (a) Nonessential amino acid


Ref:Read the text below
Sol :
-Tryptophan is an essential amino acid, as it’s synthesis in body does not take place.
-Tryptophan is involved in synthesis of melationin, serotonin, and niacin by various pathways.

Correct Answer. a

Copyright © 2014 Delhi Academy of Medical Sciences, All Rights Reserved. 17/118
(44). Kynurenine is formedfrom ?

a. Phenylalanine

b. Tryptophan

c. Tyrosine

d. Histidine

Solution. Ans-63: (bTryptophan


Ref:Read the text below
Sol :
-Kynurenin is synthesized fromtryptophan as a intermediate of it’s metabolic pathway.
-This pathway is also known as kynurenine pathway.

Correct Answer. b

(45). In arginase enzyme defect of urea cycle the amino acid excreted in the urine is ?

a. Lysine

b. Cystein

c. Both A and B

d. Arginine

Solution. Ans-64: (c) Both A and B


Ref:Read the text below
Sol :
-In arginase defect in urea cycle arginine is accumulated which leads to excessive accumulation of arginine in the plasma and it’s
excretion in the urine initially in the PCT of the kidney but in the DCT this arginine is reabsorbed in exchange for lysine and cystine
which then gets excreted in the urine.
-So in arginase defect amino acid accumulated is arginine but the one that is excreted is lysine and cystein.

Correct Answer. c

(46). Pyruvate can be converted directlyto all of the following except?

a. Phosphoenol pyruvate

b. Alanine

c. Acetyl Co A

d. Lactate

Solution. Ans-65: (a) Phosphoenol pyruvate


Ref.:Read the text below
Sol :
-For the conversion of pyruvate to phosphoenolpyruvate, pyruvate has to undergo carboxylation and decarboxylation reactions by
pyruvate carboxylase and phosphoenol pyruvate carboxykinase respectively..
-Pyruvate may get converted to alanine by transamination reaction.
-Pyruvate may get converted to acetyl Co A by Pyruvate Dehydrogenase Complex (PDH complex)
Pyruvate may get converted to lactate by LDH enzyme.

Correct Answer. a

Copyright © 2014 Delhi Academy of Medical Sciences, All Rights Reserved. 18/118
(47). Insulin causes lipogenesisby all except?

a. Increasing acetyl Co A carboxylase activity

b. Increases the transport of glucose in to the cell

c. Inhibits PDH

d. Decreases intracellular cAMP level

Solution. Ans-66 : (c) Inhibits PDH


Ref.:Read the text below
Sol :
-Insulin stimulates activity of PDH complex and thus enhances oxidative decarboxylation of pyruvate converting it to acetyl Co A.
-During lipogenesis insulin stimulates acetyl Co A carboxylase activity and thus stimulates lipogenesis.
-Insulin increases transportation of glucose in to the cell by enhancing the recruitment of GLUT to the cell membrane.
-Insulin decreases level of cAMP in the cell by inhibiting the activity of adenylyl cyclase enzyme.

Correct Answer. c

(48). Substrate level phosphorylation in TCA cycle occurs at?

a. Succinyl Co A to succinate

b. Fumarate to malate

c. Succinate to fumarate

d. Acetoacetate to a KG

Solution. Ans-67: (a) Succinyl Co A to succinate


Ref.:Read the text below
Sol :
-In TCA cycle there occurs substrate level phosphorylation only once at the step catalysed by succinate thiokinase which produces ATP in
most of the cells.
-GTP at this step is produced in few of the cycle in gluconeogenic organs, i.e. liver and kidney.

Correct Answer. a

(49). All the following TCA cycle intermediate may be added or removed by other metabolic pathways except?

a. Citrate

b. Fumarate

c. Isocitrate

d. Alpha ketoglutarate

e. Oxaloacetate

Solution. Ans-68: (c) Isocitrate


Ref.:Read the text below
Sol :
-Citrate is transported out of the mitochondria to be used as a source of cytoplasmic acetyl Co A.
-Fumarate is produced during degradation of tyrosine and phenylalanine.
-Alpha ketoglutarate can be formed from glutamate.
-Oxaloacetate may be produced from pyruvate during gluconeogenesis.

Correct Answer. c

Copyright © 2014 Delhi Academy of Medical Sciences, All Rights Reserved. 19/118
(50). Up to one third of the amino acid residues of collagen are

a. Serine

b. Hydroxyproline

c. Proline

d. Glycine

Solution. Ans 69: (d)Glycine


Reference: Read the text below
Sol :
· Collagens are a unique class of proteins in several respects.
· The amount of glycine in collagen is unusually high, composing as much as 35% of the amino acid residues.
· In addition, collagen contains two other amino acids that are present in few other proteins found in nature, hydroxyproline and
hydroxylysine.
· These residues are important in the cross-linking of collagen fibers.

Correct Answer. d

(51). Which of the following compounds is an essential fatty acid?

a. Oleic acid

b. Stearic acid

c. Linoleic acid

d. Palmitoleic acid

Solution. Ans 70: (c)Linoleic acid


Reference: Read the text below
Sol :
· Mammals lack the ability to synthesize linoleic or linolenic acid.
· Because these compounds serve as the precursors for the synthesis of other fatty acids they must be provided in the diet.

Correct Answer. c

(52). In the posttranslational modification of collagen, ascorbate (vitamin C) acts as

a. A carrier of amino acids

b. A reducing agent

c. A carrier of hydroxyl groups

d. An enzyme

Solution. Ans 71: (b)A reducing agent


Reference: Read the text below
Sol :
· Ascorbate (vitamin C) is a cofactor necessaryfor the action of the enzymes prolyl hydroxylase and lysyl hydroxylase, which catalyze the
posttranslational modification of proline and lysine in collagen to hydroxyproline and hydroxylysine.
· At their active sites these enzymes contain a ferrous atom, which must be kept in a reduced state. Because ascorbate is used as a
reducing agent, it activates the enzymes.
· A deficiency of vitamin C leads to scurvy, a disease characterized by skin lesions and blood vessel fragility. Citrus fruits, which are rich
in vitamin C, supply a sufficient amount of the vitamin when eaten regularly.

Correct Answer. b

Copyright © 2014 Delhi Academy of Medical Sciences, All Rights Reserved. 20/118
(53). Which of the following enzymes may be targets of a new drug that specifically inhibits retroviral replication?

a. DNA-dependent DNA polymerase

b. Topoisomerase II

c. RNA-dependent DNA polymerase

d. RNA polymerase

Solution. Ans-72: (c) RNA-dependent DNA polymerase


Ref:Read the text below
Sol:
· RNA-dependent DNA polymerase synthesizes DNA from an RNA template and is essential for the replication of retroviruses (but not
for cells) and therefore is a potential target of the new drug.
· Topoisomerase II relaxes supercoiled DNA.
· RNA polymerase synthesizes a primer fragment for DNA-dependent DNA polymerase.
· DNA ligase anneals the Okazaki fragments on the lagging strand of DNA synthesis.

Correct Answer. c

(54). Light house sign seen in

a. Serous otitis media

b. ASOM

c. CSOM

d. Otic hydrocephalus

Solution. Ans-161: (b) ASOM


Ref:Read the text below
Sol:
· In acute suppurative otitis media discharge may be seen extruding from a small perforation in the pars tensa portion of the ear drum.
· This is known as the light house sign.

Correct Answer. b

(55). Ideal time for treatment of hearing loss for language development is..

a. 6months

b. 1 year

c. 2 year

d. 3 year

Solution. Ans-162: (a) 6months


Ref:Read the text below
Sol:
· The deaf children of hearing parents may not have significant exposure to any language in early childhood. Because of their sensory
loss, these children perceive little of their parents’ speech.
· It is a goal for some audiologists to test and fit a deaf child with a cochlear implant by six months of age, so that they don't get behind
in learning language.
· Deaf children do not acquire speech the same as hearing children because they cannot hear the language spoken around them. In
normal language acquisition, auditory comprehension precedes the development of language.
· Without auditory input, a person with prelingual deafnessis forced to acquire speech visually through lip-reading. Acquiring spoken
language through lip-reading alone is challenging for the deaf child because it does not always accurately represent speech sounds.
· The likelihood of a deaf child successfully learning to speak is based on a variety of factors including: ability to discriminate between
speech sounds, a higher than average non-verbal IQ, and a higher socioeconomic status.
· Despite being fitted with hearing aids or provided with oral instruction and speech therapy at a young age, prelingually deaf children
are unlikely to ever develop perfect speech and speech-reception skills.
· Some researchers conclude that deaf children taught exclusively through spoken language appear to pass through the same general
stages of language acquisition as their hearing peers but without reaching the same ultimate level of proficiency.

Correct Answer. a

Copyright © 2014 Delhi Academy of Medical Sciences, All Rights Reserved. 21/118
(56). Chronic sinusitis is defined aswhen the signs and symptoms last for more than

a. 3 weeks

b. 4 weeks

c. 6 weeks

d. 12 weeks

Solution. Ans-163: (d) 12 weeks


Ref:Read the text below
Sol:
Sinusitis (or rhinosinusitis) is defined as an inflammation of the mucous membrane that lines the paranasal sinuses and is classified
chronologically into several categories:[2]
· Acuterhinosinusitis — a new infection that may last up to four weeks and can be subdivided symptomatically into severe and non-
severe;
· Recurrent acute rhinosinusitis — four or more separate episodes of acute sinusitis that occur within one year;
· Subacuterhinosinusitis — an infection that lasts between four and 12 weeks, and represents a transition between acute and chronic
infection;
· Chronicrhinosinusitis — when the signs and symptoms last for more than 12 weeks; and
Acute exacerbation of chronic rhinosinusitis — when the signs and symptoms of chronic rhinosinusitis

Correct Answer. d

(57). Management of maxillary carcinoma ?

a. Radiotherapy

b. Chemotherapy

c. Surgery + Radiotherapy

d. Surgery + Chemotherapy

Solution. Ans-164: (c) Surgery + Radiotherapy


Ref:Read the text below
Sol:
Treatment modalitites available:
1.Surgery
2.Radiotherapy
3.Chemotherapy
4.Combined
management modality
· If the tumor is confined to the inferior portion of the maxilla the condition is best managed by partial maxillectomy followed by
irradiation.
· Tumor involving the whole of the maxilla can be managed by total maxillectomy followed by irradiation.
· Involvement of orbit can be managed by combining orbital exenteration along with total maxillectomy.
· Tumors of maxilla extending to infratemporal fossa can be managed by extended maxillectomy using Barbosa technique.
· Maxillectomy is combined with condylectomy and resection of pterygoid plate and muscles attached to it.

Correct Answer. c

Copyright © 2014 Delhi Academy of Medical Sciences, All Rights Reserved. 22/118
(58). Crooked noseinvolves

a. Septum

b. Dorsum

c. Both septum and dorsum

d. None

Solution. Ans-165: (c) Both septum and dorsum


Ref:Read the text below
Sol:
· A crooked nose bends to one side or the other, lacking a straight bridge.
· These noses can have either a curved or bumpy appearance, and can also be too big or too small.
· A crooked nose can be caused by a deviated septum or an injury that has broken the bones.
· To straighten the nose, the bones need to be moved into a better position and are often reduced before being reshaped.

Correct Answer. c

(59). Ohngren’s classification of maxillary sinus carcinomais based on

a. Imaginary plane between medial canthus of eye and angle of mandible.

b. Imaginary plane between lateral canthus of eye and angle of mandible.

c. Two horizontal lines, one passing through floor of obit and other through floor of antrum

d. none

Solution. Ans-166: (a) Imaginary plane between medial canthus of eye and angle of mandible.
Ref.:Read the text below
Sol :
· In head and neck cancer, Ohngren's line is a line that connects the medial canthus of the eye to the angle of the mandible.
· The line defines a plane orthogonal to a sagittal plane that divides the maxillary sinus into (1) an anterior-inferior part, and (2)
a superior-posterior part.
· Tumours that arise in the anterior-inferior part, i.e. below Ohngren's line, generally have a better prognosis than those in the other
group

Correct Answer. a

(60). Vidian neurectomydone in:

a. Allergic rhinitis

b. Vasomotor rhinitis

c. Atrophic rhinitis

d. Drug-induced rhinitis

Solution. -NA-

Correct Answer. b

Copyright © 2014 Delhi Academy of Medical Sciences, All Rights Reserved. 23/118
(61). During Tonsillitis, pain in the ear is due to involvement of?

a. Vagus Nerve

b. Chorda tympani Nerve

c. Glossopharyngeal Nerve

d. Hypoglossal Nerve

Solution. Ans-168: (c) Glossopharyngeal Nerve


Ref.:Read the text below
Sol :
· The tympanic nerve (Jacobson nerve, a branch of cranial nerve IX) directly innervates the ear but also has pharyngeal, lingual, and
tonsillar branches to supply the posterior one-third portion of the tongue, tonsillar fossa/pillars, pharynx, eustachian tube

Correct Answer. c

(62). Phlep sign is seen in?

a. Otosclerosis

b. Glomus tumors

c. Meniere disease

d. CSOM

Solution. Ans-169: (b) Glomus tumors


Ref.:Read the text below
Sol :
PHELP'S SIGN - loss of crust of bone between carotid canal and jugular canal in glomus jugulare

Correct Answer. b

(63). Organism responsible for Rhinosporidiosis is?

a. Fungus

b. Bacteria

c. Protozoa

d. Virus

Solution. Ans-170: (c) Protozoa


Ref.:Read the text below
Sol :
· Rhinosporidiosis and its causative pathogen Rhinosporidium seeberi have been known for over a hundred years.
· Recent work included in this review are molecular biological classification of R.seeberiamong the hydrophilic organisms of the
former DRIP clade, establishment of a method for the purification of the developmental stages, and some aspects of the immunology
of R.seeberi with reference to mechanisms of immune evasion - antigenic variation, host immunoglobulin binding, immune deviation in
relation to the chronicity, recurrence and dissemination seen in rhinosporidiosis.
· The mechanism of endospore release from the sporangium has been described. Some problems involved in the resolution of enigmas
that persist are briefly discussed.
· Moreover, it shows morphological featuresresembling those of protozoa

Correct Answer. c

Copyright © 2014 Delhi Academy of Medical Sciences, All Rights Reserved. 24/118
(64). Frontal sinuses appear radiologically at the age of ?

a. 2

b. 4

c. 6

d. 12

Solution. Ans-171: (c) 6


Ref: Readthe text below
Sol:
RADIOLOGICAL APPEARANCE OF SINUSES
· MAXILLARY---4 MTHS
· FRONTAL--- 6 YRS
· SPHENOID--- 4 YRS
· ETHMOIDAL--- 1 YRS

Correct Answer. c

(65). Ciliary movement rate of nasal mucosais

a. 1-2 mm/min

b. 2-5 mm/min

c. 5-10 mm/min

d. 10-12 mm/min

Solution. Ans-172: (c) 5-10 mm/min


Ref: Readthe text below
Sol:
· Ciliary movement rate of nasal mucosais about 5-10 mm/min

Correct Answer. c

(66). Longest part of fallopian tube is :

a. Isthmus

b. Interstitial

c. Ampulla

d. Fimriae

Solution. Ans-239: (c) Ampulla


Ref.:Read the text below
Sol :
Parts of the falloplan Tube :
-There are four parts, from medical to lateral they are :
-Intramural or interstitial lying in the uterine wall and measures 1.25 cm(1/2”) in length and 1 mm in diameter.
-Isthmus almost straight and measures about 2.5 cm (1”) in length and 2.5 mm in diameter.
-Ampulla – tortuous part and measures about 5 cm (2”) in length which ends in wide.
-Influndibulum measuring about 1.25 cm (1/2”) long with a maximum diameter of 6 mm.

Correct Answer. c

Copyright © 2014 Delhi Academy of Medical Sciences, All Rights Reserved. 25/118
(67). What is “Safe period”?

a. 5th day of menstruation to 16th day of cycle

b. 1 week before menstruation and 1 week from the 1st day of menstruation

c. 14-18 day of cycle

d. None of the above

Solution. Ans-240: (b) 1 week before menstruation and 1 week from the 1st day of mentstruation
Ref.:Read the text below
Sol :
· The safe period refers to the window period where a woman is less likely to get pregnant if they have unprotected intercourse.
Technically speaking, this safe period falls between day 1-7 of the period and Day 19 to the start of the next cycle
· The normal woman’s menstrual cycle is supposed to last 28 days. Of these only days 8 to 19 are considered fertile i.e. a woman is likely
to have a baby through unprotected intercourse, all other days are safe. The cycle is considered to begin with the first day of bleeding
and ends when the bleeding starts again.

Correct Answer. b

(68). All the following are causes of disseminated Intravascular Coagulation (DIC) except

a. Intrauterine death

b. Missed abortion

c. Abruption placenta

d. Multiple pregnancy

Solution. Ans-241: (d) Multiple pregnancy


Ref:Read the text below
Sol :

Obstetric condition and trigger factors for DIC


Endothelial injury Release of thromboplastin Release of phospholipid
- Pre-eclampsia, Eclampsia Amniotic fluid embolism Fetomaternal blood
Incompatible blood
- Septicemia Abruptio placentae transfusion
-Septic abortion Hydatidiform mole Hemolysis
-Chorioamnionitis Cesarean section
-Pyeloneprhitis Intra-amniotic hypertonic
-Hypovolemia saline

Correct Answer. d

(69). Urine formation in intra uterine life starts at

a. 3 months

b. 4 months

c. 5 months

d. 6 months

Solution. Ans-242: (a) 3 months


Ref:Read the text below
Sol :
· ‘’ The fetal kidneys start producing urine at 12 weeks.
· By 18 weeks they are producing 7-14ml/day and this increases to 37 ml/hour or 650 ml/day at term’’.

Correct Answer. a

Copyright © 2014 Delhi Academy of Medical Sciences, All Rights Reserved. 26/118
(70). Molar pregnancy is diagnosed best by

a. Clinical examination

b. Ultrasound

c. β hCG

d. None of above

Solution. Ans-243: (b) Ultrasound


Ref:Read the text below
Sol :
· The simplicity , safety and reliability of ultrasonography define it as the diagnostic method of choice for patients with suspected molar
pregnancy.
· Absent gestation sac and absent fetus, ‘snowstorm’ appearance may be seen.

Correct Answer. b

(71). Sizeof Mature graffian follicle preovulatory is

a. 12 mm

b. 14 mm

c. 18 mm

d. 25 mm

Solution. Ans-244: (c) 18 mm


Ref:Read the text below
Sol :
· Ovulation is a processwhereby a secondary oocyte is released from the ovary following rupture of a mature Graafian follicle and
becomes available for conception.
· There is a prevulatory enlargement of the Graafian due to accumulation of follicular fluid and measures about 20mm in diameter.

Correct Answer. c

(72). Incidence of Ectopic pregnancy is more than with

a. OC pills

b. CuT200

c. Progestasert

d. Multiload IUCD

Solution. Ans-245: (c) Progestasert


Ref:Read the text below
Sol :
· Ectopic pregnancy is more likely to occur in women taking progestogen-only pills, but not in those taking combined OC pills.
· Progestasert is a third generation intrauterine contraceptive device (IUD) which is a T shaped device filled with 38 mg of
progenterone. The hormone is released slowly in the uterus at the rate of 65 mcg daily.
· The ectopic pregnancy rate per 1000 women years in a levonorgestrel (IUD) and Tcu380A is about 0.2.
· With progesterone IUD it is higher – about 6.8 because its action is limited to a local effect on the endometrium. Ther is an increased
incidence of ovarian pregnancy especially with progetasert device.
· Third generation of IUDs like CuT 380A and LNG-IUS give some amount of protection against an ectotic pregnancy.

Correct Answer. c

Copyright © 2014 Delhi Academy of Medical Sciences, All Rights Reserved. 27/118
(73). Granulose cell produce estrogen with the help of

a. Alkaine phosphatase

b. Aromatase

c. Acid phosphatase

d. Glucuronidase

Solution. Ans-246: (b) Aromatase


Ref:Read the text below
Sol :
· The naturally occurring estrogens are 17b – estradiol, estrone and estriol.
· They are secreted primarily by the granulose cells of the ovarian follicles, the corpus luteum and the placenta.
Their biosynthesis depends on the enzyme aromatase which converts testosterone to estradiol and androstenidione to estrone. The latter
reaction also occurs in fat, liver, muscle and brain.
· ‘’The mural granulose cells are particularly rich in aromatase and under, the control of FSH, produce estradiol, the primary steroid
secreted from the follicular phase ovary and the most potent estrogen”.

Correct Answer. b

(74). Decubitus ulcer in prolapse uterusis due to

a. Friction

b. Venous congestion

c. Trauma

d. Malignancy

Solution. Ans-247: (b) Venous congestion


Ref:Read the text below
Sol :
· Ulceration of the prolapsed uterine tissue is said to be caused by friction which is not wholly true, because the ulcer is seen on the
dependant portion and not on the sides where friction is maximum.
· The most common reason for decubitus ulcer is congestion and circulatory changes in the dependent part of the prolapse.
· The reduction of the prolapsed into the vagina and daily packing heals the ulcer in a week or two.

Correct Answer. b

(75). Contraceptives contraindicated in HIV positive patient is

a. IUCD

b. OC pills

c. Mini pills

d. Barrier methods

Solution. Ans-248: (a) IUCD


Ref:Read the text below
Sol :
Condoms are given prevention of transmission.

Correct Answer. a

Copyright © 2014 Delhi Academy of Medical Sciences, All Rights Reserved. 28/118
(76). Abruption of placenta occurs in all except

a. Smokers

b. Folic acid deficiency

c. Alcoholics

d. PIH

Solution. Ans-249: (c) Alcoholics


Ref:Read the text below
Sol :

Prevalence of abruption placenta is more with

· High birth order pregnancies- gravinda 5 and above. · Sudden uterine decompression
· Advancing age of the mother · Short cord
· Poor socioeconomics condition · Supine hypotension syndrome
· Malnutrition, smoking · Folic acid deficiency
· Tendency of recurrence in next pregnancy is 10 fold. · Torsion of uterus
· Pre-eclampsia · Sick placenta
· Trauma · Cocaine abuse
· Thromobophilas

Correct Answer. c

(77). A 26-years old pregnant woman has been detected to have a diastolic murmur in mitral area. Echocardiography revels mitral valve orifice
to be 0.8 cm square. The cause of her murmur is

a. Mild mitral stenosis

b. Severe mitral stenosis

c. Functional murmur

d. Transmitted murmur from the congenital heart disease of the fetus.

Solution. Ans-250: (b) Severe mitral stenosis


Ref:Read the text below
Sol :
· Mitral stenosis is the most commonly encountered heart disease during pregnancy.
· The mitral valve orifice is normally about 5 cm square (4-6) in diameter and may be reduced to 1 cm square or less in severe mitral
stenosis.
· Symptoms appear when the stenosis is approximately 2.5 cm square or less.

Correct Answer. b

Copyright © 2014 Delhi Academy of Medical Sciences, All Rights Reserved. 29/118
(78). All the following are signs of intra uterine fetal death except

a. Spalding’s sign

b. Hegar’s sign

c. Robert’s sign

d. Halo sign

Solution. Ans-251: (b) Hegar’s sign


Ref:Read the text below
Sol :
Sign of Intra Uterine fetal death
· Spalding’s sign - overlapping of fetal skull bones.
· Robert’s sign –gas in the fetal abdomen.
· Halo sign –elevation of the subcutaneous fat layer over the fetal skull in a dead or dying fetus ; said to be the most common radiologic
sign of fetal death.
· Hegar’s sign –sign is based on the fact that
-Upper part of the uterus is enlarged by the growing fetus
-Lower part of the body is empty and extremely soft and
The cervix is comparatively firm. Because of the variation in consistency , in bimanual examination ( 2 fingers in the anterior formix and
the abdominal fingers behind the uterus), the abdominal and vaginal fingers seem to oppose below the body of the uterus

Correct Answer. b

(79). Abortion is the expulsion or extraction from the ureus, spontaneous or induced , of the fetus or embryo weighing

a. Less than 100 gms

b. Less than 50 gms

c. Less than 250 gms

d. Less than 500 gms

Solution. Ans-252: (d) Less than 500 gms


Ref:Read the text below
Sol :
· ‘’Abortion is the expulsion or extraction from its mother an embryo or fetus weighing 500 gms or less when it is not capable of
independent survival (WHO)’’.
· This 500 gms of fetal development is attained at approximately 22 weeks of gestation.
· This expelled embryo or fetus is called abortus.
· The term miscarriage which is mostly used is synonymous with spontaneous abortion.

Correct Answer. d

(80). Medical treatment of ectopic pregnancy should be offered to those patients whose hCG level is less than

a. 10,000 IU and the size of the mass is less than 10 cm

b. 5,000 IU and the size of the mass is less than 5 cm

c. 7,500 IU and the size of the mass is less than 7.5 cm

d. 2,000 IU and the size of the mass is less than 3 cm

Solution. Ans-253: (d) 2,000 IU and the size of the mass is less than 3cm
Ref:Read the text below
Sol :
· Methotrexate, a folic acid antagonisthas been shown to destroy proliferating trophoblast and has been shown to be effective in medical
management of ectopic pregnancy.
· Criteria for methotrexate treatment are
-Gestational sac not more than 3.5 cm
-Serum hCG level not more than 10,000 mIU/L
-Fetal cardiac activity absent
-Patient can be followed up.

Correct Answer. d

Copyright © 2014 Delhi Academy of Medical Sciences, All Rights Reserved. 30/118
(81). Most common causative organism for acute mastitis is

a. Streptococcus

b. Staphylococcus aureus

c. E. coli

d. Proteus

Solution. Ans-254: (b) Staphylococcus aureus


Ref:Read the text below
Sol :
· Acute bacterial mastitis is usually associated with lactation in majority of cases.
· Most are caused by staphylococcus aureus.

Correct Answer. b

(82). Amniocentesis for prenatal diagnosis of genetic diseases can be carried out as early as

a. 6-7 weeks

b. 8-9 weeks

c. 10-12 weeks

d. 14-16 weeks

Solution. Ans-255: (d) 14-16 weeks


Ref:Read the text below
Sol :

Methods used in prenatal testing


· Ultrasound – first trimester onwards
· Chorionic villous biopsy – from 11 weeks.
· Amniocentesis – from 14 weeks.
· Cordocentesis – from 19 weeks.

Correct Answer. d

(83). The following drugs are used for ovulation induction except

a. Clomephine citrate

b. Mifepristone

c. Gonadotrophins

d. Bromocriptine

Solution. Ans-256: (b) Mifepristone


Ref:Read the text below
Sol :
· The four major types of drugs therapy for ovulation problems include clomiphene citrate , injectable gonadotropins, GnRH pump, and
bromocriptine.
· Mifepristone (RU 486) is used as a postcoitalcontraceptive – causes shedding of deciduas and prevents implantation.

Correct Answer. b

Copyright © 2014 Delhi Academy of Medical Sciences, All Rights Reserved. 31/118
(84). Mini pill can be present to the following women except

a. Lactation women

b. Women over 35 years and smokers

c. Previous history ectopic pregnancy

d. Diabetic and hypertensive women

Solution. Ans-257: (c) Previous history ectopic pregnancy


Ref:Read the text below
Sol :
Minipill ( Progestin only pill)
ADVANTAGES
- Can be used in lactating women. No adverse effect on lactation- often called lactation pill.
- Can be prescribed in patients having hypertension, diabetes mellitus , fibroids, epilepsy, smoking and in patients
with h/o thromboembolism.
- Side effects of estrogen in the combined pill are totally eliminated; easy to take as there is no on and off
regime.
- Can be prescribed to women over 35 years and smokers; those with focal migraine.
- As regards to return of fertility , it is faster than in combined pill users, because ovulation is not suppressed in
all cases.
CONTRAINDICATIONS
- Previous ectopic pregnancy, breast and genital cancers, ovarian cysts, abnormal vaginal bleeding, active liver
and arterial disease, porphyria, liver tumor, valproate, spironolactone and meprobamate.

Correct Answer. c

(85). Physiologically, oxytocin is characterized by all except

a. Circulates entirely in free form

b. Plasma half life 3-5 minutes

c. Level increase during labor

d. Released in pulsatile fashion

Solution. Ans-258: (a) Circulates entirely in free form


Ref:Read the text below
Sol :
- Oxytocin is a nonapeptide which is synthesized in the supraoptic and paraventricular nuclei of the
hypothalamus.
- Half life = 3-4 mins; duration of action = approx. 20 minutes

Correct Answer. a

(86). How much is the risk of ovarian cancer increased above normal in women with nonautosomal dominant
genotype with one first degree related with ovarian cancer

a. 2-3 times

b. 5 times

c. 10 times

d. 20 times

Solution. Ans-259: (a) 2-3 times


Ref:Read the text below
Sol :
- Familial cases account for about 10% of all ovarian cancer. Compared to a lifetime risk of 1.6% in the general
population, women with one affected first degree relative, have a 5% risk. (5/1.6 = 3 times)
- In families with two of autosomal dominant familial cancers have been identified;
- Two types of autosomal dominant familial cancers have been identified;
- Breast / ovarian cancer syndrome; and
- The lynch type II cancer family (HNPCC- hereditary nonpolyposis colorectal cancer, and ovarian cancer

Correct Answer. a

Copyright © 2014 Delhi Academy of Medical Sciences, All Rights Reserved. 32/118
(87). What is the most common method for detecting early stage ovarian cancer?

a. Evaluation of vague gastrointestinal symptoms

b. Palpation of an asymptomatic mass during routine pelvic examination

c. Screening CA- 125

d. Screening vaginal ultrasound

Solution. Ans-260: (b) Palpation of an asymptomatic mass during routine pelvic examination
Ref:Read the text below
Sol :
‘’The diagnosis of early ovarian cancer usually occurs with the palpation of an asymptomatic adnexal mass during
routine pelvic examination’’.

Correct Answer. b

(88). The data are most convincing for which theory as the pathogenesis of endometriosis in the peritoneal
cavity

a. Coelomic metapasia

b. Induction

c. Embryonic rests

d. Lymphatic and vascular metastasis

Solution. Ans-261: (a) Coelomic metapasia


Ref:Read the text below
Sol :
- The theory of coelomic metaplasia proposes that endometriosis may develop from metaplasia of cells lining the
pelvic periotenum (Iwanoff and Meyer).

Correct Answer. a

(89). The formation of primordial follicle in human fetus is complete by

a. 4 weeks

b. 8 weeks

c. 13 weeks

d. 18 weeks

Solution. Ans-262: (d) 18 weeks


Ref:Read the text below
Sol :
- Ovarian differentiations starts at 6-8 weeks
- Meiotic division begins at 11-12 weeks
- Primordial follicle formation at 18-20 weeks

Correct Answer. d

Copyright © 2014 Delhi Academy of Medical Sciences, All Rights Reserved. 33/118
(90). Trimetazidine is an antianginal agent which acts by inhibition of:

a. Beta oxidation of fatty acids

b. Anaerobic metabolism of glucose

c. Calicum channels

d. Beta adrenergic receptors

Solution. Ans-186: (a) Beta oxidation of fatty acids


Ref: Read the text below
Sol:
- Normal myocardium utilises more of fatty acids than glucose for energy production. Oxidation of fatty acids
requires more of oxygen per ATP molecule generated.
-If beta oxidation of fatty acids is inhibited, the metabolism shifts to glucose utilisation which uses less oxygen per
molecule of ATP generated, hence it is useful to the ischemic myocardium.

Correct Answer. a

(91). High anion gap acidosis is seen in

a. Salicylate poisoning

b. Renal tubular acidosis

c. Ureterosigmoidostomy

d. Diarrhea

Solution. Ans-187: (a) Salicylate poisoning


Ref: Read the text below
Sol :
High anion gap metabolic acidosis is a form of metabolic acidosischaracterized by a high anion gap
Causes include:
"Mudpiles"
The mnemonic MUDPILES is commonly used to remember the causes of increased anion gap metabolic acidosis.
- M-Methanol
- U-Uremia (chronic renal failure)
- D-Diabetic ketoacidosis
- P-Propylene glycol ("P" used to stand for Paraldehyde but this substance is not commonly used today)
- I-Infection, Iron, Isoniazid, Inborn errors of metabolism
- L-Lactic acidosis
- E-Ethylene glycol (Note: Ethanol is sometimes included in this mnemonic as well, although the acidosis caused by
ethanol is actually primarily due to the increased production of lactic acid found in such intoxication.)
- S-Salicylates

Correct Answer. a

Copyright © 2014 Delhi Academy of Medical Sciences, All Rights Reserved. 34/118
(92). Tracings from chest leads of a child suffering from cyanotic congenital heart disease are shown below.
What is the most likely diagnosis:-

a. Fallot' tetralogy.

b. Total anomalous pulmonary venous return.

c. Tricuspid atresia.

d. Single ventricle.

Solution. Ans-188: (c) Tricuspid atresia


Ref: Read the text below
Sol:
- These leads show absence of right ventricular force (no R wave in right-sided chest leads, typical of tricuspid
atresia).
- TOF, TAPVR: right ventricular predominance is seen. Single ventricle: usually RVH or biventricular hypertrophy.
- Rarely it can cause left ventricular dominance on ECG.

Correct Answer. c

(93). Cluster headache is associated with all, except:

a. Unilateral headache

b. Onset typically in 20-50 years of life

c. Affects predominantly females

d. Commonly awakens the patient from sleep

Solution. Ans-190: (c) Affects predominantly females


Ref: Read the text below.
Sol:
Cluster headache:
-Affects predominantly males (7-8 times more than females)
-Also known as Raeder's syndrome, histamine cephalalgia, and sphenopalatine neuralgia.
- Lateralized
- Peak at 30-50 years
- Excruciating, deep, explosive pain which begins without warning, reaches it's peak within 5 minutes and lasts
for 45 minutes and commonly awakens the patients from sleep and is associated with homolateral lacrimation and
reddening of eye and ptosis and nasal stuffiness.

Correct Answer. c

Copyright © 2014 Delhi Academy of Medical Sciences, All Rights Reserved. 35/118
(94). A 56 year old coal mine worker presented with joint pains, cutaneous nodules and occasional cough with
dyspnoea. His chest x-ray showed multiple small nodules in both lungs with cavitation and calcification. Most likely
diagnosis is:

a. Silicosis

b. Caplan's syndrome

c. Sjogren's syndrome

d. Sarocidosis

Solution. Ans-191: (b) Caplan's syndrome


Ref: Read the text below.
Sol:
· Caplan's syndrome is a rare condition characterized by the presence of necrobiotic nodules in the periphery of
the lung in coal workers with rheumatoid arthritis (Coal worker's pneumoconiosis + Rheumatoid arthritis)

Correct Answer. b

(95). In a case of first idiopathic venous thromboembolism event; current recommendations for warfarin
therapy is for :

a. Less than 3 months

b. 3–6 months

c. 6 months

d. 12 months to lifetime

Solution. Ans-192: (c) 6 months


Ref: Read the text below.
Sol:
Current ACCP recommendations for duration of warfarin therapy
Recommendations for Long-Term Anticoagulation

Correct Answer. c

Copyright © 2014 Delhi Academy of Medical Sciences, All Rights Reserved. 36/118
(96). Waterhouse – Friderichsen syndrome is a complication of which one of the following ?

a. Miliary tuberculosis

b. Rickettsial disease

c. Leptospirosis

d. Fulminant meningococcemia

Solution. Ans-193: (d) Fulminant meningococcemia


Ref: Ghai - 496
Sol :
§Waterhouse – Friderichsen syndrome:
Seen in Fulminant meningococcemia
Pathology Adrenal insufficiency due to hemorrhage and necrosis in adrenal glands.
Fever, purpura, prostration, altered mental status.
Clinical presentation
Hypotension, shock
3rd generation cephalosporin – ceftriaxone, cefotaxime.
Treatment for meningococcal infection.
Alternative therapy – penicillin G

Correct Answer. d

Copyright © 2014 Delhi Academy of Medical Sciences, All Rights Reserved. 37/118
(97). Which of the following is the most specific test for rheumatoid arthritis

a. Anti-ccp antibody

b. Anti Igm antibody

c. Anti IgA antibody

d. Anti IgG antibody

Solution. Ans-194: (a) Anti-ccp antibody


Ref: Harrison 17th /e p 2088 & 16th/e p 1972]
Sol :
- Rheumatoid factor is an autoantibody, usually IgM directed against the Fc region of IgG.
- Despite its name its name, rheumatoid factor is not specific for rheumatoid arthritis, it can also be seen in wide
range of autoimmune disorders, inflammatory disease and chronic infections.
- Anticitrullinated peptide antibody test (anti ccp) test is more specific than rheumatoid factor for diagnosis of
rheumatoid arthritis. It may be positive very early in the course of the disease
Oxford journal of medicine states
“Anti-cyclic citrullinate peptide (anti-ccp) antidoy testing is particularly useful in the diagnosis of rheumatoid
arthritis with high specificity , present early in the disease process and has ability to identify patients who are likely
to have severe disease and irreversible damage. Anti ccp antibodies have not been found at a significant
frequency in other disease to date and are more specific than rheumatoid factor for detecting rheumatoid
arthritis.
Also know
Diagnosis of rheumatoid arthritis
- There is no single sign, symptom or test result that allows the definitive diagnosis of rheumatoid arthritis.
- Instead, the diagnosis is based on a consideration of many factors, including the presence of characteristic signs
and symptoms of rheumatoid arthritis, the results of laboratory tests, and the results of x-ray.
- The diagnosis of R.A. is based on the presence of at least four of the following criteria:
- Mornign stiffness that lasts at least one hour and that has been present for at least six weeks
- Swelling of at least three or more joints for 6 weeks.
- Swelling of the wrist, metacarpophalangeal (MCP), or proximal interphalangeal (I.T.P.) joints for at least six weeks.
- Swelling of the same joint on both sides of the body
- Changes in hand x-rays that are characteristic of rheumatoid arthritis
- Rheumatoid nodules of the skin
- Rheumatoid factor found in the blood.
Antibodies associated with Connective tissue disease-
1. Anti double stranded DNA (Anti dsDNA) SLE
2. Anti Histone antibodies Drug induced lupus erythematosis
3. Anti Sm antibody (antibody to core lprotein of small nuclear
SLE
ribonucleoprotein
4. SSA Sjogren syndrome
(Ro) - Antibodies to ribonucleo proteins antigens
5. SSB
(Ro)
6. Antibodies of ribonucleoprotein antigen containing ULRNP Mixed connective tissue disease
7. Anti DNA topoisomerase antibody (Scl70) Systemic sclerosis (diffuse)
8. Anti centromeric antibody Limited scleroderma (crest)
9. Histidyl-tRNA synthetase Inflammatory myopathy
10. Anti mitochondrial antibody Primary biliary cirrhosis

Correct Answer. a

Copyright © 2014 Delhi Academy of Medical Sciences, All Rights Reserved. 38/118
(98). Which of the following exclusively involve neurouns

a. Spinocerebellar ataxia

b. Supranuclear palsy

c. Corticobasilar degeneration

d. Multiple system atrophy

Solution. Ans-195: (c) Corticobasilar degeneration


Ref: Harrison ed.18th .Chapter 371.Pg No.3311,3312.Ch.373.pg no.3341.
Sol :
From various sources it was clear that Spinocerebellar ataxia involves the spinocerebellar tracts ,posterior
column with neurons pons & cerebellum are involved.
Supranuclear palsy involves neurons with glial cless also.
Multiple system atrophy involves autonomic fibres.
Corticobasilar degeneration neuronal loss severly in cortex & basal gangalia with not much mention about
involvement of any other cell lineage.

Correct Answer. c

(99). Osteodystrophy secondary to azotemia occurs due to :

a. Increased erythropoietin levels

b. Secondary hyperparathyroidism

c. Hypoproteinaemia

d. Hyperkalemia

Solution. Ans-196: (b) Secondary hyperparathyroidism


Ref: Fortar - 644
Sol :
Renal osteodystrophy :
- The impairment of renal production of 1, 25 dihydroxycholecalciferol is the major factor in the development of
renal osteodystrophy. Phosphate retention, secondary hyperparathyroidism and skeletal resistance to parathyroid
hormone, intestinal malabosteodystrophy.
- Phosphate retention, secondary hyperparathyroidism and skeletal resistance to parathyroid hormone, intestinal
malabsorption of calcium leading to hypocalcemia and altered vitamin D metabolism all contribute the
disturbances in bone and mineral metabolism associated with renal osteodystrophy.

Correct Answer. b

(100). Features of mismatched blood transfusion all of the following except

a. Oliguria

b. Hypercalcemia

c. Hyperkalemia

d. Hypothermia

Solution. Ans-197: (b) Hypercalcemia


Ref: Read the text below
Sol :
- Hemolytic blood transfusion reaction
- In awake patients – chills, fever, nausea, chest pain, urticaria
- In anaesthetized patient – rise in temperature, unexplained tachycardia, hypotension, hemoglobinuria and
oozing in surgical field
- Intravascular hemolysis, hemoglobinuria and ARF with oliguria
- ARDS, non cardiogenic pulmonary edema
- Hypothermia, hyperkalemia, hypocalcemia, iron overload, metabolic alkalosis

Correct Answer. b

Copyright © 2014 Delhi Academy of Medical Sciences, All Rights Reserved. 39/118
(101). The important structure seen in the triangle of Koch is:

a. Sinoatrial node

b. Suprascapular artery

c. Femoral artery

d. Atrioventricular node

Solution. Ans-198: (d) Atrioventricular node


Ref: Read the text below
Sol:
- Triangle of Koch is situated on the septal wall of the right atrium, bounded by the tricuspid valve, coronary sinus
orifice and tendon of Todaro.
- Atrioventricular node is situated at the apex of the triangle.

Correct Answer. d

(102). The most common histological type seen in bronchogenic carcinoma is :

a. Small cell carcinoma

b. Large cell carcinoma

c. Squamous cell carcinoma

d. Adenocarcinoma

Solution. Ans-199: (d) Adenocarcinoma


Ref.: Harrison’s - 552
Sol :
- Adenocarcinoma
-Most common histological type
-Common histological type in young patients and in females metastases to opposite lung
-Peripheral distribution.
- Squamous cell carcinoma
-Most common histological type in India
-Cavitate, central distribution (pancoast tumour is peripheral tumour of squamous variety)
-Hypercalcemia due to PTH
-Best prognosis.

Correct Answer. d

(103). Interstitial pulmonary fibrosis is associated with the following except :

a. Sarcoidosis

b. Asbestosis

c. Carcinoid lung

d. Radiation exposure

Solution. Ans-210: (c) Carcinoid lung


Ref.: Read the text below
Sol :
Diseases and conditions that may cause pulmonary fibrosis as a secondary effect include:
-Inhalation of environmental and occupational pollutants, such as in asbestosis, silicosis and exposure to certain
gases. Coal miners, ship workers and sand blasters among others are at higher risk.
-Hypersensitivity pneumonitis, most often resulting from inhaling dust contaminated with bacterial, fungal, or
animal products.
-Cigarette smoking can increase the risk or make the illness worse.
-Some typical connective tissue diseases such as rheumatoid arthritis, SLE and scleroderma
-Other diseases that involve connective tissue, such as sarcoidosis and Wegener's granulomatosis.
-Infections
-Certain medications, e.g. amiodarone, bleomycin (pingyangmycin), busulfan, methotrexate, apomorphine,
andnitrofurantoin
-Radiation therapy to the ches

Correct Answer. c

Copyright © 2014 Delhi Academy of Medical Sciences, All Rights Reserved. 40/118
(104). Which congenital heart disease is associated with pre-excitation?

a. Atrial septal defect

b. Bicuspid aortic valve

c. Ebstein’s anomaly

d. patent ductus arteriosus

Solution. Ans-211: (c) Ebstein’s anomaly


Ref.: Tripathi KD- 484
Sol :
- About 50% of individuals with Ebstein anomaly have evidence of Wolff-Parkinson-White syndrome,
secondary to the atrialized right ventricular tissue.
-10-lead ECG of a woman withEbstein anomaly. The ECG shows signs of right atrial enlargement, best seen in V1.
Other P waves are broad and tall, these are termed "Himalayan" P waves. There is also a right bundle branch block
pattern and a first degree atrioventricular block (prolonged PR-interval) due to intra-atrial conduction delay. There
is no evidence of a Kent-bundle in this patient. There is T wave inversion in V1-4 and a marked Q wave in III; these
changes are characteristic for Ebstein's anomaly and do not reflect ischemic ECG changes in this patient.
- Other abnormalities that can be seen on the ECG include (1) signs of right atrial enlargement or tall and broad
'Himalayan' P waves, (2) first degree atrioventricular block manifesting as a prolonged PR-interval, (3) low
amplitude QRS complexes in the right precordial leads, (4) atypical right bundle branch block, (5) T wave
inversion in V1-V4 and Q waves in V1-V4 and II, III and aVF
-Ebstein's cardiophysiology typically presents as an (antidromic) AV reentrant tachycardia with associated preexcitation.
In this setting, the preferred pharmacological treatment agent is procainamide. Since AV-blockade may
promote conduction over the accessory pathway, drugs like beta blockers, calcium channel
blockers and digoxin are contraindicated.

Correct Answer. c

(105). Highest risk of CHD is seen in the following type of hyperlipidemia

a. Familial hypercholesterolemia

b. Lipoprotein lipase deficiency

c. Familial hypertriglyceridemia

d. Dysbetalipoproteinemia

Solution. Ans-189: (a) Familial hypercholesterolemia


Ref: Read the text below
Sol :
- Genetic nomenclature :
- I – Familial chylomicronemia
- IIa – Familial hypercholesterolemia, sitosterolemia
- IIb – Familial combined hyperlipidemia
- III – Familial dysbetalipoproteinemia
- IV & V – Familial hypertriglyceridemia

Correct Answer. a

Copyright © 2014 Delhi Academy of Medical Sciences, All Rights Reserved. 41/118
(106). The following drugs have been found to be useful in the treatment of primary pulmonary hypertension except :

a. Calcium channel blockers

b. Prostacyclins

c. Beta blockers

d. Phosphodiesterase V inhibitors (sildenafil)

Solution. Ans- : (c) Beta blockers


Ref.:Harrison’s - 1578
Sol :
Treatment of primary pulmonary hypertension :
-Some authorities advocate long – term oral anticoagulation and this is given in most patients with primary pulmonary hypertension.
-Oxygen, particularly at night, appears to improve symptoms and helps in reducing pulmonary pressure.
-Calcium channel blockers at time in large dose have been used with mixed results.
-However, calcium channel blocker can worsen RV function other negative ionotropic effects.
-Epoprostenol (prostacyclin) is administered by continuous infusion and has been shown to improve functional capacity and survival.
-Endothelin antagonists such as bosentan are also available orally.
-Phosphodiesterase inhibitor such as sildenafil is also being carefully investigated.

Correct Answer. c

(107). Which of the following tachycardias are not amenable to radiofrequency catheter ablation:

a. Torsades de pointes

b. Belhassen’s tachycardia

c. Atrioventricular nodal reentrant tachycardia

d. Atrioventricular reentrant tachycardia

Solution. Ans- : (a) Torsades de pointes


Ref:Read the text below
Sol:
· Torsades de pointes is a polymorphic ventricular tachycardia, with no specific focus of origin which can be ablated.
· Since it is associated with QT prolongation, measures to reduce the QT interval will suppress the arrhythmia.
Magnesium infusion, correction of hypokalemia, isoprenaline infusion, overdrive pacing and treatment of ischemia are the useful
measures

Correct Answer. a

(108). Primary angioplasty is preferred if the delay over and above that of thrombolysis is less than:

a. 30 minutes

b. 60 minutes

c. 90 minutes

d. 120 minutes

Solution. Ans-: (b) 60 minutes


Ref:Read the text below
Sol:
· If the anticipated delay for getting the balloon dilated in the culprit vessel (door to balloon time) is more than 60 minutes higher than
the time for initiation of thrombolytic therapy in myocardial infarction (door to needle time), thrombolytic therapy has to preferred.

Correct Answer. b

Copyright © 2014 Delhi Academy of Medical Sciences, All Rights Reserved. 42/118
(109). A 27-year-old man develops a painless 1 cm sore on his penis. It appears ulcerated with a raised margin and
minimal serous exudates. Which of the following is the most appropriate next step in the diagnosis? (See fig.)

a. Biopsy

b. Gram stain

c. Serology

d. Ultrasound

Solution. Ans 200: (c) Serology


Ref:Read the text below
Sol:
-This is likely a case of primary syphilis. Serology is the most appropriate test; it can remain negative for a period
up to 1 month after the infection is contracted.
 The serologic test for syphilis usually is positive within 1 week after the chancre appears. With therapy the
chancre heals rapidly, but will heal in 4–6 weeks even without treatment.
- Genital chancres are usually painless, unless superinfected, but extragenital chancres (e.g., fingers) can be quite
painful. Biopsy is usually not necessary for diagnosis, and the spirochetes are seen with dark-field examination,
not with Gram stain.

Correct Answer. c

(110). The important mechanism of action of ranolazine is:

a. Inhibits late sodium current

b. Inhibits beta receptors

c. Inhibits L – type calcium channels

d. Inhibits transient outward potassium current

Solution. Ans-201: (a) Inhibits late sodium current


Ref: Read the text below
Sol:
- Ranolazine also inhibits beta oxidation of fatty acids partially.
-It acts as an antianginal agent by both these mechanism.

Correct Answer. a

Copyright © 2014 Delhi Academy of Medical Sciences, All Rights Reserved. 43/118
(111). A decrease in the Anion Gap (AG) can be due to

a. ↑ In unmeasured cations

b. ↓ In the major plasma anion albumin concentration

c. ↓ In the effective anionic charge on albumin by acidosis

d. Hypoviscosity

Solution. Ans-202: (d) Hypoviscosity


Ref: Read the text below
Sol :
Anion Gap :
- Represents those unmeasured anions in plasma (normally 10 to 12 m mol/L)
- AG = Na+- (C1- + HCO3
-)
-The unmeasured anions include anionic proteins (albumin), phosphate, sulfate, and organic anions
A decrease in the AG can be due to
-In unmeasured cations (calcium, magnesium, potassium)
-Addition abnormal cations, such as lithium (lithium intoxication) or cationic immunoglobulins (plasma cell
dyscrasias) to the blood
--In the major plasma anion albumin concentration (nephritic syndrome)
- In the effective anionic charge on albumin by acidosis
- Hyperviscosity and severe hyperlipidemia, which can lead to an underestimation of sodium and chloride
concentrations .
- A fall in serum albumin by 1 g/dl from the normal value (4.5 g/dL) decreases the AG by 2.5 mEq/L

Correct Answer. d

(112). Euvolemic hyponatremia is seen in

a. Adrenocortical failure

b. Burns

c. SIADH

d. Cirrhosis

Solution. Ans-203: (c) SIADH


Ref: Read the text below
Sol :
Hypervolemic hyponatremia — Both sodium & water content increase: Increase in sodium content leads to
hypervolemia and water content to hyponatremia. Total body water and sodium are regulated independently[10]
- cirrhosis of the liver
- congestive heart failure
- nephrotic syndrome in the kidneys
- massive edema of any cause
Euvolemic hyponatremia — there is no volume expansion in the body, no edema, but hyponatremia occurs
- states of severe pain or nausea
- in the setting of trauma or other damage to the brain
- SIADH (and its many causes)
- Hypothyroidism
- Glucocorticoid (steroid) deficiency
Hypovolemic hyponatremia — The hypovolemia (extracellular volume loss) is due to total body sodium loss. The
hyponatremia is caused by a relatively smaller loss in total body water.
- any cause of hypovolemia such as prolonged vomiting, decreased oral intake, severe diarrhea
- diuretic use (due to the diuretic causing a volume depleted state and thence ADH release, and not a direct result
of diuretic-induced urine sodium loss)
- Addison's disease and congenital adrenal hyperplasia in which the adrenal glands do not produce sufficient
steroid hormones (combined glucocorticoid and mineralocorticoid deficiency)

Correct Answer. c

Copyright © 2014 Delhi Academy of Medical Sciences, All Rights Reserved. 44/118
(113). Which one of the following is not likely to be a feature in a patient with secondary hypothyroidism ?

a. Low fee T4

b. Low TSH

c. Low ACTH

d. Low prolactin

Solution. Ans-204: (d) Low prolactin


Ref.: Harrison’s - 2229
Sol :
- Secondary hypothyroidism means hypothyroidism due to cause other than thyroid tissue like pituitary failure
hypothalamic failure, etc.

Correct Answer. d

(114). Epstein – Barr virus has been associated with the following disorders except :

a. Burkitt’s lymphoma

b. Nasopharyngeal carcinoma

c. Hairy cell leukemia

d. Oral hairy leukoplakia

Solution. Ans-205: (c) Hairy cell leukemia


Ref.: Read the text below
Sol :
-Epstein-Barr virus (EBV), a ubiquitous B-lymphotrophic herpesvirus, has been found in the tumor cells of a
heterogeneous group of malignancies (Burkitt's lymphoma, lymphomas associated with immunosuppression, other
non-Hodgkin's lymphomas, Hodgkin's disease, nasopharyngeal carcinoma, gastric adenocarcinoma,
lymphoepithelioma-like carcinomas, and immunodeficiency-related leiomyosarcoma).
-Several studies utilizing immunofluorescence staining have clearly documented a very strong association of
Epstein-Barr virus (EBV) with Oral hairy leukoplakia

Correct Answer. c

(115). The level of which one of the following compounds is elevated in bronchial asthma?

a. PGI

b. PGH2

c. Leukotriens

d. Thromboxane

Solution. Ans-209: (c) Leukotriens


Ref.:Harrison’s -1596
Sol :
Pathogenesis of asthma :
-Extrinsic (IgE mediated, triggered by allergens)
-Intrinsic (non – IgE mediated, triggered by infection).
-Mixed, exercise induced or aspirin induced.
-Inhalation of allergen leads to biphasic response
Early reaction Late phase
-Occurs in about two-thirds of patients.
-Within 10 minutes of exposure to allergens.
-It develops 3-4 hours later with peak at -12 hrs.
-Release of histamine, leukotriens C4, D4 and E4, prostaglandins,
-Release of mast cell mediators.
platelet activating factor and bradykinin from mast cells
-Not prevented by premedication with b2 agonist drugs
-This phase is inhibited by b2agonist drugs and mast cell stabilizing
-Inhibited by premedication with steroids and cromolyn or
agents (cromolyn or nedocromil).-
nedocromil.

Correct Answer. c

Copyright © 2014 Delhi Academy of Medical Sciences, All Rights Reserved. 45/118
(116). The curve depicted below plots the normal relationship of arterial PO2 and percentage of hemoglobin
saturation with other variables controlled at pH 7.4, PaCO2 40 kPa, temperature 37C (98.6F), and hemoglobin 15
g/dL. Which of the following statements regarding this oxygen dissociation relationship is true?

a. Modest decrements of arterial PO2 have a major effect on alveolar oxygen uptake

b. Modest decrements of hemoglobin saturation have a major effect on tissue oxygen uptake

c. The curve shifts to the left with acidosis

d. The curve shifts to the left following banked blood transfusion

Solution. Ans-212: (d) The curve shifts to the left following banked blood transfusion
Ref: Read the text below
Sol:
- The shape of the oxygen dissociation curve translates into several physiologic advantages.
- The relatively flat slope above a PO2 of 50 pKa means that, in this region of the curve, hemoglobin saturation
decreases slightly with decrements in PO2; loading of oxygen at the alveolar level is therefore affected minimally
with mild to moderate degrees of hypoxemia.
- The steeper slope at the lower end of the curve means that, as the hemoglobin becomes desaturated, arterial
PO2 drops only minimally, and a gradient that favors oxygen diffusion into tissue cells is maintained.
- Acidosis, a rise in PaCO2, and elevation of temperature all shift the curve to the right, which enhances tissue
oxygen uptake. Red blood cell organic phosphates, particularly 2,3-diphosphoglycerate (2,3-DPG), also affect the
dissociation curve. Banked blood, being low in 2,3-DPG, shifts the curve to the left and therefore decreases tissue
oxygen uptake. 2,3-DPG levels increase with chronic hypoxia.
- Chronic lung disease, therefore, results in a shift of the curve to the right, which enhances oxygen delivery to
peripheral tissues.

Correct Answer. d

Copyright © 2014 Delhi Academy of Medical Sciences, All Rights Reserved. 46/118
(117). An immunoglobulin (see the figure below) is hydrolyzed by papain to form two A fragments and one B fragment. It is true of fragment A
that it

a. Contains the constant regions

b. Is the heavy chain

c. Contains the light chain

d. Is not functional as an antibody-combining site

Solution. Ans-: (c) Contains the light chain


Ref:Read the text below
Sol :
· The light chain and part of the heavy chain at the amino terminal contain the antibody-combining site in the “hypervariable regions.”
· These regions are all contained in fragment A, which is known as Fab. Fragment B is known as Fc.
· The Fc fragment contains a site for binding of complement. The Fab fragments mediate complement fixation.
· Each of the fragments can be further dissociated into two subunits by breaking its disulfide bridge with mercaptoethanol or some
other reducing agent.

Correct Answer. c

(118). Milk ring test is used for :

a. Screening for brucellosis

b. To detect fat content of milk

c. To know the effectiveness of pasteurization

d. To detect sulphates in milk

Solution. Ans-: (a) Screening for brucellosis


Ref.:Read the text below
Sol :
Milk ring test is used in brucellosis.
It is used for the detection of Brucella antibodies in the milk of the infected animal.
Tests for detection of brucellosis in cattle herds :
§Rapid plate agglutination test
§Rose Bengal card test
Tests for antibody detection in milk :
§Milk ring test
§Whey agglutination test

Correct Answer. a

Copyright © 2014 Delhi Academy of Medical Sciences, All Rights Reserved. 47/118
(119). Urinary antigen detection is used as diagnostic test in:

a. Staphylococcus aureus

b. Brucellosis

c. Typhoid fever

d. Legionella Pneumophila

Solution. Ans:: (d) Legionella Pneumophila


Ref.:Read the text below
Sol :
§Urinary antigen detection by Latex agglutination or ELISA is used in diagnosis of legionella infection.
§Antigen in urine is detectable 3 days after the onset of clinical disease and disappears over two months.
§The test is not affected by antibiotic administration.
Urine antigen detection is used for :
§Pneumococcus
§Legionella pneumophila serogroup 1.

Correct Answer. d

(120). Which of the following organisms grows contiguously in tissues with no respect to anatomic barriers?

a. Actinomyces israelii

b. Mycobacterium leprae

c. Mycobacterium kansasii

d. Mycobacterium tuberculosis

Solution. Ans-: (a) Actinomyces israelii


Ref:Read the text below
Sol :
· Actinomyces israelii grows contiguouslyin tissues, crossing anatomic barriers; thus, it often invades bone.
· Mycobacterium tuberculosis may be hematogenously spread to any tissue.

Correct Answer. a

(121). The Ghon complex, a lesion in the lung and regional lymph nodes, is found in

a. Actinomycotic mycetoma.

b. Tuberculoid leprosy.

c. Primary tuberculosis.

d. Secondary tuberculosis.

Solution. Ans-: (c) Primary tuberculosis.


Ref: Read the text below
Sol :
• The Ghon complex is the combination of the initial Ghon lesion at the site of Mycobacterium tuberculosis infection in the lung and
involvement of the adjoining lymph node. This lesion heals after primary tuberculosis.

Correct Answer. c

Copyright © 2014 Delhi Academy of Medical Sciences, All Rights Reserved. 48/118
(122). Sporocidal disinfectant is following except:

a. Glutaraldehyde

b. Formaldehyde

c. Ethylene oxide

d. Benzalkonium chloride

Solution. Ans-: (d) Benzalkonium chloride


Ref.:Read the text below
Sol :
Sporocidal agents are :
E – Ethylene oxide
F – Formaldehyde
G – Glutaraldehyde
H – Halogens (Moderate action on spores)

Correct Answer. d

(123). Sterilization of culture media containing serum is by:

a. Autoclaving

b. Micropore filter

c. Gamma radiation

d. Centrifugation

Solution. Ans-: (a) Autoclaving


Ref.:Read the text below
Sol :
Most of culture media are sterilized by Autoclaving at 121° C for 15 min at 15 Ib pressure/inch2.
Steam under pressure (Autoclaving or steam sterilizer)
Used for sterilizing dressings, instruments, laboratory ware, media, pharmaceutical products and aqueous solution.

Correct Answer. a

(124). pH of Sabourauds dextrose agar is adjusted to :

a. 4-6

b. 1-2

c. 6-8

d. 8-10

Solution. Ans-: (a) 4-6


Ref.:Read the text below
Sol :
· Sabouraud agar is a type of agar containing peptones.
· It is used to cultivatedermatophytes and other types of fungi, and can also grow filamentous bacteria such as Nocardia.
· It was created by, and is named after, Raymond Sabouraud in 1892.
· Later adjusted by Chester W. Emmons when the pH level was brought closer to the neutral range and the dextrose concentration
lowered to support the growth of other fungi. The 5.6 pH of traditional Sabouraud agar formulation inhibits bacterial growth.

Correct Answer. a

Copyright © 2014 Delhi Academy of Medical Sciences, All Rights Reserved. 49/118
(125). Which of the following organisms is implicated in the causation of botryomycosis.

a. Staphylococcus aureus

b. Staphylococcus albus

c. Pseudomonas aeruginosa

d. Streptococcus pneumonia

e. Streptococcus pyogenes

Solution. Ans-: (a) Staphylococcus aureus


Ref.:Read the text below
Sol :
§Botyromycosis is a chronic granulomatouscondititon similar to mycetoma, usually involves the skin and characterized by granules in the
pus, consisting of masses of bacteria generally staphylococcus aureus.

Correct Answer. a

(126). Conjugate vaccine are available for the prevention of invasive disease caused by all of the following bacteria except:

a. H. influenzae

b. Strep pneumonia

c. Neisseria meningitides (group-C)

d. Neisseria meningitides (Group-B)

Solution. Ans-: (d) Neisseria meningitides (Group-B)


Ref.:Read the text below
Sol :
“No vaccine is availablefor protection against group B meningococci.”
Conjugate vaccines are prepared by conjugating protein carrier with polysaccharide capsule.
Conjugate vaccine are available for :
Hemophilus influenza B
Menigococcal serotypes A, C, Y and W- 135.
Streptococcal pneumonia (Pneumococcus).

Correct Answer. d

(127). Best way to sterilize all-glass syringes is :

a. Boiling

b. Autoclave

c. Hot air oven

d. Formaldehyde

Solution. Ans-: (c) Hot air oven


Ref.:Ananthanarayan’s Microbiology, 8th ed, p-31
Sol :
§Dry heat can be used to sterilize items, but as the heat takes much longer to be transferred to the organism.
§The standard setting for a hot air oven is at least two hours at 160°C (320 °F). It is used to sterilize glassware, forceps, scissors,
scalpels, all-glass syringes, swabs, liquid paraffin etc.
§A rapid method heats air to 190 °C (374 °F) for 6 minutes for unwrapped objects and 12 minutes for wrapped objects.
§Dry heat has the advantage that it can be used on powders and other heat-stable items that are adversely affected by steam.

Correct Answer. c

Copyright © 2014 Delhi Academy of Medical Sciences, All Rights Reserved. 50/118
(128). Which of the following immunoglobulins (Ig) is at its highest level in a normal adult?

a. IgA

b. IgG

c. IgM

d. IgD

Solution. Ans-: (b) IgG


Ref:Read the text below
Sol :
· Immunoglobulin G (IgG) is the predominant immunoglobulin (approximately 73% of total immunoglobulin) found in the adult.

Correct Answer. b

(129). The most common neoplastic manifestation of HIV disease is

a. Adenocarcinoma

b. Kaposi’s sarcoma

c. Leukemia

d. Lymphoma

Solution. Ans-: (b) Kaposi’s sarcoma


Ref: Read the text below
Sol :
Lymphoma, especially of thebrain, is fairly common in advanced HIV disease.
Kaposi’s sarcoma is quite frequent.
In the past, KS was diagnosed at presentation in 40 percent of all HIV + patients; today it is diagnosed in about 15 percent.

Correct Answer. b

(130). Cryptococcus is transmitted to humans via?

a. Aerosolized particles from soil

b. Animal bites

c. Contaminated water

d. Human-to-human contact

Solution. Ans: (a) Aerosolized particles from soil


Ref: Read the text below
Sol :
Cryptococcus is endemic in pigeons andother birds and excreted in their droppings.
The fungus remains in the soil and can be inhaled in aerosolized particles when the soil is disturbed.
There is no evidence of human-to-human or animal (insect)-to-human transmission.

Correct Answer. a

Copyright © 2014 Delhi Academy of Medical Sciences, All Rights Reserved. 51/118
(131). Funduscopic changes typical of CMV retinitis include

a. Copper spots

b. Hyperpigmentation

c. Neovasularization

d. White, cheesy exudates

Solution. Ans-: (d) White, cheesy exudates


Ref: Read the text below
Sol :
§White to yellow cheese-like exudates are they typical characteristics of CMV retinitis.

Correct Answer. d

(132). The virulence factor of Nelsseria gonorrhoeae includes all of the following except :

a. Outer membrane proteins

b. IgA protease

c. M protein

d. Pilli

Solution. Ans-: (c) M protein


Ref.:Read the text below
Sol :
Virulence factors of Neisseria gonorrhea
1.Outer Membrane Proteins
Opacity associated protein (Protein II)
§Pilli
Lipoprotein H. 8
§Porin (Protein I and III)
Transferrin and lactoferrin binding protein
§IgA1 protease
2. Lipooligosaccharide (endotoxin)

Correct Answer. c

(133). SARS virus contains :

a. dsDNA

b. ssDNA

c. dsRNA

d. ssRNA

Solution. Ans-: (d) ssRNA


Ref.:Read the text below
Sol :
SARS is caused by corona virus type 4; a ssRNA virus
Severe active respiratory syndrome :
Caused by corona virus type 4
Spreads by inhalation
Shows features of Pneumonitis 25% of patients have diarrhea.
Risk factors of severe disease – age > 50 years, co morbidities such as cardiovascular disease, diabetes and hepatitis
Death due to respiratory failure

Correct Answer. d

Copyright © 2014 Delhi Academy of Medical Sciences, All Rights Reserved. 52/118
(134). Louis Pasteur is associated with all of the following except

a. Discovery of rabies vaccine

b. Germ theory of disease

c. Staining techniques

d. Techniques of sterilization

Solution. Ans-: (c) Staining techniques


Ref.:Read the text below
Sol :
§Staining techniques were discovered by Robert koch.
§Historical aspects of microbiology
Scientists Discovery
Father of Modern Microbiology
§Disproved theory of spontaneous generation
§Proposed Germ theory of disease
§Microbial theory of fermentation
Louis Pasteur §Techniques of sterilization
§Steam sterilizer, hot-air over, autoclave
§Coined the term “vaccine”
§Vaccines for chicken cholera, anthrax, rabies
§Introduced complex media

Correct Answer. c

(135). The function of an adjuvant in a vaccine is to enhance its:

a. Distribution

b. Immunogenicity

c. Absorption

d. Shelf life

Solution. Ans-: (b) Immunogenicity


Ref.:Read the text below
Sol :
An adjuvant is any substance that enhances the immunogenicity of an antigen.
Adjutants :
Any substance that enhances immunogenicity of an antigen
May confer immunogenicity on non-antigenic substances
Increase concentration and persistence of circulating antibody
Enhance degree of cellular immunity
May produce “adjuvant diseases” such as allergic disseminated encephalomyelitis.

Correct Answer. b

(136). The secondary vitreous develops from:

a. Neuroectoderm

b. Mesoderm

c. Neural crest

d. Surface ectoderm

Solution. Ans-173: (a) neuroectoderm


Ref:Read the text below
Sol:
• The vitreous humour is the clear gel that fills the space between the lens and the retina of the eyeball of
humans. It is often referred to as the vitreous body or simply "the vitreous".
▪ The primary vitreous regresses in adult eye; and develops from mesoderm .

Correct Answer. a

Copyright © 2014 Delhi Academy of Medical Sciences, All Rights Reserved. 53/118
(137). Maximum refraction takes place between :

a. Air tear film

b. Tear film and cornea

c. Cornea and aqueous

d. Aqueous lens

Solution. Ans : A
Refractive index of tear film and cornea are same , so it's same as air cornea interface.

Correct Answer. a

(138). Objective method to assess visual acuity in malingering patients

a. Photostress test

b. Snellen’s chart

c. Inducing optokinetic nystagmus (OKN)

d. Perimeter

Solution. Ans-175: (c) inducing OKN


Ref.: Read the text below
Sol : The patient is made to sit in front of a rotating drum, if his visual pathway is normal we will see the
optokinetic nystagmus ie saccadic and pursuit movements . This is OK -Drum test .

Correct Answer. c

(139). In Laser assisted epithelial keratomilieusis , the minimal stromal bed recommended is :

a. 220 microns

b. 240 microns

c. 270microns,

d. 250 microns

Solution. Ans-176: (c)


Minimum 270 micron of stromal bed is essential to prevent complications like keratoconus and
descematocoele.

Correct Answer. c

(140). Eye involvement is seen in :

a. Seropositive poly articular JRA late onset

b. Seronegative Pauciarticular JRA late onset

c. Seronegative poly articular JRA early onset

d. Seronegative pauciarticular JRA early onset

Solution. Ans : D
Uveitis is a feature of pauciarticular seronegative early onset JRA. Early onset cases are positive for ANA
antibodies.

Correct Answer. d

Copyright © 2014 Delhi Academy of Medical Sciences, All Rights Reserved. 54/118
(141). Equatorial diameter of the crystalline lens is:

a. 7mm

b. 8mm

c. 9mm

d. 10mm

Solution. Ans-178: (c) 9mm


Ref:Read the text below
Sol:
▪ The lens is a transparent, biconvex structure in the eye that, along with the cornea, helps torefract light to
be focused on the retina.
▪ The lens, by changing shape, functions to change thefocal distance of the eye so that it can focus on objects at
various distances, thus allowing a sharp real image of the object of interest to be formed on the retina.
▪ The diameter of the crystalline lens is 9 mm. average power of adult lens is +17D
▪ It has two surfaces, i.e. anterior and posterior.
▪ The anterior surface is less convex than the posterior surface.

Correct Answer. c

(142). The most sensitive part of the Retina is called:

a. Foveola

b. Fovea centrails

c. Choroidal plexus

d. Optic disc

Solution. Ans-179 : (a) foveola


Ref:Read the text below
Sol:
▪ Foveola , a depression or pit has only cones in the neuroepithelial layer.
▪ Rods are absent in this region.

Correct Answer. a

(143). Which nerve carries the parasympathetic fibers?

a. Nerve to the medial rectus

b. Nerve to the superior rectus

c. Nerve to the inferior oblique

d. Nerve to the superior oblique

Solution. Ans-180 : (c) Nerve to the inferior oblique


Ref:Read the text below
Sol:
▪ The parasympathetic fibres start from the Edinger-Westphal nucleus and run in the main trunk of the third
nerve, as far in the orbit.
▪ Then the fibres passes into the branch which supplies the inferior oblique muscle; leaving it by the short root
of the ciliary ganglion.

Correct Answer. c

Copyright © 2014 Delhi Academy of Medical Sciences, All Rights Reserved. 55/118
(144). Leber’s optic neuropathy has the following features except:

a. It commonly affects healthy males

b. It causes uniocular, sudden painless, progressive and permanent visual loss

c. Pupillary reflexes are affected early in the disease

d. Typical visual field defects are centrocaecal which eventually become absolute

Solution. Ans-181: (c) Pupillary reflexes are affected early in the disease
Ref.: Read the text below
Sol :
▪ In Leber’s optic neuropathy the pupillary light reactions frequently remain fairly brisk despite severe visual loss.
▪ It commonly affects healthy males
▪ It causes uniocular, sudden painless, progressive and permanent visual loss
▪ Typical visual field defects are centrocaecal which eventually become absolute

Correct Answer. c

(145). Papilloedema has all the following characteristic except :

a. Marked loss of vision

b. Blurring of disc margins

c. Hyperemia of disc

d. Field defect

Solution. Ans-182: (a) Marked loss of vision


Ref.: Read the text below
Sol :
▪ Visual acuity in papilloedema is usually normal unless the fovea is involved with haemorrhage, exudates or
edema.

Correct Answer. a

(146). The rate of facility of aqueous outflow is:

a. 0.22microlitres per minute/ mmHg

b. 0. 23 microlitres per minute/mmHg

c. 0.24microlitres per minute/mmHg

d. 0.21microlitres per minute/mmHg

Solution. Ans : C
Facility of aqueous outflow is depicted as C value and is measured by tomography. Normal C value is 0.24-
0.28microlitres/min/mmHg

Correct Answer. c

Copyright © 2014 Delhi Academy of Medical Sciences, All Rights Reserved. 56/118
(147). “Pie in the floor” field defects are seen in:

a. Optic tract lesions

b. Lateral geniculate lesions

c. Temporal lobe lesions

d. Parietal lobe lesions

Solution. Ans-184 : (d) Parietal lobe lesions


Ref.: Read the text below
Sol :
▪ If the superior fibers of the optic radiations are involved, an inferior quadrantic hemianopia is produced called
“Pie in the Floor”.

Correct Answer. d

(148).

The above picture is showing:

a. Sauce and cheese retinopathy

b. Candle –wax drippings

c. Splashed –sauce appearance

d. Headlight-in –fog appearance

Solution. Ans 185: (c) Splashed –sauce appearance


Ref:Read the text below
Sol:
• It is showing splashed-sauce appearance which is a feature of CRVO.

Correct Answer. c

(149). Composite bone matrix strength is due to :

a. Phosphate

b. Calcium Hydroxyapatite

c. Osteocalcin

d. Collagen

Solution. Ans-: (b) Calcium Hydroxyapatite


Ref.:Read the text below
Sol :
The primary tissue of bone, osseous tissue, is a relatively hard and lightweight composite material.
It is mostly made up of a composite material incorporating the mineral calcium phosphate in the chemical arrangement termed
calcium hydroxylapatite(this is the osseous tissue that gives bones their rigidity) and collagen, an elastic protein which improves fracture
resistance.
It has relatively high compressive strength of about 170 MPa (1800 kgf/cm²) but poor tensile strength of 104–121 MPaand very low shear
stress strength (51.6 MPa)

Correct Answer. b

Copyright © 2014 Delhi Academy of Medical Sciences, All Rights Reserved. 57/118
(150). Mode of repair in “tendon injury of hand” :

a. Tidy wounds are treated with secondary repair

b. Both tendon as well as the flexor sheaths are repaired together

c. Tidy wounds are treated with primary repair

d. None of the above.

Solution. Ans-: (c) Tidy wounds are treated with primary repair
Ref.:Read the text below
Sol :
The palmar aspect of hand is divided into 8 zones for purposes of treatment of cut flexor tendons.
Zone I– From the base of the distal phalanx to the middle of the middle phalanx: contains only one tendon F.D.P.
Zone II– From the middle of the middle phalanx to the distal palmar crease : contains two tendons F.D.P. and F.D.S.
Zone III– From the distal palmar crease to the distal margin of the flexor Retinaculum.
Zone IV– Flexor Retinaculum – Carpal tunnel.
Zone V– Distal forearm – proximal to the wrist crease.

Correct Answer. c

(151). The abnormal deposition of calcium pyrophosphate dehydrate crystals (CPPD) would not cause :

a. Pseudogout

b. Chondrocalcinosis

c. Apical plate excrescences

d. Pseudoankylosing spondylitis

Solution. Ans-: (d) Pseudoankylosing spondylitis


Ref.:Read the text below
Sol :
Calcium pyrophosphate dehydrate deposition disease.
Pseudogout,Chondrocalcinosis,Apical plate excrescences,Calcium crystal arthritis ,Pyrophosphate arthropathy
Calcium pyrophosphate deposition disease (CPDD) is a variety of arthritis caused by the deposition of calcium pyrophosphate crystals.
CPDD is divided into several varieties, primarily pseudogout and chondrocalcinosis.
Pseudogout, an acute gout like arthritis, is recognized by characteristic synovial calcium pyrophosphate crystals seen on polarizing
microscopy.

Correct Answer. d

(152). All of the following are done in osteoarthrosis except :

a. Synovectomy

b. Osteotomy

c. Arthrodesis

d. Arthroplasty

Solution. Ans-: (a) Synovectomy


Ref.:Read the text below
Sol :
Operative Treatment of Osteoarthrosis :
The procedures which may be adopted in osteoarthritis include :
Osteotomy :Pain is sometimes relieved by correcting deformity. If also initiates biological regeneration by reducing vascular
engorgement in the subchondral bone.
Replacement Arthroplasty : This is indicated in case of old persons. E.g. are total knee and hip replacements.
Arthrodesis :This is best in abolishing pain. But this offers total stiffness. In this process the joint bones are fused together.
Excision of the joint.
Manipulation of the joint under anesthesia with hydrocortisone injection. This will increase joint movement and will minimize fibrosis of
the capsule.
This is indicated which the main complaint of the patient is stiffness of the joint and not pain.

Correct Answer. a

Copyright © 2014 Delhi Academy of Medical Sciences, All Rights Reserved. 58/118
(153). Barlow’s test and Ortolani’s test is useful in :

a. Instability and dislocation of shoulder joint

b. Instability and dislocation of hip joint

c. Instability of knee joint

d. Fracture knee joint

Solution. Ans-: (b) Instability and dislocation of hip joint


Ref.:Read the text below
Sol :
The Ortolani test or Ortolani maneuver is part of the physical examination fordevelopmental dysplasia of the hip, along with the Barlow
maneuver.
It relocates the dislocation of the hip joint that has just been elicited by the Barlow maneuver.
It is performed by an examiner first flexing the hips and knees of a supine infant to 90 degrees, then with the examiner's index fingers
placing anterior pressure on thegreater trochanters, gently and smoothly abducting the infant's legs using the examiner's thumbs.
A positive sign is a distinctive 'clunk' which can be heard and felt as the femoral head relocates anteriorly into the acetabulum

Correct Answer. b

(154). A child Knows age and sex; counts 3 objects correctly; repeats 3 numbers or a sentence of 6 syllables; most of speech intelligible to
strangers at ......years

a. 1

b. 2

c. 3

d. 4

Solution. Ans 289: (c) 3


Ref– Read the text below
Sol:
IMPORTANT MILESTONES AT THREE YEARS
Social and Emotional
Copies adults and friends
Shows affection for friends without prompting
Takes turns in games
Shows concern for crying friend
Understands the idea of “mine” and “his” or “hers”
Shows a wide range of emotions
Separates easily from mom and dad
May get upset with major changes in routine
Dresses and undresses self
Language/Communication
Follows instructions with 2 or 3 steps
Can name most familiar things
Understands words like “in,” “on,” and “under”
Says first name, age, and sex
Names a friend
Says words like “I,” “me,” “we,” and “you” and some plurals (cars, dogs, cats)
Talks well enough for strangers to understand most of the time
Carries on a conversation using 2 to 3 sentences
Cognitive (learning, thinking, problem-solving)
Can work toys with buttons, levers, and moving parts
Plays make-believe with dolls, animals, and people
Does puzzles with 3 or 4 pieces
Understands what “two” means
Copies a circle with pencil or crayon
Turns book pages one at a time
Builds towers of more than 6 blocks
Screws and unscrews jar lids or turns door handle
Movement/Physical Development
Climbs well
Runs easily
Pedals a tricycle (3-wheel bike)
Walks up and down stairs, one foot on each step

Correct Answer. c

Copyright © 2014 Delhi Academy of Medical Sciences, All Rights Reserved. 59/118
(155). Enuresis is defined as the repeated voiding of urine into clothes or bed at least twice a week for at least 3 consecutive months in a child
who is at least.......yr of age.

a. 3

b. 4

c. 5

d. 6

Solution. Ans 290: (c) 5


Ref– Read the text below
Sol:
Enuresis is defined as the repeated voiding of urine into clothes or bed at least twice a week for at least 3
consecutive months in a child who is at least 5 yr of age and has a high prevalence in school aged children.
In primary enuresis (PE) children have never gained control over urination or has been dry for<6months.
While in secondary enuresis children have developed incontinence after a period of at least 6 months of urinary
control.

Correct Answer. c

(156). National Institutes of Health Consensus Development Panel recommends Phenylalanine restricted diet till

a. 12 yrs

b. 18 yrs

c. 20yrs

d. Life

Solution. Ans 291: (d) Life


Ref– Read the text below
Sol:
Restriction of dietary phenylalanine. The generally accepted goal of treatment for the hyperphenylalaninemias is
normalization of the concentrations of Phe (phenylalanine) and Tyr (tyrosine) in the blood and thus prevention of
the cognitive deficits that are attributable to this disorder are generally regarded as safe.
A diet restricted in Phe should be initiated as soon as possible after birth and continued at least into adolescence
[Pietz et al 1998], perhaps for life [National Institutes of Health Consensus Development Panel ].
It is clear that if the diet is not followed closely (especially during childhood) and if plasma Phe concentration is
allowed to rise frequently above the recommended concentration, some impairment is inevitable.

Correct Answer. d

Copyright © 2014 Delhi Academy of Medical Sciences, All Rights Reserved. 60/118
(157). A 10 day old child presents with fever, jaundice, seizures and hypoglycemia. His Liver function tests show
Bilirubin 5 mg/dl
SGOT 150 IU/L
SGPT 100 IU/L
Blood glucose 10mg/dl
The most probable diagnosis is

a. Galactosemia

b. HFI

c. Tyrosenemia

d. Alpha AT deficiency

Solution. Ans 292: (a) Galactosemia


Ref– Nelson's textbook of Pediatrics 19th Edition; 81.2
Sol:
The diagnosis of uridyl transferase deficiency should be considered in newborn or young infants with any of the
following features: jaundice, hepatomegaly, vomiting, hypoglycemia, seizures, lethargy, irritability, feeding
difficulties, poor weight gain or failure to regain birth weight, aminoaciduria, nuclear cataracts, vitreous
hemorrhage, hepatic failure, liver cirrhosis, ascites, splenomegaly, or mental retardation.
Symptoms are milder and improve when milk is temporarily withdrawn and replaced by intravenous or lactosefree
nutrition.
Patients with galactosemia are at increased risk for Escherichia coli neonatal sepsis; the onset of sepsis often
precedes the diagnosis of galactosemia. Pseudotumor cerebri can occur and cause a bulging fontanel.
Death from liver and kidney failure and sepsis may follow within days.
When the diagnosis is not made at birth, damage to the liver (cirrhosis) and brain (mental retardation) becomes
increasingly severe and irreversible.

Correct Answer. a

(158). According to WHO cut off level for hemoglobin in child for anemia is?

a. 10 gm

b. 11 gm

c. 12 gm

d. 13 gm

Solution. Ans-293: (b) 11 gm


Ref.: Read the text below
Sol :
The definition of anemia in infancy and childhood is different from adults.

Correct Answer. b

Copyright © 2014 Delhi Academy of Medical Sciences, All Rights Reserved. 61/118
(159). A child is having nocturnal asthmatic attack 2 times in a week, day time attack is 3 times or more, can be categorized as?

a. Severe persistent asthma

b. Moderate persistent asthma

c. Mild intermittent

d. Mild persistent

Solution. Ans-294: (b) Moderate persistent asthma


Ref.: Read the text below
Sol :
Intermittent asthma
Asthma is considered intermittent if without treatment any of the following are true:
Symptoms (difficulty breathing,wheezing, chest tightness, and coughing):
Occur on fewer than 2 days a week.
Do not interfere with normal activities.
Mild persistent asthma
Asthma is considered mild persistent if without treatment any of the following are true:
Symptoms occur on more than 2 days a week but do not occur every day.
Attacks interfere with daily activities.
Nighttime symptoms occur 3 to 4 times a month.
Lung function tests are normal when the person is not having an asthma attack. The results of these tests are 80%
or more of the expected value and may vary a small amount (PEF varies 20% to 30%) from morning to afternoon.
Moderate persistent asthma
Asthma is considered moderate persistent if without treatment any of the following are true:
Symptoms occur daily. Inhaled short-acting asthma medication is used every day.
Symptoms interfere with daily activities.
Nighttime symptoms occur more than 1 time a week, but do not happen every day.
Lung function tests are abnormal (more than 60% to less than 80% of the expected value), and PEF varies more
than 30% from morning to afternoon.
Severe persistent asthma
Asthma is considered severe persistent if without treatment any of the following are true:
Symptoms:
Occur throughout each day.
Severely limit daily physical activities.
Nighttime symptoms occur often, sometimes every night.
Lung function tests are abnormal (60% or less of expected value), and PEF varies more than 30% from morning to
afternoon.

Correct Answer. b

(160). A child having IQ of 25 falls in which category?

a. Educable

b. Trainable

c. Dependent

d. Needs life support

Solution. Ans-295: (c) Dependent


Ref.: Read the text below
Sol :
Classification of mental retardation :
Mild MR Moderate MR Severe MR Profound MR
IQ 50-70 35-50 20-35 <20
Quality of life Educable Trainable Dependent Needs life support
Progress Class 6 Class 2 - -

Correct Answer. c

Copyright © 2014 Delhi Academy of Medical Sciences, All Rights Reserved. 62/118
(161). All of the following cause proximal muscle weakness except:

a. Polymyositis

b. Duchene muscular dystrophy

c. Myotonic dystrophy

d. Becker’s muscular dystrophy

Solution. Ans-296: (c) Myotonic dystrophy


Ref.: Read the text below
Sol :
Patterns of muscle weakness Conditions
Proximal > distal Polymypositis, Dermatomyositis, Muscular dystrophy
Oculopharyngeal muscular dystrophy, mitochondrial myopathy and
Ptosis, Extra ocular muscles
microtubular myopathy
Facial and scapular winging Facioscapulohumeral dystrophy
Facialk distal handgrip, quadriceps Myotonic dystrophy
Proximal and distal handgrips and quadriceps IBM
Distal Distal myopathy
Dropped head Myasthenia gravis, polymyositis, Amyotrophic lateral sclerosis

Correct Answer. c

(162). The best test to diagnose metabolic disorders at birth is

a. ELISA

b. PCR

c. TMS

d. GCMS

Solution. Ans 297: (c) TMS


Ref– Reference: Nelson's textbook of Pediatrics 19th Edition; Chapter 78
Sol:
An Approach to Inborn Errors of Metabolism During the past half-century, methods have been developed to
screen all infants inexpensively with accurate and fast-yielding results.
Tandem mass spectrometry (MS/MS) is the latest technical advance in the field. This method requires a few drops
of blood to be placed on a filter paper and mailed to a central laboratory for assay.

Correct Answer. c

(163). Satelite sequences during G0 phase are seen in?

a. Nucleolus

b. Terminal cenntrioles

c. Kinetochore

d. Chromosome

Solution. Ans-298: (c) Kinetochore


Ref.: Read the text below
Sol :
“Satellite sequences are repetitive sequences in a chromosome and form the non transcriptional structural portion
and are mainly located around the cenntromere of the chromosome, the area also known as kinetochore”

Correct Answer. c

Copyright © 2014 Delhi Academy of Medical Sciences, All Rights Reserved. 63/118
(164). Which is the commonest cardiac lesion in Tuberous Sclerosis?

a. ASD

b. Mitral Stenosis

c. Rhabdomyoma

d. VSD

Solution. Ans-299: (c) Rhabdomyoma


Ref.: Read the text below
Sol :
Tuberous sclerosis :
Also known as Bourneville’s disease
It is a neurocutaneous syndrome
Genetics :
Autosomal dominant
It is caused by mutation of either of :
TSC1 gene on chromosome 9 q which encodes a protein Hamartin
TSC2 gene on chromosome 16p which encodes a protein Tuberin (Hamartin and Tuberin together form a complex
that negatively regulate cell growth and proliferation by inhibition of mTOR. So, mutation causes uncontrolled
proliferation in numerous tissues including kidneys, skin, CNS and heart.

Correct Answer. c

(165). Expressed breast milk can be store in room temperature for how many

a. 4

b. 8

c. 16

d. 24

Solution. Ans-300: (b) 8


Ref.: Read the text below
Sol :
Expressed breast milk can be stored in :
Room temperature for 8 hours.
Refrigerator for 24 hours
Deep freezer for 3 months

Correct Answer. b

Copyright © 2014 Delhi Academy of Medical Sciences, All Rights Reserved. 64/118
(166). Antiapoptiotic proteins are

a. Bcl-2 and Bcl-x

b. Apaf-1

c. Bax and Bim

d. Bim and Bid

Solution. Ans-: (a) Bcl-2 and Bcl-x


Ref.:Read the text below
Sol :
§Phagocytosis of apoptotic cells/cell bodies, usually by macrophages
Pro-apoptotic factors Anti-apoptotic factors
Intrinsic (Mitochondrial) Pathway Extrinsic (Death receptor)
· Apaf-1 Pathway §Bcl-2
· BH3-only proteins: Bim, Bid, Bad §Type 1 TNF receptor §Bcl-x
· Bax §(TNFR1) §Mcl-1
· Bak §Fas (CD95)
Initiator caspases : caspase 8 and 10
Executioner caspases : caspase 3 and 6
P53

Correct Answer. a

(167). Granulomatous tissue response may be seen in all of the following diseases except :-

a. Sarcoidosis

b. Hypersensitivity pneumonitis

c. Fungal pneumonia

d. Pneumocystis pneumonia

Solution. Ans-: (d) Pneumocystis pneumonia


Ref:Read the text below
Sol:
· Pneumocystis pneumonia presents histologically with alveolar spaces filled with a foamy amorphous material consisting of cell debris
and organisms.
· Granulomas are not produced.

Correct Answer. d

(168). Red infarct is seen in

a. Lung

b. Heart

c. Kidney

d. Spleen

Solution. Ans-: (a) Lung


Ref.:Read the text below
Sol :
Red (hemorrhagic) infarcts White (anemic) infarcts
§Ovary (torsion, venous occlusion) §Arterial occlusion in solid organ with end arterial circulation
§Lung (dual circulation, loose tissue) §Heart
§Small intestine (dual circulation) §Kidney
§Heart (after angioplasty) §Spleen

Correct Answer. a

Copyright © 2014 Delhi Academy of Medical Sciences, All Rights Reserved. 65/118
(169). DNA content and membrane antigen in tumor cells are measured by

a. Flow cytometry

b. Immunocytometry

c. DNA microarray

d. Tumour markers

Solution. Ans-: (a) Flow cytometry


Ref.:Read the text below
Sol :
Flow cytometry
Rapidly and quantitatively measure membrane antigens and the DNA content of the cell
Useful in identification & classification of tumors arising from T and B lymphocytes and mononuclear cells immunohistochemistry
Categorization of undifferentiated malignant tumors
Determination of site of origin of metastatic tumors
Detection of molecules that have prognostic or therapeutic significance
DAN microarray analysis and proteomics : Used to obtain gene expression signatures (molecular profiles) of cancer cells.

Correct Answer. a

(170). Which of the following statement is NOT true for Mycosis fungoides :-

a. M/C cutaneous lymphoma

b. Pautrier’s abscess

c. Curable

d. Erythroderma present

Solution. Ans-: (c) Curable


Ref:Read the text below
Sol:
· Mycosis fungoides is also known as cutaneous T cell lymphoma
· Mycosis fungoides is an indolent lymphoma with patients often having several years of eczematous or dermatitis skin lesions before
the diagnosis is finally established. A particular syndrome in patients with this lymphoma involves erythroderma and circulating tumor
cells. This is known as Sezary’s syndrome.
· Rare patients with localized early stage mycosis fungoides can be cured with radiotherapy, often total-skin electron beam irradiation.
More advanced disease has been treated with topical glucocorticoids,topical nitrogen mustard, phototherapy, psoralen with ultraviolet A
(PUVA), electron beam radiation, interferon,antibodies, fusion toxins, and systemic cytotoxic therapy. Unfortunately, these treatments
are palliative.

Correct Answer. c

(171). Following is not true concerning amyloidosis:-

a. Amyloidosis associated with multiple myeloma has the poorest prognosis.

b. Fine - needle biopsy of subcutaneous abdominal fat is a simple & reliable method for diagnosing secondary systemic amyloidosis.

c. Hepatic amyloid disease produces hepatomegaly but rarely jaundice.

d. Amyloidosis of the spleen is associated with severe anemia.

Solution. Ans-: (d) Amyloidosis of the spleen is associated with severe anemia.
Ref:Read the text below
Sol:
· Amyloidosis of the spleen characteristically is not associated with leukopenia & anemia.
· When Amyloidosis develops in patients with multiple myeloma, manifestations leading to initial hospitalization are more apt to be
related to amyloid disease than to myeloma.
· In these cases prognosis is very poor, and life expectancy is usually less than 6 months.

Correct Answer. d

Copyright © 2014 Delhi Academy of Medical Sciences, All Rights Reserved. 66/118
(172). A newborn infant manifests weakness, poor motor tone, and decreased spontaneous motor activity.The possible diagnoses include all of
the following except :-

a. Congenital fiber-type disproportion

b. Progressive spinal muscular atrophy

c. Duchenne muscular dystrophy

d. Genetic defect of glycogen metabolism

Solution. Ans-: (c) Duchenne muscular dystrophy


Ref:Read the text below
Sol:
· Duchenne muscular dystrophy is X-linked, How ever, newborn boys with this disease are normal at birth and during the very early
stages of development.

Correct Answer. c

(173). A 24- year- old man with acute appendicitis undergoes surgical removal of the inflamed appendix . The incision site is sutured . A
trichrome- stained section of the site is shown in the figure above. How long after the surgery would this appearance most likely be seen?

a. 1 day

b. 2 to 3 days

c. 4 to 5 days

d. 4 weeks

Solution. Ans-: (d) 4 weeks


Ref:Read the text below
Sol :
The figure shows dense collagen with some remaining dilated blood vessels, typical of the final phase of wound healing , which is
extensive by the end of the first month.

Correct Answer. d

(174). Which subtype of Hodgkin lymphoma is negative for CD15 and CD30 markers :-

a. Nodular sclerosis

b. Mixed cellularity

c. Lymphocyte depletion

d. Lymphocyte predominance

Solution. Ans-: (d) Lymphocyte predominance


Ref:Read the text below
Sol:
· Lymphocyte predominance subtype has tumor cells negative for CD15 and CD30; they are positive for CD20.

Correct Answer. d

Copyright © 2014 Delhi Academy of Medical Sciences, All Rights Reserved. 67/118
(175). Metastasis to heart are mostly from :-

a. Breast

b. Prostate

c. Ovary

d. All

Solution. Ans-: (a) Breast


Ref:Read the text below
Sol:
· Most frequent tumors involving heart are lung, breast, oesophagus (directly) and kidney, liver (by direct contiguous venous extension)
Also remember "Spider cells" seen in Rhabdomyoma, which is most frequent primary tumor of heart in children.
· Major current limitation to long term success of cardiac transplantation is "Late, progressive, diffuse stenosing intimal proliferation of
coronaries" (Graft arteriopathy).
· It may lead to Silent MI.

Correct Answer. a

(176). A 15-month-old girl presents with a large abdominal mass, weight loss, and fever. At surgery,a large infiltrative tumor with areas of
hemorrhage and necrosis is removed. A photomicrograph of a section from this tumor is shown in Fig. Which of the following is the most
likely diagnosis?

a. Embryonal rhabdomyosarcoma

b. Malignant lymphoma

c. Neuroblastoma

d. Teratoma

Solution. Ans-: (c) Neuroblastoma


Ref:Read the text below
Sol:
· This is a neuroblastoma and is one of a group of childhood tumors described as “small, round, blue-cell tumors,” consisting as they do
of monotonous small cells with dense, blue nuclei.
· The characteristic microscopic feature of a neuroblastoma is the pseudorosette, a ring of primitive neuroblasts surrounding a central
space filled with fibrillar extensions from the cells. Many of these can be seen in Fig.
· These are called pseudorosettes because they do not have a central lumen as is found, for example, in the rosettes in
retinoblastoma.Embryonal rhabdomyosarcoma (choice A) is a type of rhabdomyosarcoma typically found in children under the age of 10.
It can arise in a number of locations and sarcoma botryoides is one form. It does not form rosettes.
· Malignant lymphomas and leukemias are together the most common malignancies of childhood in the United States, but rosette
formation is not a distinctive microscopic feature.
· Teratomas in infants and young children are usually benign tumors found in the midline (e.g., sacrococcygeal, mediastinal). They are
composed of tissues with a normal histologic appearance derived from all three germ layers.
·

Correct Answer. c

Copyright © 2014 Delhi Academy of Medical Sciences, All Rights Reserved. 68/118
(177). Jigsaw puzzle pattern is seen in

a. Cylindroma

b. Eccrine poroma

c. Tricho epitheliosis

d. Basal cell carcinoma

Solution. Ans-: (a) Cylindroma


Ref.:Read the text below
Sol :
· Cylindroma is a benign adnexaltumor, which occurs as solitary dermal nodules on the scalp and forehead.
· Cylindroma of the breast is a rarelesion. Fine-needle aspiration is often the first line of investigation.
· In this communication, we illustrate the cytomorphological feature of cylindroma in a 61-year-old female patient who presented with a
breast lump.
· Fine-needle aspiration cytology in this case reveals the islands of tumor cells formed a “jig-saw” pattern, along with the bland, basaloid
cells associated with globular, extracellular material.

Correct Answer. a

(178). All are seen in McCune Albright syndrome except

a. Precocious puberty

b. Osteogenesis imperfects

c. Café au lait spots

d. Polyostotic fibrous dysplasia

Solution. Ans-: (b) Osteogenesis imperfects


Ref.:Read the text below
Sol :
McCune–Albright syndrome is suspected when two of the three following features are present:
· Autonomous endocrinehormone excess, such as in precocious puberty
· Polyostotic fibrous dysplasia
· Unilateral, i.e. one-sided Café-au-lait spots
· Within the syndromethere are bone fractures and deformity of the legs, arms and skull, different pigment patches on the skin, and
early puberty with increased rate of growth.
· Approximately 20-30% of fibrous dysplasiasare polyostotic, which means fibrous dysplasia and sclerotic bone are present in multiple
sites; two thirds of patients are polyostotic before the age of ten. The disease frequently involves the skulland facial bones, pelvis, spine
and shoulder girdle. The sites of involvement are the femur(91%), tibia(81%), pelvis (78%), ribs, skull and facial bones (50%), upper
extremities, lumbar spine, clavicle, and cervical spine, in decreasing order of frequency. The craniofacial pattern of the disease occurs in
50% of patients with the polyostotic form of fibrous dysplasia.

Correct Answer. b

(179). Kimmelstiel–Wilson disease is related with

a. Benign nephrosclerosis

b. Amyloidosis

c. Diabetic glomerulosclerosis

d. SLE

Solution. Ans-: (c) Diabetic glomerulosclerosis


Ref.:Read the text below
Sol :
· Diabetic nephropathy(nephropatia diabetica), also known as Kimmelstiel–Wilson syndrome, or nodular diabetic glomerulosclerosis and
intercapillary glomerulonephritis, is a progressive kidney diseasecaused by angiopathyof capillariesin the kidneyglomeruli.
· It is characterized by nephrotic syndromeand diffuse glomerulosclerosis.
· It is due to longstanding diabetes mellitus, and is a prime indication for dialysisin many Western countries.
· It is classified as a microvascularcomplication of diabetes

Correct Answer. c

Copyright © 2014 Delhi Academy of Medical Sciences, All Rights Reserved. 69/118
(180). Poor prognostic marker of AML

a. Inv (16)

b. t (8;21)

c. t (15;17)

d. inv (3)

Solution. Ans-: (d) inv (3)


Ref:Read the text below
Sol:
Chromosome findings with good prognosis.
· Patients with
Ø t(8;21),
Ø inv(16)
Ø t(15;17)
· While those with no cytogenetic abnormality have a moderately favorable outcome when treated with high-dose cytarabine.

Chromosome findings with Poor prognosis


· Patients with
Ø a complex karyotype
Ø inv(3), or –7
· Molecular markers such as the presence of a PTD7 of MLL or the ITD8 of FLT3 may also predict poor outcome of AML patients who
otherwise have an intermediate prognosis.

Correct Answer. d

(181). A 5-week-old girl, who appeared to be healthy at birth, develops diarrhea and vomiting a few days after birth. Your current examination
reveals that she has hepatomegaly, jaundice, and early cataract formation and is not meeting developmental milestones. You suspect that
she has which of the following conditions?

a. Galactosemia

b. Hurler syndrome

c. Pyloric stenosis

d. Tay-Sachs disease

Solution. Ans-: (a) Galactosemia


Ref:Read the text below
Sol:
· Galactosemia is an autosomal recessive disorderdue (in this more common and more severe form of the disease) to a lack of galactose-
1-phosphate uridyl transferase. This results in the formation and accumulation of galactose metabolites. If the infant’s diet is not
modified to exclude milk products, this will result in damage to the liver (fatty change, cholestasis, cirrhosis, liver failure), eyes (cataract
formation), and brain (mental retardation).
· Hurler syndromeis a severe form of mucopolysaccharidosis that typically becomes apparent between 6 months and 2 years of age.
· Prominent features include coarse facies,dwarfism, organomegaly, cataracts, and mental retardation, not diarrhea, vomiting, and
jaundice.
· Pyloric stenosiscan occur as a congenital condition, more frequently in baby boys (M:F = 4:1). It is marked by projectile vomiting in
the first month of life, but not the other findings in this case.
· Tay-Sachs disease is a lipid storage disease due to a deficiency of hexosaminidase A. There is an inexorable deterioration of mental
and motor functions within a few months of birth culminating in a vegetative state and death within 3 or 4 years.

Correct Answer. a

Copyright © 2014 Delhi Academy of Medical Sciences, All Rights Reserved. 70/118
(182). All of the following are the good prognostic features for Hodgkin’s disease except :-

a. Haemoglobin >10gm/dl

b. WBC count < 15000/mm3

c. Absolute lymphocyte count <600/ml

d. Age<45 yrs.

Solution. Ans-: (c) Absolute lymphocyte count <600/ml


Ref:Read the text below
Sol:
The following seven adverse prognostic factors were described for advanced Hodgkin’s disease:
1. Male gender
2. Age > 45 years
3. Stage IV disease
4. Haemoglobin <10.5g/dl
5. Leucocytosis with WBC>15000
6. Lymphocytopenia with either one or both of the criteria :
a. Absolute lymphotcyte count <600/ml
b. Lymphocytes<8% of WBC
7. A Serum albumin level <4g/dl

Correct Answer. c

(183). A 10- year old boy presents with 2-cm spongy , dull red, circumscribed lesions on the upper outer left arm. The presents state that this
lesion has been present since infancy. The lesion is excised , and its microscopic appearance is shown. Which of the following is the most
likely diagnosis?

a. Kaposi sarcoma

b. Angiosarcoma

c. Lymphangioma

d. Hemangioma

Solution. Ans-: (d) Hemangioma


Ref: Read the text below
Sol :
· The figure shows dilated, endothelium- lined spaces filled with RBCs. The circumscribed nature of this lesion and its long, unchanged
course suggest its benign nature.

Correct Answer. d

Copyright © 2014 Delhi Academy of Medical Sciences, All Rights Reserved. 71/118
(184). If a drug has a short duration of action, the action can be prolonged by all, except

a. Enteric coating and other methods to retard absorption

b. Interfering with renal excretion

c. Inhibiting drug metabolism

d. Parenteral administration of the drug

Solution. Ans-110: (d) Parenteral administration of the drug


Ref:Read the text below
Sol :
- The action can be prolonged by retarding absorption , Metabolism or excretion.
- Paranteral administration actually shortens the duration of action of the drugs.

Correct Answer. d

(185). All are highly plasma protein bound, except

a. Warfarin

b. Atenolol

c. Digitoxin

d. Phenytoin

Solution. Ans-111: (b) Atenolol


Ref:Read the text below
Sol :
- Atenolol is not much plasma protein bound.

Correct Answer. b

(186). Most nicotinic cholinergic activity is seen with

a. Bethanechol

b. Methacholine

c. Carbachol

d. Pilocarpine

Solution. Ans-112: (c) Carbachol


Ref:Read the text below
Sol :
- Carbachol has maximum nicotinic cholinergic activity.

Correct Answer. c

(187). The selective beta-2 sympathomimetic drugs differ from epinephrine in all of the following aspects, except

a. They are active when administered orally

b. They have a slower onset of action

c. They have a shorter duration of action

d. Generally do not stimulate the myocardium directly

Solution. Ans-113: (c) They have a shorter duration of action


Ref:Read the text below
Sol :
- The beta-2 agonists like salbutamol and terbutaline have longer duration of action than adrenaline.

Correct Answer. c

Copyright © 2014 Delhi Academy of Medical Sciences, All Rights Reserved. 72/118
(188). A patient has returned from a mountain trekking with a painful sprained ankle as well as dry cough. Which of the following agents would
relieve both the pain and the cough?

a. Meperidine

b. Naoxone

c. Dextromethorphan

d. Codeine

Solution. Ans-114: (d) Codeine


Ref:Read the text below
Sol :
- Of the drugs listed in the question , codeine has both antitussive as well as analgesic effects.
- The analgesic meperidine (pethindine) has no antitussive activity.
- The cough suppressant dextromethorphan has no analgesic activity.
- Naloxone has neither effect: it is a pure opiod antagonist and is used to reverse the unwanted effects of opioids
(e.g. , in neonates whose mother received morphine during labor).

Correct Answer. d

(189). A young child who ingests a fatal dose of dimenhydrinate will most likely die from

a. Renal dysfunction

b. Generalized CNS stimulation with convulsions

c. Anaphylactic shock

d. Serve cardiac arrhythmia

Solution. Ans-115: (b) Generalized CNS stimulation with convulsions


Ref:Read the text below
Sol :
- Dimenhydrinate is a “classic” antihistaminic (H-1 receptor antagonist) agent. A toxic dose of dimenhydrinate
results in generalized CNS stimulation with convulsions (although lower doses are CNS depressant).
- Bronchospasm, suffocation, cardiac arrhythmias and renal dysfunction are not the adverse effects of H-1
receptor antagonists.
- Allergic manifestations may be observed with topical application of the drugs, but these are not severe enough
to constitute anaphylactic shock and would not be lethal.

Correct Answer. b

(190). Sudden hearing loss most likely occur after the administration of which one of the following

a. Mannitol

b. Ethacrynic acid

c. Chlorthiazide

d. Metolazone

Solution. Ans-116: (b) Ethacrynic acid


Ref:Read the text below
Sol :
- Ethacrynic acid is known to cause sudden hearing loss, however not commonly.
- Any high ceiling diuretic (e.g. frusemide) may cause ototoxicity especially in case of renal insufficiency .

Correct Answer. b

Copyright © 2014 Delhi Academy of Medical Sciences, All Rights Reserved. 73/118
(191). A 67 year-old man is found to have tachypnea, pitting edema, and audible S3 and S4 heart sounds. A chest X-ray supports the diagnosis
of congestive heart failure. Which of the following is an appropriate diuretic ?

a. Mannitol

b. Hydrochlorothiazide

c. Bumetanide

d. Spironolactone

Solution. Ans-117: (c) Bumetanide


Ref:Read the text below
Sol :
- A rapid-acting potent “loop” diuretic is needed in a patient presenting with acute congestive heart failure.
- Bumetanide (like furosemide) is such a diuretic.
- The mannitol, hydrochlorothiazide and spironolactone would not give the immediate diuretic response that is
needed.

Correct Answer. c

(192). Which of the following is not an adverse effect of chronic amiodarone therapy:

a. Pulomonary fibrosis

b. Hypothyroidism

c. Hyperthyroidism

d. Systemic lupus erythematosus

Solution. Ans-118: (d) Systemic lupus erythematosus


Ref:Read the text below
Sol :
AMIODARONE:
A Antiarrhythmic
M Multiple actions i.e. block K+, Na+, Ca++ channel & receptors.
I Iodine containing (so interfere with thyroid functions: hypo- or Hyper-thyroidism).
O Orally used mainly.
D Duration of action is very long (t ½ 3-8 weeks)
A APD & ERP increases (so conduction decreases & depress ectopics).
R Ressitant AF, VT & Recurrent VF are the only clinical uses because of it’s toxicity.
O On I.V injection myocardial depression & Blood pressure decrease.
On prolonged use pulmonary fibrosis and alveolitis.
N Neuropathy may occur.
E Eye: Corneal microdeposits may occur.

Correct Answer. d

(193). Which of the following relieves angina-pectoris by decreasing myocardial work, but may precipitate CHF

a. Phentolamine

b. Phenoxybenzamine

c. Propranolol

d. Atropine

Solution. Ans-119: (c) Propranolol


Ref:Read the text below
Sol :
- Propranolol is useful in angina pectoris.
- It acts reducing oxygen demand by decreasing myocardial contractility.
- Propranolol may precipitate CHF by sympatholytic action.

Correct Answer. c

Copyright © 2014 Delhi Academy of Medical Sciences, All Rights Reserved. 74/118
(194). A patient with severe hypertension us being treated with an agent that stimulates presynaptic alpha2 receptors in the vasomotor center
of the brain. The patient might be taking

a. Guanethidine

b. Reserpine

c. Clonidine

d. Prazosin

Solution. Ans-120: (a) Guanethidine


Ref:Read the text below
Sol :
- Clonidine, by stimulating presynaptic α2 – adrenergic receptor in the vasomotor center of the brain, decreases
sympathetic outflow to the peripheral vessels.
- Both guanethidine and reserpine are agents that block postganglionic adrenergic neurons.
- Prazosin is a selective postsynaptic α1 – adrenergic receptor blocker.

Correct Answer. a

(195). With Which of the following theophylline has an antagonistic interaction ?

a. Histamine receptors

b. Bradykinin receptors

c. Adenosine receptors

d. Imidazoline receptors

Solution. Ans-121: (c) Adenosine receptors


Ref:Read the text below
Sol :
- Theophylline (methyl-xanthines) acts by blocking adenosine receptors.
- Since adenosine causes smooth muscle contraction, Theophyllines cause smooth muscle relaxation and hence
chodilatation.

Correct Answer. c

(196). A patient who is undergoing menopause complains that the “hot flashes” are extremely disturbing. Which of the following drugs could
relieve her symptoms.

a. Ethinyl estradiol

b. Ethynodiol

c. Nandrolone

d. Fluoxymesterone

Solution. Ans-122: (a) Ethinyl estradiol


Ref:Read the text below
Sol :
- Estrogens are used during and after menopause to relieve vaso motor symptoms such as flushing.
- Ethinyl, estradiol, an estrogen would be indicated.
- Ethynodiol, a progestin, nandrolone, an anabolic steroid, and fluoxymesterone, an androgen, are not indicated.

Correct Answer. a

Copyright © 2014 Delhi Academy of Medical Sciences, All Rights Reserved. 75/118
(197). Maximum glucocorticoid activity is found in

a. Dexamethasone

b. Paramethasone

c. Hydrocortisone

d. Cortisone

Solution. Ans-123: (a) Dexamethasone


Ref:Read the text below
Sol :
- Maximum glucocorticoid activity is found in dexamethasone (and betamethasone).

Correct Answer. a

(198). Which of the following laxatives is a surface active agent that eases defection principally by softening the stool

a. Dioctyl sod. Sulfosuccinate

b. Phenolphthalein

c. Magnesium hydroxide

d. Bisacodyl

Solution. Ans-124: (a) Dioctyl sod. Sulfosuccinate


Ref:Read the text below
Sol :
- Stool softners are dioctyl sod. Sulfosuccinate (docusate) and liquid paraffin.

Correct Answer. a

(199). Corticosteroids cause all of the following , except

a. Hypertrophy of muscles

b. Hypokalemia

c. Neuropsychiatric symptoms

d. Hyperglycemia

Solution. Ans-125: (a) Hypertrophy of muscles


Ref:Read the text below
Sol :
- Corticosteroids do not cause hypertrophy of muscles.
- They cause myopathy and muscular weakness due to hypokalemia.
- Also, corticosteroids on long term administration cause neuropsychiatric symptoms like euphoria, mania and
psychosis.
- Hyperglycemia is also caused leading to precipitation/ aggravation of diabetes mellitus.

Correct Answer. a

Copyright © 2014 Delhi Academy of Medical Sciences, All Rights Reserved. 76/118
(200). Which of the following is not a dose related reaction

a. Crystalluria by sulpha drugs

b. Hypoglycemia by tolbutamide

c. Digitalis induced arrhythmia

d. Drugs fever of sulpha drugs

Solution. Ans-126: (d) Drugs fever of sulpha drugs


Ref:Read the text below
Sol :
- Sulphonamides (sulfa drugs) are notorious for causing allergic reactions like rashes, drugs fever, angioneurotic
edema, agranulocytosis and anaphylactic reactions.
- The allergic reactions are non-dose related (type II), i.e., may occur even at very little doses.
- The dose related (Type I) adverse effects are mainly extension of pharmacological effects of the drug, e.g., the
other three examples mentioned in the question. These can be avoided at lower doses.

Correct Answer. d

(201). Rate of excretion of sodium in a normal male when GFR = 125 mL/min is :

a. 400-425 μeq/min

b. 60-70 μeq/min

c. 50-60 μeq/min

d. 70-80 μeq/min

Solution. Ans-27: (b) 60-70 μeq/min


Ref.: Read the text below
Sol :
Quantitative aspects of Na reabsorption in a
Normal man
GFR = 125 mL/min; Plasma HCO = 27 meq/L; Plasma Na =
145 meq/L
18,125 ueq.
• Na+ filtered per minute 14,585 ueq.
• Reabsorbed with CI- 3,375 ueq.
50 ueq.
• Reabsorbed while reabsorbing 3375 ueq of HCO3
• Reabsorbed in association with formation of filterable acidity and ammonia 50 ueq.
• Reabsorbed in association with secretion of K+ 18,060 ueq.
• Total Na+ reabsorbed per minute

Correct Answer. b

Copyright © 2014 Delhi Academy of Medical Sciences, All Rights Reserved. 77/118
(202). In gametogenesis, meiosis differs significantly from mitosis. All of the following are characteristics of meiosis except :

a. Crossing over occurs between sister chromatids during prophase I

b. Daughter cells enter a second M phase of the cell cycle after division I without passing through the GI phase and S phase

c. Independent assortment of maternal and paternal genetic material occurs in division I

d. The result of division I is a chromosomal reduction to the haploid number.

Solution. Ans-28: (d) The result of division I is a chromosomal reduction to the haploid number.
Ref.: Read the text below
Sol :
. Gametogenesis uses meiosis to admix the parental genetic material and to produce a haploid gamete.
. There is pairing of homologous chromosomes and chromosomal duplication during prophase I.
. The paired homologues align in the metaphase and crossing over occurs between sister chromatids.
. Also at this stage the independent assortment of maternal and paternal genetic material occurs. Because the
chromosomal replication results in a tetraploid chromosomal number, division I produces cells with the diploid
number.
. After division I the daughter cells enter a second M phase of the cell cycle without passing through the GI phase
and S phase. Thus, the result of division. It is a chromosomal reduction to the haploid number.

Correct Answer. d

(203). All the following are characteristics of the plasma membrane except:

a. It serves as a selective filtration barrier through the facilitation of both active and passive transport

b. It possesses a thickness of 1 to 2/nm

c. It is involved in the generation of ionic gradients between the cytoplasm and the external environment

d. It serves as a sensor of signals from the extra-cellular environment

Solution. Ans-29: (b) It possesses a thickness of 1 to 2 nm


Ref.: Read the text below
Sol :
Cell membranes range in thickness from 7 to 10nm.
A cell membrane surrounds all eukaryotic cells and sub serves a number of essential functions.
The plasma membrane forms a boundary to the external environment and contains a large variety of receptors
that function as ligands for hormones, growth factors, cytokines, and other extra – cellular factors.
It’s a selective barrier that controls the entry and exit of substances to and from the cell.
The plasma membrane is involved in both energy – dependent (active) and energy independent (passive)
transport and the generation of ionic gradients between inside and outside of the cell.
The plasma membrane and the internal membranes of the cell such as those surrounding organelles like the
nucleus and mitochondria – are similar in their morphologic appearance under electron microscopy.
The organelles membranes are usually thinner and differ in overall biochemical composition from that of the
plasma membrane.

Correct Answer. b

Copyright © 2014 Delhi Academy of Medical Sciences, All Rights Reserved. 78/118
(204). Closure of the aortic valve occurs at the onset of which phase of the cardiac cycle?

a. Isovolumetric contraction

b. Rapid ejection

c. Protodiastole

d. Isovolumetric relaxation

Solution. Ans-30: (d) Isovolumetric relaxation


Ref.: Read the text below
Sol :
Closure of the semilunar valves (aortic and pulmonic valves) marks the beginning of the isovolumetric
relaxation phase of the cardiac cycle.
During this brief period (approximately 0.06s), the ventricles are closed and myocardial relaxation, which began
during protodiastole, continues.
Intraventricular pressure falls rapidly, although ventricular volume changes little.
When intraventricular pressure falls below atrial pressure, the mitral and tricuspic valves open and rapid filling
of the ventricles begins.

Correct Answer. d

(205). Which of the following is known as the inducible buffer in the kidney?

a. Ammonia

b. Bicarbonate

c. Phosphate

d. Proteins

Solution. Ans-31: (a) Ammonia


Ref: Read the text below
Sol :
In chronic acidosis, the amount of ammonia excreted at any given urine pH also increases.
The effect of this adaptation of ammonia secretion is a further removal of H+ from the tubular fluid and
consequently a further enhancement of H+ secretion.

Correct Answer. a

(206). Massage and application of balms to painful areas in the body relieves pain due to

a. Stimulation of endogenous analgesic system

b. Release of endorphins by the first order neurons in the brain stem

c. Release of glutamate and substance P in the spinal cord

d. Inhibition by large myelinated afferent fibers

Solution. Ans-32: (d) Inhibition by large myelinated afferent fibers


Ref: Read the text below
Sol :
Touch is carried by A£] sensory fibers and pain is carried by A£_ and C type of sensory fibers.
Activation of large myelinated A£] fibers causes pre synaptic inhibition of the pain pathway (Gate control
theory of Melzac and Wall).

Correct Answer. d

Copyright © 2014 Delhi Academy of Medical Sciences, All Rights Reserved. 79/118
(207). Excessive formation of a substance/ secretion in the body is controlled in order to maintain homeostasis by

a. Positive feedback mechanism

b. Negative feedback mechanism

c. Osmosis

d. Haemodynamics

Solution. Ans-33: (b) Negative feedback mechanism


Ref: Read the text below
Sol :
Negative feedback mechanisms are generally stabilizing

Correct Answer. b

(208). Depolarization of ventricles begins from –

a. Posterolateral area of left ventricle

b. Base of left ventricle

c. Left part of septum

d. Right part of septum

Solution. Ans-34: (c) Left part of septum


Ref: Ganong 23rd edition pg 492
Sol :
Spread of depolarization in the ventricles:-
First to be depolarized is the left upper part of the inter- ventricular septum; septal depolarization is from left to
rightQ; the wave of depolarization spreads down the septum to the apex, proceeding from endocardium to
epicardiumQ.
Last parts of the ventricles to be depolarized are:-
Posterobasal portion of the left ventricle
Pulmonary conus
A small uppermost portion of the septum

Correct Answer. c

(209). All of the following are renal vasodilators except

a. Dopamine

b. Ach

c. High protein diet

d. Prostaglandins

Solution. Ans-35: (d) Prostaglandins


Ref: Read the text below
Sol :
PGs increase flow in the cortex and decrease blood flow in renal medulla
• Renal vasodilators
– Dopamine
– Ach
– High protein diet (increases renal blood flow and increases glomerular capillary pressure)
• Vasoconstrictors
– NE (causes constriction of afferent arterioles)
– Angiotensin II (causes constriction of both afferent and efferent arterioles)
• PGs increase flow in the cortex and decrease blood flow in renal medulla

Correct Answer. d

Copyright © 2014 Delhi Academy of Medical Sciences, All Rights Reserved. 80/118
(210). ANP acts on

a. PCT

b. DCT

c. CD

d. Loop of henle

Solution. Ans-36: (c) CD


Ref: Read the text below
Sol :
Actions of ANP:
(i) Dilates afferent arterioles
(ii) Relaxes mesangial cells
(iii) Decreases Na reabsorption from CD
(iv) Increases capillary permeability
(v) Relaxes vascular smooth muscle

Correct Answer. c

(211). Transpulmonary pressure is the difference between

a. The bronchus and atmospheric pressure

b. Pressure in alveoli and intrapleural pressure

c. Atmosphere and intrapleural pressure

d. Atmosphere and intraalveolar pressure

Solution. Ans:37: (b) Pressure in alveoli and intrapleural pressure


Ref: Read the text below
Sol :
Transpulmonary or distending pressure is the difference between intrapulmonary (also k/a intraalveolar or
airway) pressure and intrapleural pressure

Correct Answer. b

(212). At 20 degree celsius atmospheric temperature, body reacts by

a. Cutaneous vasoconstriction

b. Shivering

c. Increase BMR

d. None of the above

Solution. Ans-38: (a) Cutaneous vasoconstriction


Ref: Read the text below
Sol :
The first response on exposure to cold is always vasoconstriction

Correct Answer. a

Copyright © 2014 Delhi Academy of Medical Sciences, All Rights Reserved. 81/118
(213). False regarding EPSP is

a. They obey all or none phenomenon

b. They are not self propagating in nature

c. Directly proportional to strength of stimulus

d. Generated at post synaptic region

Solution. Ans-39: (a) They obey all or none phenomenon


Ref: Read the text below
Sol :
EPSP is a local potential, which is a graded response...i.e. proportional to strength of stimulus

Correct Answer. a

(214). CO poisoning causes which type of hypoxia

a. Hypoxic

b. Anemic

c. Stagnant

d. Histotoxic

Solution. Ans-40: (b) Anemic


Ref: Read the text below
Sol :
CO poisoning has a very strong affinity for Hb which causes a decrease in its O2 saturation, thereby causing a
type of anemic hypoxia.
In anemic hypoxia there is no effect on the dissolved O2 in the blood- therefore it has no effect on the
peripheral chemoreceptors i.e., the carotid and aortic bodies

Correct Answer. b

(215). The substance that contributes maximally to the osmolality inside the cell is

a. Protein

b. Phosphate

c. Urea

d. Potassium

Solution. Ans-41: (d) Potassium


Ref: Read the text below
Sol :
Most important cation and anion in the ECF are sodium and chloride
Most important cation and anion in the ICF are potassium and phosphates

Correct Answer. d

Copyright © 2014 Delhi Academy of Medical Sciences, All Rights Reserved. 82/118
(216). The Equilibrium potential for K+ is

a. -90 mV

b. -95 mV

c. +65 mV

d. +60 mV

Solution. Ans-42: (b) -95 mV


Ref: Read the text below
Sol :
Ion Conc in ICF (mmol/L) Conc in ECF (mmol/L) Equilibrium potential (mV)

+
Na 12 145 +65

+
K 155 4 -95

+ -5 -5
H 13 X 10 3.8 X 10 -32

-
Cl 3.8 120 -90

-
HCO3 8 27 -32

….
Organic anions 155 0 (because cell membrane is
impermeable to organic anions)

Correct Answer. b

(217). Blood brain barrier formed by :

a. Astrocyte

b. Schwann cell

c. Microglia

d. Endothelium of cerebral capillaries

Solution. Ans-43: (d) Endothelium of cerebral capillaries


Ref.: Read the text below
Sol :
The site of the blood brain barrier is the endothelium of cerebral capillaries and the choroid plexus epithelium.
Some physiologists use the term blood – brain barrier to refer to the barrier in the capillary walls and the term
blood CSF barrier to refer to the barrier in the choroid epithelium.
However, the barriers are similar, and it seems more appropriate, to use the term blood-brain barrier to refer to
net exchange across both barriers.

Correct Answer. d

(218). Volume of air inspired during normal inspiration is :

a. 500 ml

b. 1200 ml

c. 2200 ml

d. 2700 ml

Solution. Ans-44: (a) 500 ml


Ref.: Read the text below
Sol :
The tidal volume is the volume of air inspired or expired with each normal breath; it amounts to about 500
milliliters in the average young adult man.

Correct Answer. a

Copyright © 2014 Delhi Academy of Medical Sciences, All Rights Reserved. 83/118
(219). An increase in plasma PTH level would lead to an increase in which of the following?

a. Renal synthesis of calcitriol

b. Renal proximal tubular reabsorption of Ca+2

c. The number of active osteoblasts

d. Plasma inorganic phosphate concentration

Solution. Ans-45: (a) Renal synthesis of calcitriol


Ref.: Read the text below
Sol :
The major renal effect of parathyroid hormone (PTH) is stimulation of proximal tubular 1 α–hydroxylase, an enzyme that converts
calcidiol to calcitriol.
PTH affects renal Ca+2 reabsorption in two ways: it reduces Ca+2 reabsorption in the proximal tubule and increases Ca+2 reabsorption in
the distal tubule.
The net effect is an increase in tubular Ca+2 reabsorption.
This renal effect of PTH can be lessened by PTH’s effect on bone.
PTH increases plasma [Ca+2] by promoting bone resorption, which causes an increase in the renal filtered load of Ca+2 and, thus,
hypercalciuria.
Also, PTH inhibits the proximal reabsorption of phosphate, thereby favoring phosphate excretion.
PTH acts on bone by dissolving the non readily exchangeable calcium phosphate “pool” known as stable bone.
PTH activates the osteoclast cells that cause osteolysis by their high content of lysosomal enzymes. PTH also
stimulates osteocytes, which are bone – bound osteoblasts that mediate osteocytic osteolysis.

Correct Answer. a

(220). Ataxia with tremor present during volitional movements is likely associated with a lesion of the :

a. Motor cortex

b. Basal ganglion

c. Cerebellum

d. Pyramidal tract

Solution. Ans-46: (c) Cerebellum


Ref.: Read the text below
Sol :
Ataxia is loss of muscle coordination : Ataxia and intention tremor suggest cerebellar pathology.
Motor cortex and pyramidal tract damage result in spasticity, while basal ganglia damage causes resting tremor

Correct Answer. c

(221). Gastric secretion is :

a. Inhibited by curare

b. Stimulated by norepinephrine

c. Increased by stomach distension

d. Stimulated by an increase in phonic activity

Solution. Ans-47: (c) Increased by stomach distension


Ref.: Read the text below
Sol :
Gastric secretion is stimulated by stomach distension, vagal activity and acetylcholine.

Correct Answer. c

Copyright © 2014 Delhi Academy of Medical Sciences, All Rights Reserved. 84/118
(222). The phases of the ventricular muscle action potential are represented by the lettered points on the diagram.At which point on the above
ventricular action potential is membrane potential most dependent on calcium permeability?

a. Point A

b. Point B

c. Point C

d. Point D

Solution. Ans-48: (d) Point D


Ref:Read the text below
Sol :
The plateau phase (phase 2) is the result of the influx of calcium.
Although calcium channels begin to open during the upstroke (phase 0), the greatest number of calcium
channels is open during the plateau. The upstroke is primarily dependent on the opening of Na channels.
The initial repolarization (phase 1) is dependent on the inactivation of Na+channels and the opening of a
transient K+channel.
Repolarization (phase 3) is produced by the inactivation of Ca2+channels and the activation of the delayed
rectifier K+channels.

Correct Answer. d

(223). Confounding can be removed by?

a. Assign confounders to both cases and con trolls

b. Stratification

c. Matching

d. All of the above

Solution. Ans-137: (d) All of the above


Ref.: Read the text below
Sol :
- There are various ways to modify a study design to actively exclude or control confounding variables.
- In case – control studies assign confounders to both groups, cases and controls, equally.
- In cohort studies matching is often done by only admitting certain age groups or a certain sex into the study
population and thus all cohorts are comparable in regard to the possible confounding variable.
Stratification :
- Controlling for confounding by measuring the known confounders and including them as covariates in
multivariate analyses.

Correct Answer. d

Copyright © 2014 Delhi Academy of Medical Sciences, All Rights Reserved. 85/118
(224). Millennium development goals aim to reduce MMR by?

a. 3/4

b. 2/3

c. 1/4

d. 1/2

Solution. Ans-138: (a) 3/4


Ref.: Read the text below
Sol :
- Goal 5-Target 6: Reduce by three quarters, between 1990 and 2015, the maternal mortality ratio.

Correct Answer. a

(225). Screening of cervical cancer at PHC is by

a. History

b. Colposcopy

c. PAP smear

d. VIA

Solution. Ans-139: (d) VIA


Ref: Read the text below
Sol:
- VIA is the method used at PHC.
- PAP & Colposcopy is used at district hospital.

Correct Answer. d

(226). The National Health Programmes of India collects data for morbidity and mortality data mainly by which type of surveillance?

a. Active surveillance

b. Passive surveillance

c. Sentinel Surveillance

d. None of these

Solution. Ans-140: (b) Passive surveillance


Ref.: Read the text below
Sol :
- Passive surveillance : Data is itself reported to the health system Eg: patient presenting with fever to a PHC
- (The National Health Programmes in India mostly (not all) rely on passive surveillance for morbidity and
mortality data collection)
- Active surveillance : There is active seeking of data by the health system Eg: Health worker going fortnightly to
detect fever cases and collect blood slides under Malaria Component of National Vector Borne Disease Control
Programme (NVBDCP)
- Sentinel surveillance : Sentinel data is extrapolated to the entire population to estimate disease prevalence in
the total population. Sentinel Surveillance is done in National AIDS Control Programme.

Correct Answer. b

Copyright © 2014 Delhi Academy of Medical Sciences, All Rights Reserved. 86/118
(227). COPRA recognizes how many rights to the consumer.

a. 3

b. 4

c. 5

d. 6

Solution. Ans:141: (d) 6


Ref.: Read the text below
Sol :
Consumer Protection Act (Copra)
- Enacted in 1986
- Provided a for speedy redressal of customer grievances against medical services.
- Decision should be taken within 3 to 6 months.
- No court fee payment
- It recognizes six rights of the consumer: right to safety, right to be informed, right to choose, right to be heard,
right to seek redressal, right to consumer education.
- Monetary limits of compensation grantable by consumer courts :
- District consumer court: UptoRs. 20 Lakhs
- State commission: Rs. 20 lakhs to Rs. 1 crore
- National commission: Above Rs. 1 crore

Correct Answer. d

(228). WHO classifies mental retardation into four categories : mild, moderate, severe and profound. IQ range in moderate MR is:

a. 35-49

b. 20-34

c. 50-69

d. 70-90

Solution. Ans-142: (a) 35-49


Ref.: Read the text below
Sol :
Classification of mental retardation (ICD-10)
Mental status IQ Range
Normal IQ 70 & above
Mild MR 50-69
Moderate MR 35-49
Severe MR 20-34
Profound MR Under 20

Correct Answer. a

Copyright © 2014 Delhi Academy of Medical Sciences, All Rights Reserved. 87/118
(229). This is the logo of

a. RNTCP

b. NLEP

c. MNREGA

d. DOTS

Solution. Ans-143: (b) NLEP


Ref: Read the text below
Sol:
The NLEP Emblem symbolizes beauty and purity in lotus: Leprosy can be cured and a leprosy patient can be a
useful member of the society in the form of a partially affected thumb; a normal fore-finger and the shape of
house; the symbol of hope and optimism in a rising sun. The Emblem captures the spirit of hope positive action in
the eradication of Leprosy.

Correct Answer. b

(230). Graph to compare two quantitative data is?

a. Histogram

b. Scatter diagram

c. Line Diagram

d. Frequency curve

Solution. Ans-144: (b) Scatter diagram


Ref.: Read the text below
Sol :
- A scatter plot, scatterplot, or scattergraph is a type of mathematical diagramusing Cartesian coordinates to
display values for two variables for a set of data.
- The data is displayed as a collection of points, each having the value of one variable determining the position on
the horizontal axis and the value of the other variable determining the position on the vertical axis.
- This kind of plot is also called a scatter chart, scattergram, scatter diagram,or scatter graph.

Correct Answer. b

(231). The measure used to express the global burden of disease, i.e. how a healthy life is affected by disease is–

a. Disability – Adjusted life year

b. Case fatality rate

c. Life expectancy

d. Age – specific incidence rate

Solution. Ans-145: (a) Disability – Adjusted life year


Ref:Park-24
Sol:
DALY (Disability - Adjusted Life Year) :
- DALY is a measure of the burden of disease in a defined population and the effectiveness of the interventions.
- DALYs express years of life lost to premature death and years lived with disability adjusted for the severity of
the disability.
- One DALY is “one lost year of healthy life”.

Correct Answer. a

Copyright © 2014 Delhi Academy of Medical Sciences, All Rights Reserved. 88/118
(232). What is Berkesonian bias –

a. Mis-classification bias

b. Recall bias

c. Selection bias

d. Non-response bias

Solution. Ans-146: (c) Selection bias


Ref:Read the text below
Sol:
In statistics, the term bias refers to several different concepts:
- Selection bias, where individuals or groups are more likely to take part in a research project than others,
resulting in biased samples. This can also be termed Berksonian bias[.
o Spectrum bias arises from evaluating diagnostic tests on biased patient samples, leading to an overestimate of
the sensitivity and specificity of the test.
- The bias of an estimator is the difference between an estimator's expectation and the true value of the
parameter being estimated.
o Omitted-variable bias is the bias that appears in estimates of parameters in a regression analysis when the
assumed specification is incorrect, in that it omits an independent variable that should be in the model.

Correct Answer. c

(233). After administrating live vaccine, immunoglobulins are give after –

a. 1 week

b. 2 weeks

c. 10 weeks

d. 12 weeks

Solution. Ans-147: (d) 12 weeks


Ref:Park19-99
Sol:
- Normal human Ig is used to prevent measles in highly susceptible individuals and to provide temporary
protection (upto 12 weeks) against hepatitis A infection for travelers to endemic areas and to control institutional
& household outbreaks of hepatitis A infection.
- Live vaccines should not normally be given for 12 weeks after an injection of normal human Ig, and if a live
vaccine has already been given.
- NHIg injection should be deferred for 2 weeks.

Correct Answer. d

(234). All of the following are contraindication of DPT except –

a. Serious reaction to a previous injection of DPT

b. Parent or sibling with seizures

c. Brain problem that is getting worse

d. Inconsolable crying/screaming within 7 days of DPT immunization in the past

Solution. Ans-148: (d) Inconsolable crying/screaming within 7 days of DPT immunization in the past
Ref: Read the text below
Sol:
The following events or conditions may have an impact on the risk of a reaction with the DPT vaccine:
- Serious reaction to a previous injection of DPT (or any of the components of the vaccine)
- Moderate or severe illness
- History of seizure(s)
- Parent or sibling with seizures
- Brain problem that is getting worse
- Alterations of consciousness within 7 days of previous DPT vaccine
- Inconsolable crying/screaming or collapse/shock-like state within 48 hours of DPT immunization in the past
- High fever

Correct Answer. d

Copyright © 2014 Delhi Academy of Medical Sciences, All Rights Reserved. 89/118
(235). Acculturation is?

a. Triage

b. Cultural changes due to socialization

c. Attitude

d. Belief

Solution. Ans-149: (b) Cultural changes due to socialization


Ref.: Read the text below
Sol :
- Acculturation explains the process of cultural and psychological change that results following meeting between
cultures.
- The effects of acculturation can be seen at multiple levels in both interacting cultures.

Correct Answer. b

(236). Which one of the following pairs for water quality standards is correctly matched?

a. a

b. b

c. c

d. d

Solution. Ans-150: (b) Nitrates 50 mg/litre


Ref.: Park - 579
Sol :
Standards and maximum range of inorganic constituents :
Chloride 200 mg/litre (slandered), 600 mg/litre (mximum)
Hardness 500 mg/liter (maximum)
pH 6.5 to 8.5 (slandered)
Total dissolved solid 600 mg/litre (slandered), 1000 mg/litre (maximum),
Manganese 0.1/Litre (slandered), 0.5 mg/litre (maximum),
Nitrate (as No3) 50
Nitrite (as No2) 3

Correct Answer. b

Copyright © 2014 Delhi Academy of Medical Sciences, All Rights Reserved. 90/118
(237). The concept of “social physician” was brought about by:

a. Srivastava committee

b. Kartar singh

c. Bhore committee

d. Bajaj committee

Solution. Ans-151: (c) Bhore committee


Ref.: Read the text below
Sol :
The concept of “Social Physician” was brought about by Bhore committee, 1946 Bhore Committee, 1946.
- Also known as Health Survey & Development Committee
Recommendations
- Integration of preventive & curative services
- Short term measure : 1 PHC per 40,000 population
- Long term program me (also known as 3 million plan); Primary health units with 75 bedded hospitals per 10,000-
20,000 population; secondary health units with 650 bedded hospitals; regional health units (surrounding district
hospitals) with 2500 beds
- Social physician : 3 months training in SPM in medical schools

Correct Answer. c

(238). Janani Suraksha Yojana (JSY) is a safe motherhood intervention under the National Rural Health Mission (NRHM) implemented with the
objective of reducing maternal and neonatal mortality by promoting institutional delivery among poor pregnant women. The scheme was
launched in which year?

a. 2001

b. 2005

c. 2006

d. 2008

Solution. Ans-152: (b) 2005


Ref.: Read the text below
Sol :
Janani Suraksha Yojana
- Launched on 12th April 2005.
- A modification of national Maternity Benefit scheme
Objectives
- Reduction of maternal & infant mortality
- Institutional care
- Features
- 100% centrally sponsored.
- Integrates benefit of cash assistance with institutional care.

Correct Answer. b

Copyright © 2014 Delhi Academy of Medical Sciences, All Rights Reserved. 91/118
(239). Measles vaccine, after reconstitution, is to be used within:

a. Four hours

b. One hour

c. Two hours

d. Three hours

Solution. Ans-153: (b) One hour


Ref.: Read the text below
Sol :
Measles vaccine after reconstitution is to be used within one hour.
- Measles vaccine:
- Live attenuated; freeze dried.
- Strain: Edmonston Zagreb strain, Moraten strain, Schwartz strain
- Administration : Single s/c dose of 0.5 ml of reconstituted vaccine
- Diluents for reconstitution: Distilled water or sterile water.
- Age of administration as per National Immunization Schedule : 9 months (6-9 months in epidemics &
malnutrition)
- Cold chain temperature for storage : 2 to 8° C
- Incubation period of vaccine induced measles : 7 days.
Park Textbook mentions one hour but Mohfw Immunization handbook states four hours Mohfw ref. is more
authentic.

Correct Answer. b

(240). In 1980, ICIDH (International Classification of Impairment, Disability and Handicap) was published by the WHO as a manual of
classification relating to the consequences of disease. In an accident, a person lost his leg. According to ICIDH this is an example for:

a. Disease

b. Impairment

c. Disability

d. Handicap

Solution. Ans-154: (b) Impairment


Ref.: Read the text below
Sol :
Disease – impairment – disability – handicap
- Disease : any abnormal condition of an organism that impairs function
Eg. Accident
- Impairment : any loss or abnormality of psychological, physiological or anatomical structure or function.
Eg. Loss of foot.
- Disability : any restriction or inability to perform an activity because of the impairment.
Eg. Cannot walk.
- Handicap: a disadvantage resulting form an impairment/disability.
Eg. Unemployment (above person can’t walk – can’t work – unemployed).

Correct Answer. b

Copyright © 2014 Delhi Academy of Medical Sciences, All Rights Reserved. 92/118
(241). What is the actual intention of doing sentinel surveillance ?

a. To know the total number of cases

b. For health planning

c. To know the natural history of the disease

d. To prevent the disease

Solution. Ans-155: (a) To know the total number of cases


Ref: Read the text below
Sol:
Sentinel Surveillance :
- A method for identifying the missing cases and thereby supplementing the notified cases.
- The sentinel data is extrapolated to the entire population to estimate the disease prevalence in the total
population.
- Advantage is the reporting biases are minimized and feedback information to the providers are simplified.

Correct Answer. a

(242). The dose of OPV, given at 14 weeks is –

a. Zero dose OPV

b. OPV-1

c. OPV-2

d. OPV-3

Solution. Ans-156: (d) OPV-3


Ref:Park19-105
Sol:
For infants
At birth - BCG and OPV-0 dose
(for institutional deliveries)
At 6 weeks - BCG (if not given at birth)
- DPT-1, OPV-1 and Hepatitis B-1
At 10 weeks - DPT-2, OPV-2 and Hepatitis B-2
At 14 weeks - DPT-3, OPV-3 and Hepatitis B-3
At 9 months - Measles

Correct Answer. d

(243). Measles vaccine is kept in refrigerator in –

a. Chilled tray

b. Freezer

c. Tray below the freezer

d. Shelves in the door

Solution. Ans-157: (c) Tray below the freezer


Ref:Park19-99
Sol:
- Among the vaccines, polio is the most sensitive to heat, requiring storage at minus 20 degree C.
- Vaccines which must be stored in the freezer compartment are: polio and measles.
- Vaccines which must be stored in the cold part but never allowed to freeze are : typhoid, DPT, tetanus toxoid,
DT, BCG and diluents.

Correct Answer. c

Copyright © 2014 Delhi Academy of Medical Sciences, All Rights Reserved. 93/118
(244). Carrier stage seen in –

a. Pertusis

b. Plague

c. Tetanus

d. None.

Solution. Ans-158: (a) Pertusis


Ref:Park19-90
Sol:
HEALTHY CARRIERS
- Healthy carriers emerge from subclinical cases.They are victims of subclinical infection who have developed
carrier state without suffering from overt disease, but are nevertheless shedding the disease agent, e.g.
poliomyelitis, cholera, meningococcal meningitis, salmonellosis, and diphtheria.
- It is well to remember that a person whose infection remains subclinical may or may not be a carrier.
- For example, in polio the infection may remain subclinical and the person may act as a temporary carrier by
virtue of shedding the organism.

Correct Answer. a

(245). Nalgonda technique for defluoridation is in what sequence?

a. Lime+Alum

b. Soda+Alum

c. Alum+ Soda

d. Alum + Lime

Solution. Ans-159: (d) Alum + Lime


Ref.: Read the text below
Sol :
- Nalgonda Technique involves addition of aluminium salts, lime and bleaching powder followed by rapid mixing,
flocculation, sedimentation, filtration and disinfection. Aluminium salt may be added as aluminium sulphate or
aluminium chloride or combination of these two.
- Aluminium salt is only responsible for removal of fluoride from water. The dose of aluminium salt increases
with increase in the fluoride and alkalinity levels of the raw water.
- The selection of either aluminium sulphate or aluminium chloride also depends on sulphate and chloride
contents of the raw water to avoid them exceeding their permissible limits.
- The dose of lime is empirically 1/20th that of the dose of aluminium salt. Lime facilitates forming dense floc for
rapid settling.
- Bleaching powder is added to the raw water at the rate of 3 mg/l for disinfection.

Correct Answer. d

Copyright © 2014 Delhi Academy of Medical Sciences, All Rights Reserved. 94/118
(246). Triage has how many colors?

a. 2

b. 3

c. 4

d. 5

Solution. Ans:160: (c) 4


Ref.: Read the text below
Sol :
Triage :
- When quantity and severity of injuries overwhelm the operative capacity of health facilities, a different
approach to medical treatment must be adopted.
- The principle of first come first treated is not followed in mass emergencies.
- Triage consists of rapidly classifying the injured on the basis of severity of their injuries and likelihood of their
survival with prompt medical intervention.
- Triage is the only approach that can provide maximum benefit to the greatest number of injured in a major
disaster situation
- The most common classification uses the internationally accepted 4 colour code system.
Red High priority or transfer
Yellow Medium priority
Green Ambulatory patients
Black Dead or moribund

Correct Answer. c

(247). A 71 years old man with carotid artery stenosis develops severe depression and hypersomina.The best choice for initial treatment would
be ?

a. Amitriptyline

b. Doxepine

c. Nortriptyline

d. Phenelzine

Solution. Ans 276: - (c) Nortriptyline.


Ref- Read the text below
Sol.
- Elderly patients are often sensitive to the hypotensive,sedative and anticholinergic effects of
antidepressants,this patient is at special risk for hypotension because of his carotid artery stenosis,nortryptiline is
least likely to cause these side effects.

Correct Answer. c

Copyright © 2014 Delhi Academy of Medical Sciences, All Rights Reserved. 95/118
(248). Methods of treating autistic disorder include all of the following except

a. A highly structured classroom setting

b. Behavior modification

c. Imipramine administration

d. Psychotherapy

Solution. Ans-277: (c) Imipramine administration


Ref: Read the text below
Sol:
- Administration of imipramine is not effective for treating autistic disorder. Behavior modification is the most
suitable treatment for autistic disorder.
- It aims to promote social behavior and relatedness, to maximize language skills, and to eliminate idiosyncratic
behavior.
- A structured educational setting with a low student-to-teacher ratio and a constant environment are helpful.
- Psychotherapy may be helpful when the family is having trouble coping with an autistic child or when
environmental factors have complicated the illness.
- Psychotherapy for the child may be indicated in those children with sufficient language skill and intellect.

Correct Answer. c

(249). A 35-year-old man with an obsessive-compulsive personality disorder is likely to exhibit all of the following symptoms except

a. Perfectionism that interferes with performance

b. Compulsive checking behavior

c. Preocupation and concern for rules

d. Indecisiveness

Solution. Ans-278: (b) Compulsive checking behavior


Ref: Read the text below
Sol:
- It is important to remember that the obsessive-compulsive disorder (i.e., the anxiety disorder) is a different
illness form the obsessive-compulsive personality disorder.
- Individuals with the obsessive – compulsive personality disorder seen to have perfectionistic and inflexible
personalities.
- These individuals are often indecisive, stingy with compliments and preoccupied by trivial details, but they do
not exhibit symptoms of the anxiety disorder, such as compulsive checking and excessive washing.

Correct Answer. b

(250). A 34-year-old man recurrently perceives the smell of rotten eggs.This kind of hallucination is relatively rare and is most commonly
encountered in patients with

a. Parietal tumors

b. Narcolepsy

c. Grand mal epilepsy

d. Partial complex seizures

Solution. Ans 279: (d) Partial complex seizures


Ref– Read the text below
Sol:
- Hallucinations involving smell, taste, or kinesthetic experiences (body movements) are rare.
- They are most commonly encountered in patients with partial complex seizures, although occasionally they are
reported by patients with somatization disorder, psychosis, and hypochondriasis.
- Tumors involving the olfactory areas of the brain must also be considered in the differential diagnosis.

Correct Answer. d

Copyright © 2014 Delhi Academy of Medical Sciences, All Rights Reserved. 96/118
(251). Seasonal circadian rhythm has been implicated in the etiology of

a. Depression

b. Sociopathy

c. Capgras syndrome

d. Panic disorder

Solution. Ans 280: (a) Depression


Ref– Read the text below
Sol:
- Seasonal circadian rhythm has been implicated in the etiology of seasonal depressive disorder,which is
characterized by recurrent depression that starts in November and resolves in March.
- Winter’s short days cause a change in the pattern of secretion of nocturnal melatonin, which is inhibited by
light. In animals this causes a decrease in activity level, reproduction-related behaviors, and aggressive behavior.
- The same mechanism is postulated to be effective in seasonal depression, which, in fact, responds to light
therapy.

Correct Answer. a

(252). Anxiety is a common symptom of all of the following illnesses except

a. hypoglycemia

b. Hyperparathyroidism

c. Pheochromocytoma

d. Porphyria

Solution. Ans-281: (b) Hyperparathyroidism


Ref: Read the text below
Sol:
- Hyperparathyroidism is more likely to cause depression than anxiety.
- Anxiety caused by hypoglycemia and pheochromocytoma may be accompanied by signs of increased adrenergic
activity such as sweating and tachycardia.
- Porphyria can produce a variety of psychiatric symptoms, including anxiety.

Correct Answer. b

Copyright © 2014 Delhi Academy of Medical Sciences, All Rights Reserved. 97/118
(253). The cystographic appearance shown below is seen in?

a. Thimble bladder

b. Neurogenic bladder

c. Carcinoma urinary bladder

d. Schistosomiasis of bladder

Solution. Ans-: (b) Neurogenic bladder


Ref:Read the text below
Sol :
· A “pine cone bladder or christmas tree bladder” is a cystogramappearance in which the bladder is elongated and pointed with
thickened trabeculated wall.
· It is typically seen in severe neurogenic bladderwith increased sphincter tone (detrusor sphincter dyssynergia) due to suprasacral
lesions (above S2-S4) or epiconal lesions (in and around S2-S4)

Correct Answer. b

(254). Terry Thomas sign is related with : -

a. Scapholunate dislocation

b. Perilunate dislocation

c. Perilunate dislocation with associated triquetral dislocation (mid-carpal)

d. Lunate dislocation.

Solution. Ans: (1) Scapholunate dislocation


Ref –Read the text below
Sol:
· With scapholunate dislocations, there is an increased (> 3mm) gap between the scaphoid and the lunate on the AP view.
· This is described as the Terry Thomas sign

Correct Answer. a

Copyright © 2014 Delhi Academy of Medical Sciences, All Rights Reserved. 98/118
(255). A 34-year-old amateur spelunker develops cough,dyspnea, and fever 2 weeks after a caving expedition to caves. On physical examination,
the patient’s temperature is 102°F and respiratory rate is 24. On pulmonary examination, there are diffuse crackles bilaterally. A CXR is
shown in Figure 1-1.

Which of the following is the most likely cause of disease in this patient?

a. The patient likely developed influenza from close contact with the other members of the caving expedition.

b. The patient likely has disseminated aspergillosis.

c. The patient likely has miliary tuberculosis.

d. The patient likely has acute pulmonary histoplasmosis.

Solution. Ans-: (d) The patient likely has acute pulmonary histoplasmosis.
Ref:Read the text below
Sol:
· The patient has diffuse interstitial infiltrates on CXR that correspond in time and presentation to acute inhalation histoplasmosis.This
would be seen in a patient, such as an amateur spelunker, who has been in a cave with bats. It is the act of crawling through the cave
that disturbs the spores of histoplasmosis that grow in the bat guano.
· The incubation period for influenza is 1–2 days. It is passed primarily by secretions from the nose spread by hands. The other members
of the expedition were not sick, as they might be with influenza.
· Disseminated aspergillosis occurs in immunocompromised patients who have defects in both cell-mediated and humoral immunity.
This patient does not have this. While the CXR could mimic military tuberculosis, the association with caving 14 days before would make
tuberculosis less likely and histoplasmosis more likely.
· There is no history that the patient is immunocompromised with HIV and would be at risk for P. jiroveci pneumonia.

Correct Answer. d

(256). MRI rooms are shielded completely by a continuous sheet or wire mesh of copper or aluminum to shield the imager from external
electromagnetic radiations, etc. It is called;

a. Maxwell cage

b. Faraday cage

c. Edison’s cage

d. Ohms cage

Solution. Ans : (b) Faraday cage


Reference: Read the text below
Sol:
· MRI roomsare shielded by Faraday cage completely by a continuous sheet or wire mesh of copper or aluminum to shield the imager
from external electromagnetic radiations

Correct Answer. b

Copyright © 2014 Delhi Academy of Medical Sciences, All Rights Reserved. 99/118
(257). ‘ If large bowel is interposed between the liver and the diaphragm, leading to incorrect diagnosis of free intraperitoneal air’ is known as :
-

a. “Blueberry muffin” syndrome

b. Dancing feet syndrome.

c. “Dumping syndrome,”

d. Chilaiditi's syndrome

Solution. Ans : (d) Chilaiditi's syndrome


Ref – Clinical Radiology by Hugue Ouellette - 28
Sol:
1.In some patients, large bowel is interposed between the liver and the diaphragm, which may lead to incorrect diagnosis of free
intraperitoneal air (Chilaiditi's syndrome).

Correct Answer. d

(258). Coffee bean or an inverted U shaped grossly dilated loop of colon on radiograph is a feature of : -

a. Sigmoid volvulus

b. Appendicitis

c. Intussuception

d. Ulcerative colitis

Solution. Ans : (a) Sigmoid volvulus


Reference – Clinical Radiology by Hugue Ouellette - 33
Sol:
Sigmoid volvulus
1.Twisted sigmoid often appears on the radiograph as a dilated loop of large bowel in the lower abdomen, resembling a coffee bean or an
inverted U.
2.The remainder of the colon is usually dilated.

Correct Answer. a

Copyright © 2014 Delhi Academy of Medical Sciences, All Rights Reserved. 100/118
(259). The condition shown below occurs due to?

a. Keloid

b. Mucous gland hypertrophy

c. Sweat gland hypertrophy

d. Sebaceous gland hypertrophy

Solution. Ans-: (d) Sebaceous gland hypertrophy


Ref:Read the text below
Sol :
· Rhinophyma(potato nose) occurs due to sebaceous gland hypertrophy.It is a glandular form of acne rosacea.
· Enlarged hilar and mediastinal nodes in sarcoidosis are called “potato nodes”

Correct Answer. d

(260). A 28 yea old rugby player comes to the OPD with skin lesion shown in image.What is the most probable diagnosis?

a. Tinea corporis

b. Lupus vulgaris

c. Cutaneous leishmaniasis

d. Erythema migrans

Solution. Ans-: (a) Tinea corporis


Ref:Read the text below
Sol :
The history of contact sports(rugby) along with skin lesion showing central clearing is seen in Tineacorporis.
Central scarring is seen in Lupus vulgaris
ØCentral crustingis seen in Cutaneous Leishmaniasis

Correct Answer. a

Copyright © 2014 Delhi Academy of Medical Sciences, All Rights Reserved. 101/118
(261). Pohl-Pincus constriction is the counterpart of

a. Mees line

b. Beau’s line

c. Muehrcke’s line

d. Burton’s line

Solution. Ans-: (b) Beau’s line


Ref.:Read the text below
Sol :
• Beau’s line is the counterpart of Pohl-Pincus constriction.
• Mees’ line is seen in the nail in arsenic poisoning.
• Muehrocke’s lines are parallel white lines seen in the distal nail in hypoalbumunaemia of any cause.

Correct Answer. b

(262). The nail finding shown below is associated with ?

a. Psoriasis

b. Leprosy

c. Dermatitis Herpetiformis

d. Lichen planus

Solution. Ans-: (d) Lichen planus


Ref:Read the text below
Sol :
In 10-20% of patients with Lichen planus,longitudinalridging,grooving and scarring leads to dorsal “pterygium” formation
ØLichen planus may be associated with Hepatitis C infection

Correct Answer. d

Copyright © 2014 Delhi Academy of Medical Sciences, All Rights Reserved. 102/118
(263). A patient with which of the following dermatological conditions can have X-ray findings as shown below?

a. Psoriasis

b. Leprosy

c. Pemphigus

d. Lichen planus

Solution. Ans-: (a) Psoriasis


Ref:Read the text below
Sol :
The X-ray shows the characteristic “pencil in cup deformity” seen in psoriatic arthritis.
Wright and Moll classification is for Psoriatic arthropathy

Correct Answer. a

(264). Vascular inflow occlusion of the liver is by

a. Clamping the hepatic Artery

b. Occluding the portal veins

c. Clamping the hepatic veins

d. Pringle maneuver

Solution. Ans-214: (d) Pringle maneuver


Ref:Read the text below
Sol:
- The Pringle maneuver allows for an almost total occlusion of the liver blood supply. A large hemostat is used to
clamp the lesser omentum ceasing the blood flow through a. hepatica and v. portae and thus helping to control
bleeding from the liver.

Correct Answer. d

(265). Who said these words “To study phenomenon of disease without books is to sail an uncharted sea, while to study without patients is not
to go sea at all”

a. Hamilton Bailey

b. Sir Robert Hutchison

c. Sir William Osler

d. JB Murphy

Solution. Ans-215: (c) Sir William Osler


Ref:Read the text below
Sol:
“To study phenomenon of disease without books is to sail an uncharted sea, while to study without patients is not
to go to sea at all”.
Sir William Osler, 1849-1919 , Professsor of medicine, UK

Correct Answer. c

Copyright © 2014 Delhi Academy of Medical Sciences, All Rights Reserved. 103/118
(266). In Ranson’s criteria to predict severity of acute pancreatitis

a. A serum amylase that is 4 times normal is significant

b. Serum amylase level is not a criteria

c. Rising serum amylase level are important

d. Serum amylase equals urine amylase

Solution. Ans-216: (b) Serum amylase level is not a criteria


Ref:Read the text below
Sol:
- Ranson scoring system is used to ptedict severity of acute pancreatitis.

At admission In the first 48 hours

· Hematocrit drops by more than 10%


· Age > 55 years
· BUN rise > 5 mg/dl
· WBC counts> 16,000/ml
· Arterial PO2 < 60 mmHg
· Blood glucose> 200 mg/dl (10 mmol/1)
· Serum cacluim < 8mg/dI
· Serum LDH > 700 Units/I
· Base deficit > 4 meq/I
· AST >250 units/I
· Fluid sequestration > 6 litres

Correct Answer. b

(267). The operative wound afer surgery for gangrenous, perforated appendicitis is best managed by

a. Primary suture

b. Delayed primary closure

c. Healing by secondary intention

d. Antibiotic only

Solution. Ans-217: (b) Delayed primary closure


Ref:Read the text below
Sol:
- “Delayed wound closure has been traditionally advocated following open apendicectomy for gangrenous or
perforated appendicitis to improve wound infection rates ranging in some reports a high as 32% with immediate
closure.
- A cost utility analysis identified similar wound infection rates, between 4-5 for both primary and delayed
wound closure is the favored option as long as wound infection rates area not excessive”.

Correct Answer. b

Copyright © 2014 Delhi Academy of Medical Sciences, All Rights Reserved. 104/118
(268). The best guide to adequate tissue perfusion in the fluid management of a patient with burns, is to ensure a minimum hourly urine output
of

a. 10-30 ml

b. 30-50 ml

c. 50-70 ml

d. 70-100 ml

Solution. Ans-218: (b) 30-50 ml


Ref:Read the text below
Sol:
- In burns patients, because the hourly urinary output is a generally reliable and readily available index of
resuscitation adequacy, an indwelling catheter should be placed and urinary output measured and recorded each
hour.
- The fluid infusion rate is adjustes to obtain 30 to 50 ml of urine per hour in the adult and 1 mg/kg body weight
per hour in children weighing less than 30 kg.

Correct Answer. b

(269). Poland’s syndrome of the congenital absence of the sterna head of the pestoralis major muscle is associated with

a. Mastagia

b. Amazia

c. Polymazia

d. Gynecomazia

Solution. Ans-219: (b) Amazia


Ref:Read the text below
Sol:
- Congenital absence of the sterna head of pectoralis major associated with absence of breast on one or both
sides (amazia ) is called Poland’s syndrome. It is more common in males.

Correct Answer. b

(270). Who said “SKIN” is the best dressing ?

a. Joseph Lister

b. John Hunter

c. James paget

d. Mc Neill Love

Solution. Ans-220: (a) Joseph Lister


Ref:Read the text below
Sol:
- "Skin is the Best dressing"- Joseph Lister
- Joseph Lister was a Scottish surgeon who picked up the work of Louis Pasteur and used it to
change the success rates of surgery.

Correct Answer. a

Copyright © 2014 Delhi Academy of Medical Sciences, All Rights Reserved. 105/118
(271). 5% dextrose in normal saline is :

a. Hypotonic

b. Isotonic

c. Hypertonic

d. A blood substitute

Solution. Ans-221: (c) Hypertonic


Ref:Read the text below
Sol:

Saline solutions

- 0.49 % sodium chloride Hypotonic

- 0.9% sodium chloride (Normal saline) Isotonic


- 3% - 5% sodium chloride Hypertonic

Dextrose in saline solutions

- Dextrose 5% in 0.9 sodium chloride (Norma saline) Hypertonin

- Dextrose 5% in 0.45% sodium chloride Hypertonin

Multiple electrolytes solutions

- Ringer lactate Isotonic

- Dextrose 5% in Ringer lactate Hypertonin

Correct Answer. c

(272). Acute bacterial sialadenitis is managed by

a. Immediate sialography

b. Fuid intake restriction

c. Cuture of the FNAC from the involved gland

d. Use of board spectrum antibiotics

Solution. Ans-222: (d) Use of board spectrum antibiotics


Ref:Read the text below
Sol:
- Inflammation of the submandibular gland is termed sialdeinitis
- Acute bacterial sialedinitis is more common and occurs secondary to obstruction of Whartons duct.
- Following infection and despite control of acute symptoms with antibiotics , the gland frequently becomes
chronically inflamd and requires formal excision.
- In parotid gland acute bacteria ascending sialadenitis is historically described in dehydrated elderly patients
following majors surgery – hence fluid restriction is not a treatment.
- Treatment is with intravenous antibiotics.

Correct Answer. d

Copyright © 2014 Delhi Academy of Medical Sciences, All Rights Reserved. 106/118
(273). A thrombosed external hemorrhoid (or perianal hematoma ) is

a. Painless, with graduak onset

b. The 5 Day , Painful , self curing lesion

c. Part of internal hemorrhoids

d. A sentinel pile

Solution. Ans-223: (b) The 5 Day , Painful , self curing lesion


Ref:Read the text below
Sol:
- A thrombosed external hemorrhoid (perianal hematoma) appears suddenly and is very painful and on
examination, a tense tender swelling which resembles a semiripe black currant is seen.
- The hematoma is usually situated in lateral region of the anal margin.
- Untreated, it may resolve, suppurate, fibrose and give rise to a cutaneous tag or burst and extrude the cot, or
continue bleeding. In the majority of cases resolution or firbosis occurs.
- Indeed this condition has been called “ a 5-day painful self curing lesion,”(Milligan).

Correct Answer. b

(274). Axillary vein thrombosis is not a complication of

a. Throracic outlet syndrome

b. Simple mastectomy

c. Excessive upper limb exercise

d. Straphangers

Solution. Ans-224: (b) Simple mastectomy


Ref:Read the text below
Sol:
Risk factors for axillary vein thrombosis
- Vigorous and prolonged use of upper limb in sporting/other activity (primary ‘effort-induced’ from)
- Upper limb trauma
- Central venous catheter e.g. internal jugular vein, subclavian vein (probably strongest risk factor )
- Recent hospitalization
- Severe infection
- Neoplasia
- Thrombophilia (possible further increased risk if have thrombophilia and on oral contraceptive pill)
- Anatomical abnormality of thoracic drug use
- Intravenous recreational drug use
- Straphanger us a nickname for a standing subway or bus passenger who grips a hanging strap (nowadays usually
an overhead horizontal bar ) for support. The name is thought to have originated in the later 1800s when elevated
trains had leather straps for the passengers to hold on to.

Correct Answer. b

(275). The ratio of males to female in the incidence of strangulated inguinal hernia in infancy is

a. 9 : 1

b. 1 : 2

c. 2 : 1

d. 1 : 5

Solution. Ans-226: (d) 1 : 5


Ref:Read the text below
Sol:
“The incidence of strangulated inguinal hernia during infancy is 4% (Gross) and the ratio of F : M = 5 : 1’’
- Most frequently the hernia is irreducible but NOT strangulated.
- In most cases of strangulated inguinal occurring in female infants, the content of sac is an ovary, or an ovary
plus its fallopian tube.

Correct Answer. d

Copyright © 2014 Delhi Academy of Medical Sciences, All Rights Reserved. 107/118
(276). Chest drain insertion is safest when performed

a. In the apex of the chest for pneumothorax

b. In the base of the chest for hemothorax

c. In the ‘traingle of safety’ on the anterolateral chest wall

d. With the smallest bore chest tube available

Solution. Ans-227: (c) In the ‘traingle of safety’ on the anterolateral chest wall
Ref:Read the text below
Sol:
- Insertion of chest drain is indicated when there is air or fluid in the pleural cavity.
- The site if insertion is in the triangle of safety which lies
- Anterior to the midaxillary line
- Above the level of the nipple
- Below and llateral to the pectoralis is major muscle, this will ideally find the fifth space
-A large bore tube is used for drainage of blood and fluids whereas a smaller bore tube is used for removal of
air.
- For traumatic hemothorax, the procedure is similar to drainage for pneumothorax, but a wide bore tube (> 28
Fr ) is required and a basal drain is sometimes necessary.

Correct Answer. c

(277). Type III category of basal fracture of skull are :

a. Those that run in the coronal plane from lateral end od one petrous through sella turcica to lateral end of one petrous ridge

b. Run from side to side in the coronal plane but do not pass through the sella turcica

c. Run from front to the contralateral back passing through the sella turcica

d. Run from front to back involving al cranial fossae and sella turcica

Solution. Ans-228: (b) Run from side to side in the coronal plane but do not pass through the sella turcica
Ref:Read the text below
Sol:
- Transverse fractures of the base of the skull completely bisect the base of the skull thereby creating a “hinge”
and so are also called “hinge fractures”
Types of basal skull fractures

Run in the coronal plane extending from the lateral end of one petrous ridge, through the sella turcica to
Type I (MC form)
the contralateral pertrous ridge

Type II Run from front to the contraleteral back passing through the sella turcica

Type III Run from side to side in the coronal plane but do not pass through the sella turcica

Correct Answer. b

Copyright © 2014 Delhi Academy of Medical Sciences, All Rights Reserved. 108/118
(278). The initial therapy of documented DVT in a postoperative case is

a. Subcutaneous heparin therapy

b. IV herparin therapy

c. Antithrombosis therapy with urokinase

d. Aspirin therapy

Solution. Ans-230: (a) Subcutaneous heparin therapy


Ref:Read the text below
Sol:
Standard treatment for DVT
- Low Molecular Weight Herparin (LMWH) , given once/twice daily via SC injection at a dose of 100 U/kg,
followed by a VKA (vitamin K Antagonist).
- LMWH does not require laboratory monitoring and is given in a fixed dose, usually adapted for body weight
(categories).
- VKA (warfrain) can be safety started at the same day and close is titrated according to the international
normalized ratio (INR) with a target of 2.5 (range 2-3).
- LMWH therapy should be continued for at least 5 days and can be discontinued if the INR is > 2 on two
consecutive measurement at least 24 h apart.

Correct Answer. a

(279). Out of the following locations of intracranial hemorrhage surgery is most useful in

a. Putamen hemorrhage

b. Thalamic hemorrhage

c. Cerebellar hemorrhage

d. Pontine hemorrhage

Solution. Ans-231: (c) Cerebellar hemorrhage


Ref:Read the text below
Sol:
- Intraparenchymal hemorrhage is the most common type of intracranial hemorrhage. It accounts for ~10% of all
strokes and is associated with a 50% case fatality rate.
- Treatment
- Evacuation of supratentorial hematomas does not appear to improve outcome. When compared with medical
management.
- For cerebellar hemorrhages, a neurosurgeon should be consulted immediately to assist with the evaluation;
most cerebellar hematomas > 3 cm in diameter will require surgical evacuation.

Correct Answer. c

(280). The most acceptable method of treatment of esophageal perforation during endoscopy, which is associated with a leak into pleural space
is,

a. Observation alone

b. Observation + antibiotics

c. Insertion of left chest tube

d. Drainage and surgical repair of the injury

Solution. Ans-232: (d) Drainage and surgical repair of the injury


Ref:Read the text below
Sol:
Once the diagnosis of esophageal perforation with leak into the pleural fluid is made, immediate thoracotomy
with drainage of the mediastinum and pleural space is indicated.
- A delay of only several hours is associated with a much higher mortality than if treatment is initiated promptly.
- The tear in the esophagus should be repaired and high doses of broad spectrum systemtic antibiotics
administrated.

Correct Answer. d

Copyright © 2014 Delhi Academy of Medical Sciences, All Rights Reserved. 109/118
(281). All the following are characteristics of hypoparathyroidism occurring after accidental removal of parathyroid during thyroid surgery
except

a. Decreased serum phosphate levels

b. Carpopedal serum phosphate levels

c. Seizures

d. Stridor

Solution. Ans-233: (a) Decreased serum phosphate levels


Ref:Read the text below
Sol:
The most common cause of hypoparathytoidism is damage to the parathyroid glands during thyroid surgery.
Although this complication occurs in only 1% of thyroidectomies.
Features of hypoparathyrodism

In children In adults Biochemical

Charateristic triad: · MC tingling in hands and feet, around the


· Carpopedal spasm mouth. Calcium
· Stridor · Less often – painful carpopedal spasm
· Convulsions · Rare: stridor and fits Phosphate

Correct Answer. a

(282). The investigation of choice for gallstones is

a. USG

b. ERCP

c. Oral cholecystography

d. Sulfur colloid scan

Solution. Ans-234: (a) USG


Ref:Read the text below
Sol:
- Ultrasonography is the method of choice to diagnose gallstones and their complications.
- Oral cholecystography and CT scan may also be used.
- MRCP is becoming increasing available and can demonstrate gallstones and their complications.

Correct Answer. a

Copyright © 2014 Delhi Academy of Medical Sciences, All Rights Reserved. 110/118
(283). Palliative treatment for esophageal carcinoma

a. Ivor lewis operation

b. Endoscopic Laser

c. Cryotherapy

d. Subtotal Oesophagectomy

Solution. Ans-236: (b) Endoscopic Laser


Ref.: Read the text below
Sol :
ESOPHAGEAL CARCINOMA MANAGEMENT :
- Radical oesophagectomy is the most important aspect of curative treatment
- For SCC : oesophagectomy (proximal 10 cm and distal 5 cm resection from the macroscopic tumor)
- Photodynamic therapy (PDT) or Chemoradiotherapy for early CA and those unfit or unwilling for surgery
- Post cricoids carcinoma and upper 3rd growth – radiotherapy
- Middle third growth – lvor lewis operation
- Lower third growth – subtotal oesophagectomy with proximal gastrectomy
Palliative treatment :
- Endoscopic laser treatment
- Brachytherapy
- SEMS – Self expanding metallic stents
- Transhiatal Oesophagectomy

Correct Answer. b

(284). Proctoscope can be used to inspect lower rectum up to

a. 5 cm

b. 10 cm

c. 12 cm

d. 15 cm

Solution. Ans-237: (b) 10 cm


Ref.: Read the text below
Sol :
- Proctoscopy – to inspect the anus, anorectal junction and lower rectum (upto 10 cm)
- Sigmoidoscopy rigid – 18 cm
- Flexible sigmoidoscopy – 60 cm
- Colonoscopy – 160 cm

Correct Answer. b

Copyright © 2014 Delhi Academy of Medical Sciences, All Rights Reserved. 111/118
(285). Red currant jelly stool is diagnostic of

a. Ulcerative colitis

b. Intussusception

c. Rectal carcinoma

d. Intestinal TB

Solution. Ans-238: (b) Intussusception


Ref.: Read the text below
Sol :
- An intussusception is a medical condition in which a part of the intestine hasinvaginated into another section of
intestine, similar to the way in which the parts of a collapsible telescope slide into one another
- Early symptoms can include nausea, vomiting (sometimes bile stained [green color]), pulling legs to the chest
area, and intermittent moderate to severe cramping abdominal pain.
- Pain is intermittent not because the intussusception temporarily resolves, but because the intussuscepted
bowel segment transiently stops contracting. Later signs include rectal bleeding, often with "red currant jelly" stool
(stool mixed with blood and mucus), and lethargy.
- Physical examination may reveal a "sausage-shaped" mass felt upon palpation of the abdomen.
- Fever is not a symptom of intussusception.
- However, intussusception can cause a loop of bowel to become necrotic, secondary to ischemia due to
compression to arterial blood supply. This leads to perforation and sepsis, which causes fever.

Correct Answer. b

(286). A 50-year-old man presents to the emergency room with a 6-h history of excruciating abdominal pain and distention. The abdominal film
shown below is obtained. The next diagnostic maneuver should be

a. Emergency celiotomy

b. CT scan of the abdomen

c. Barium enema

d. Sigmoidoscopy

Solution. Ans-235: (d) Sigmoidoscopy


Ref: Read the text below
Sol :
- The film shows a markedly distended colon. The differential diagnosis includes tumor, foreign body, and colitis,
but far more likely is either cecal or sigmoid volvulus.
- Sigmoid volvulus may be ruled out quickly by proctosigmoidoscopy, which is preferable to barium enema, since
sigmoid volvulus may be treated successfully by rectal tube decompression via the sigmoidoscope.
- If sigmoidoscopy is negative, the working diagnosis, based on this classic film, must be cecal volvulus; barium
enema would clinch the diagnosis, but the colon might rupture in the intervening 1–2 h. Emergency celiotomy
should be done.

Correct Answer. d

Copyright © 2014 Delhi Academy of Medical Sciences, All Rights Reserved. 112/118
(287). Stomach washis absolutely contraindicated in :

a. Corrosive poisoning

b. Convulsant poisoning

c. Sigmoidoscopy

d. Hypothermia

Solution. Ans-: (a) Corrosive poisoning


Ref.:Read the text below
Sol :
Gastric lavageis useful within three hours after ingestion of a poison.
Contraindications:
(i) The only absolute contraindication is corrosive poisoning (except carbolic acid)
due to danger of perforation,
(ii) Stomach wash is done with proper precautions in
(a) Convulsant poisons, as it may lead to convulsions.
(b) Comatose patients, because of the risk of aspiration of fluid into the air-passages.
(c) Volatile poison, which may be inhaled.
(d) Upper alimentary disease e.g. oesophageal varices
(e) In patient with marked hypothermia.

Correct Answer. a

(288). Postmortem nasal swab in death is taken following :

a. Cocaine poisoning

b. Hanging

c. Drowning

d. Strychnine poisoning

Solution. Ans-: (a) Cocaine poisoning


Ref.:Read the text below
Sol :
Hair and nasal swabs
· Head hair can be cut near the root, sectioned to reflect the time-line of hair growth, and tested to determine the use of some drugs
and the apparent time-course of that drug use.
· A nasal swab can be used to test for cocaine or other drugs taken by nasal insufflation.

Correct Answer. a

(289). Dribbling of salivais seen in a case of

a. Hanging

b. Strangulation

c. Drowning

d. Suffocation

Solution. Ans-108: (a) Hanging


Ref.:Narayan Reddy’s-160.
Sol :
§ In hanging, saliva may be found dribbling from the angle of the mouth when the head is drooping forward. This is due to increased
salivation just before death due to the stimulation of the salivary glands by the ligature.
§ Slight hemorrhage or bloody froth is sometimes seen at the mouth or nostrils.
Other signs of hanging are
- Ligature mark around the neck.
- Presence of ecchymoses, abrasions and redness around the ligature mark
- Ecchymoses of the larynx or epiglottis.
- Rupture of the intima of the carotid
- Postmortem signs of asphyxia.

Correct Answer. a

Copyright © 2014 Delhi Academy of Medical Sciences, All Rights Reserved. 113/118
(290). Legally “Abortion” is a termination of pregnancy

a. Before 6 weeks

b. Before 16 weeks

c. Before 26 weeks

d. Before full term

Solution. Ans-109: (d) Before full term


Ref.:Narayan Reddy’s-200.
Sol :
- Legally, abortion (miscarriage) is the premature expulsion of the fetus from the mother’s womb at any time of pregnancy before the
term of pregnancy is completed.

Correct Answer. d

(291). Dilatation of pupils is seen in poisoning by

a. Dhatura

b. Opium

c. Arsenic

d. Sulfuric acid

Solution. Ans-: (a) Dhatura


Ref.:Robbin’s - 686-87
Sol :
- The seeds and fruit of dhatura contain traces of atropine which leads to dilated pupil.
Dilated pupils are seen in poisoning by –
- Amphetamine
- Antihistamines
- Botulin toxin
- Carbon monoxide
- Cocaine
- Cyanide
- Dhatura
- Ephedrine

Correct Answer. a

Copyright © 2014 Delhi Academy of Medical Sciences, All Rights Reserved. 114/118
(292). Which of following environmental condition favors adipocere formation

a. Hot & humid

b. Dry & hot

c. Dry & optimum

d. Humid & optimum

Solution. Ans-: (a) Hot & humid


Ref: K.Narayana reddy - 137
Sol :
Hot and humid – Fatty acids are converted into fatty acids and glycerol in hot and humid cliamate, aided by enzymes of Cl. Welchii.
Adipocere ( Sapification)
1. It is seen in body immersed in water on Damp warm environment.
2. Adipocere, contains
- Palmitic acid
- Oleic acid
- Hydroxy – stearic acid
- Stearic acid
3. The Change is due to grandual hydrolysis and hycholysis and hydrogenation of pre-existing fat. Water for hycholysis is obtained from
Body Tissues
4. Intrinsic Lipasis and Clostridium , perfringens are responsible for adipocere formation.
5. Adipocere is Delayed by cold and formed rapidly by warm humid climate .
6. It has ammonical odour
7. Fresh adipocere is soft, moist, whitish and translucent resembles RANCID BUTTER but old samples are dry, hard, cracked, yellowish
and brittle
It is formed first in Subcutaneous Tissue
- In temperature country , the shortest time for its formation
- In India , it has been observed within 3 days.
- Fetus under 7 months do not show this change

Correct Answer. a

(293). In autopsy spinal cord is opened by following approach

a. Anterolateral

b. Lateral

c. Posterior

d. Anterior

Solution. Ans-: (c) Posterior


Ref: K.Narayana reddy -98
Sol :
· Usually posterior approach, because it is easier. However dripping of blood stained fluid is a problem.
· Less common is anterior approach in which dripping does not occur, but the procedure is more complicated, so rarely undertaken.
· There is nothing called lateral or anterolateral approach

Correct Answer. c

Copyright © 2014 Delhi Academy of Medical Sciences, All Rights Reserved. 115/118
(294). According MCI regulation, medical education granted by Indian universitiescomes under -

a. Schedule I of MCI

b. Schedule II of MCI

c. Part I of Schedule III

d. Part II of Schedule III

Solution. Ans-: (a) Schedule I of MCI


Ref: K.Narayana reddy--19
Sol :
Indian Medical council 1956
- The Indian Medical Council consists of medical practitioners elected from cach state, each university and few nominated by government
of India.
- The First Schedule* of the Act contains the recognized medical qualification granted by universities in India
- The Second Schedule* those granted outside India Schedule I of MCI as well as Part I of Schedule III

Correct Answer. a

(295). Common position of hymentears in a virgin

a. 3 O’clock position

b. 7 o’clock position

c. 10 o’clock position

d. 12 o’clock position

Solution. Ans-: (b) 7 o’clock position.


Ref:Read the text below
Sol:
· The portion of the hymen vulnerable to primary rupture following intercourse are the postro-lateral segments viz 3 O’clock and 7
O’clock positions.
· The second most vulnerable position is the posterior 6 O’clock position.

Correct Answer. b

(296). Which sexual perversion is punishable

a. masochism

b. Voyeurism

c. Tranvestism

d. Fetishism

Solution. Ans-: (b) Voyeurism


Ref:Read the text below
Sol:
·
In 2013, Indian Parliament made amendments to the Indian Penal Code, introducing voyeurism as a criminal offence.
· A man committing the offence of voyeurism would be liable for imprisonment not less than one year and which may extend up to
three years for the first offence, and shall also be liable to fine and for any subsequent conviction would be liable for imprisonment for
not less than three year and which may extend up to seven years and with fine.
· Amended section 354 C IPC. (3 to 7 years of imprisonment)

Correct Answer. b

Copyright © 2014 Delhi Academy of Medical Sciences, All Rights Reserved. 116/118
(297). Dowry deathis covered under which section of IPC

a. 304- A

b. 304- B

c. 176

d. 302

Solution. Ans-: (b) 304- B


Ref:Read the text below
Sol:
This Section of the Indian Penal Code was inserted by a 1986 amendment. The wording of the law states:
Section 304B. Dowry death
(1) Where the death of a woman is caused by any burns or bodily injury or occurs otherwise than under normal circumstances within
seven years of her marriage and it is shown that soon before her death she was subjected to cruelty or harassment by her husband or any
relative of her husband for, or in connection with, any demand for dowry, such death shall be called "dowry death" and such husband or
relative shall be deemed to have caused her death.
Explanation:-For the purpose of this sub-section, "dowry" shall have the same meaning as in section 2 of the Dowry Prohibition Act, 1961
( 28 of 1961).
(2) Whoever commits dowry death shall be punished with imprisonment for a term which shall not be less than seven years but which
may extend to imprisonment for life

Correct Answer. b

(298). Fish tailing of margins in stab wound is seen with:

a. Single edged knife

b. Double edged knife

c. Bayonet

d. None

Solution. Ans-: (a) Single edged knife


Ref:Read the text below
Sol:
· Stab by a single edged weapon causes fish tailing wound
· Stab by a double edged weapon causes spindle shaped wound.
A Bayonet is a sharp weapon (usually double edged) placed in front of gun for close combat.

Correct Answer. a

(299). Spalding sign is seen in

a. Maceration

b. Adipocere

c. Mummification

d. Rigor mortis

Solution. Ans-: (a) Maceration


Ref:Read the text below
Sol:
· Spalding's sign is an indicator of fetal death.
· When fetal death has occurred, abdominal x-ray examination may reveal an overriding of the fetal cranial bones.

Correct Answer. a

Copyright © 2014 Delhi Academy of Medical Sciences, All Rights Reserved. 117/118
(300). First internal sign of Putrefaction is appears at

a. Below the liver

b. In mediastinum

c. Below the spleen

d. In kidney

Solution. Ans-: (a) Below the liver


Ref:Read the text below
Sol:
· In females, ovaries and non-gravid uterus are last to decompose.
· In Males, prostate and testis are last to decompose.
· First external sign of putrefaction is greenish discolouration over caecum and flanks (12-24 hours).
· First signs of internal putrefactionis a greenish discolouration on the undersurface of the liver.

Correct Answer. a

Test Answer
1.(b) 2.(a) 3.(b) 4.(c) 5.(a) 6.(c) 7.(a) 8.(c) 9.(b) 10.(d)

11.(a) 12.(d) 13.(a) 14.(d) 15.(a) 16.(d) 17.(d) 18.(d) 19.(c) 20.(b)

21.(a) 22.(a) 23.(a) 24.(d) 25.(d) 26.(b) 27.(a) 28.(b) 29.(d) 30.(a)

31.(a) 32.(a) 33.(a) 34.(a) 35.(a) 36.(b) 37.(d) 38.(d) 39.(a) 40.(b)

41.(d) 42.(c) 43.(a) 44.(b) 45.(c) 46.(a) 47.(c) 48.(a) 49.(c) 50.(d)

51.(c) 52.(b) 53.(c) 54.(b) 55.(a) 56.(d) 57.(c) 58.(c) 59.(a) 60.(b)

61.(c) 62.(b) 63.(c) 64.(c) 65.(c) 66.(c) 67.(b) 68.(d) 69.(a) 70.(b)

71.(c) 72.(c) 73.(b) 74.(b) 75.(a) 76.(c) 77.(b) 78.(b) 79.(d) 80.(d)

81.(b) 82.(d) 83.(b) 84.(c) 85.(a) 86.(a) 87.(b) 88.(a) 89.(d) 90.(a)

91.(a) 92.(c) 93.(c) 94.(b) 95.(c) 96.(d) 97.(a) 98.(c) 99.(b) 100.(b)

101.(d) 102.(d) 103.(c) 104.(c) 105.(a) 106.(c) 107.(a) 108.(b) 109.(c) 110.(a)

111.(d) 112.(c) 113.(d) 114.(c) 115.(c) 116.(d) 117.(c) 118.(a) 119.(d) 120.(a)

121.(c) 122.(d) 123.(a) 124.(a) 125.(a) 126.(d) 127.(c) 128.(b) 129.(b) 130.(a)

131.(d) 132.(c) 133.(d) 134.(c) 135.(b) 136.(a) 137.(a) 138.(c) 139.(c) 140.(d)

141.(c) 142.(a) 143.(c) 144.(c) 145.(a) 146.(c) 147.(d) 148.(c) 149.(b) 150.(c)

151.(d) 152.(a) 153.(b) 154.(c) 155.(c) 156.(d) 157.(a) 158.(b) 159.(b) 160.(c)

161.(c) 162.(c) 163.(c) 164.(c) 165.(b) 166.(a) 167.(d) 168.(a) 169.(a) 170.(c)

171.(d) 172.(c) 173.(d) 174.(d) 175.(a) 176.(c) 177.(a) 178.(b) 179.(c) 180.(d)

181.(a) 182.(c) 183.(d) 184.(d) 185.(b) 186.(c) 187.(c) 188.(d) 189.(b) 190.(b)

191.(c) 192.(d) 193.(c) 194.(a) 195.(c) 196.(a) 197.(a) 198.(a) 199.(a) 200.(d)

201.(b) 202.(d) 203.(b) 204.(d) 205.(a) 206.(d) 207.(b) 208.(c) 209.(d) 210.(c)

211.(b) 212.(a) 213.(a) 214.(b) 215.(d) 216.(b) 217.(d) 218.(a) 219.(a) 220.(c)

221.(c) 222.(d) 223.(d) 224.(a) 225.(d) 226.(b) 227.(d) 228.(a) 229.(b) 230.(b)

231.(a) 232.(c) 233.(d) 234.(d) 235.(b) 236.(b) 237.(c) 238.(b) 239.(b) 240.(b)

241.(a) 242.(d) 243.(c) 244.(a) 245.(d) 246.(c) 247.(c) 248.(c) 249.(b) 250.(d)

251.(a) 252.(b) 253.(b) 254.(a) 255.(d) 256.(b) 257.(d) 258.(a) 259.(d) 260.(a)

261.(b) 262.(d) 263.(a) 264.(d) 265.(c) 266.(b) 267.(b) 268.(b) 269.(b) 270.(a)

271.(c) 272.(d) 273.(b) 274.(b) 275.(d) 276.(c) 277.(b) 278.(a) 279.(c) 280.(d)

281.(a) 282.(a) 283.(b) 284.(b) 285.(b) 286.(d) 287.(a) 288.(a) 289.(a) 290.(d)

291.(a) 292.(a) 293.(c) 294.(a) 295.(b) 296.(b) 297.(b) 298.(a) 299.(a) 300.(a)

Copyright © 2014 Delhi Academy of Medical Sciences, All Rights Reserved. 118/118

Potrebbero piacerti anche